Sr. Resident Cards Set 2

Lakukan tugas rumah & ujian kamu dengan baik sekarang menggunakan Quizwiz!

A 30-year-old female describes a history of a large lump in her breast that has increased from unnoticeable to its present size over the past 6 weeks. A physical examination demonstrates a well-defined solitary, rubbery, and mobile 4 cm nodule. CNB is performed, and the pathology reveals phyllodes tumor of moderate grade. The next step in management should include A) Serial ultrasound examinations and CBEs every 4 to 6 months B) Cessation of oral contraceptives C) Lumpectomy with at least 1 cm margins D) Lumpectomy with at least 1 cm margins and radiation E) Mastectomy

To demonstrate understanding of the diagnosis and management of phyllodes tumors Answer C is correct Fibroepithelial lesions encompass a spectrum of breast abnormalities, ranging from the fibroadenoma to the phyllodes tumor. Fibroadenomas are the most common benign breast lesions and occur most frequently in the second and third decades of life. Their natural history is one of stability or slow growth. Patients will often give a history of a solitary nontender nodule. Physical examination will usually demonstrate a well-defined, solitary, rubbery, and mobile nodule. Ultrasonography is particularly useful in younger women and demonstrates a well-defined oval or round hypoechoic mass with discrete margins. Mammography should be obtained (in addition to ultrasonography) in women over 35 years of age and typically demonstrates a well-defined radiopaque mass with smooth borders. If the history, physical examination, and imaging are consistent with a fibroadenoma, careful clinical follow-up is reasonable with ultrasonography and breast examination at 6-month intervals to assess the stability of the lesion. When the diagnosis is uncertain, CNB should be performed. For women who request excision of a benign fibroadenoma, enucleation of the lesion is adequate. Conversely, if the lesion increases in size during clinical follow-up, surgical excision is recommended to rule out a phyllodes tumor. Phyllodes tumors are unusual, representing only 0.3 to 0.5% of breast neoplasms. They have the same clinical appearance and imaging characteristics as fibroadenoma, but unlike fibroadenomas, they are characterized by a clinical history of rapid growth and have larger dimensions. Given that these lesions do not have any features on imaging that would be helpful in differentiating them from fibroadenomas, CNB is recommended. However, it can be difficult to differentiate between fibroadenoma and phyllodes tumor on CNB. A pathologic diagnosis of a fibroepithelial lesion on CNB necessitates excision to rule out a phyllodes tumor. Tumors are classified histologically as low, intermediate, or high grade. Although most phyllodes tumors have minimal metastatic potential, they have a proclivity for local recurrence and should be excised with at least a 1 cm margin. Local recurrence has been correlated with excision margins but not with tumor grade or size. The most common site of metastasis from malignant phyllodes tumors is the lungs.

A 23-year-old woman with hyperthyroidism is referred to you for surgical management. All of the following disorders causing hyperthyroidism can be managed surgically except: A) De Quervain thyroiditis B) Graves disease C) Toxic multinodular goiter D) Solitary toxic adenoma E) All of the above are managed surgically

To understand the surgically treatable causes of hyperthyroidism Answer A is correct Multiple causes of hyperthyroidism exist, some of which can be treated surgically. Graves disease, also known as diffuse toxic goiter, results from antibodies to the TSH receptor, which stimulate release of thyroid hormone. Treatment options include medical management (thionamides and beta blockade), radioactive iodine therapy, and thyroidectomy. Solitary toxic adenoma and toxic multinodular goiter cause hyperthyroidism when one or multiple nodules produce thyroid hormone in excess. Solitary toxic adenoma and toxic multinodular goiter may respond to thionamides, but radioiodine or surgery is often required. De Quervain thyroiditis, also known as subacute thyroiditis, is thought to be the result of a viral infection causing thyroid inflammation. Most patients can be treated with nonsteroidal antiinflammatory drugs (NSAIDs) or prednisone, and their symptoms resolve within a few months. Surgical therapy is not indicated.

A 48-year-old man undergoes CT after a motor vehicle accident. This reveals a cystic pancreatic mass. Further workup confirms an intraductal papillary mucinous neoplasm (IPMN). Which of the following IPMNs does not require resection? A) Symptomatic large duct IPMN B) Asymptomatic small duct IPMN with a 4 cm cyst C) Symptomatic small duct IPMN with a 2 cm cyst D) Asymptomatic large duct IPMN E) Asymptomatic small duct IPMN with a 2 cm cyst

Key Concept/Objective: Current management recommendations of intraductal papillary mucinous neoplasm (IPMN) Answer E is correct Large duct IPMN: As this entity is highly premalignant, resection is indicated once the diagnosis is made. Pancreatoduodenectomy is usually required as the disease affects the duct in the head of the pancreas in most cases. Although the duct in the body and tail of the gland may be dilated, this does not mean that it is involved in dysplastic changes as dilation may be secondary to obstruction caused by the viscous mucus in the proximal duct. Therefore, the decision of whether to perform a complete pancreatectomy is made at the time of pancreatoduodenectomy, based on frozen section of the transected pancreatic neck. In most cases, the frozen section will show normal ductal epithelium or only mild or moderate dysplasia. Such patients do not require more pancreas to be resected. Occasionally, carcinoma, carcinoma in situ, or severe dysplasia is found, necessitating the resection of part or all of the remaining pancreas. Small duct IPMN: This entity is less likely to degenerate into malignancy, and the probability of malignant degeneration is related to size. Current indications for resection are cysts exceeding 3 cm in diameter or smaller cysts that are associated with symptoms, internal excrescences, pericyst solid tumor, or pancreatic duct obstruction manifested as a dilation of the duct distal to the cyst. With either type of IPMN, lifelong follow-up with axial imaging is needed. This problem should be thought of as a field defect in the pancreas. Large duct and small duct IPMNs may coexist, and pancreatic intraepithelial neoplasia lesions (dysplastic lesions of the duct) may coexist with IPMN. Most patients who require total pancreatectomy tolerate the procedure well when they are enrolled in a program keyed to this operation. Frank mucinous cancers may appear in patients with IPMN as well; they should be managed in much the same fashion as other adenocarcinomas, with the additional requirement that the resection should encompass the entire IPMN-bearing portion of the pancreas. The mucinous cancers associated with IPMN have a better prognosis than ductal adenocarcinomas do.

A 78-year-old man with metastatic lung cancer develops a symptomatic pericardial effusion. In a patient with a pericardial effusion resulting in cardiac tamponade, which of the following treatment modalities is considered the safest and carries an acceptably low rate of recurrence? A) Image-guided pericardiocentesis B) Subxiphoid pericardiostomy C) Left thoracoscopic pericardiostomy D) Right thoracoscopic pericardiostomy E) Pericardiectomy

Key Concept/Objective: Different options for drainage of a pericardial effusion Answer B is correct. Three procedures are commonly performed for surgical diagnosis and treatment of pericardial effusion: pericardiocentesis, subxiphoid pericardiostomy (pericardial window), and thoracoscopic pericardiostomy (via either the right or the left pleural space). The choice of a surgical approach to the pericardial space depends on the clinical condition of the patient, the presence or absence of associated pleural effusion or other thoracic process, and the underlying diagnosis (if known). Patients with tamponade may decompensate rapidly during the vasodilatation and positive pressure ventilation associated with general anesthesia. Accordingly, careful consideration must be given to the type of anesthesia employed for pericardial drainage procedures. Pericardiocentesis is routinely done with local anesthesia only and may be the best choice in an acutely unstable patient with tamponade. If this option is chosen, however, the choice must be made with the understanding that pericardiocentesis, because of its high recurrence rate and its limited diagnostic capacity, is unlikely to constitute definitive therapy. Subxiphoid pericardiostomy is generally done with initial local anesthesia followed by induction of general anesthesia, and most patients with tamponade can undergo this procedure. The subxiphoid approach provides the hemodynamic benefits of pericardiocentesis, offers the enhanced diagnostic capability of pericardial biopsy, and has a low recurrence rate. Consequently, it is the procedure of choice for patients with tamponade who are stable enough to be transported to the operating suite. Thoracoscopic pericardiostomy has the advantage of enabling simultaneous treatment of pleural processes, which are commonly present in these patients. Ipsilateral pleural and pericardial spaces can be fully explored, pleural effusions can be drained, loculations can be divided, and biopsy specimens can be obtained as needed. The thoracoscopic approach can be especially useful in the case of a known loculated effusion that is limited to one area of the pericardium, in that a pericardial window can be created via either pleural space. This approach also allows resection of a larger segment of pericardium, which may improve the diagnostic yield and reduce the likelihood of recurrent effusion. The major limitation of thoracoscopic pericardiostomy is the need for lung isolation and lateral positioning, which should not be attempted in patients with evidence of tamponade. By weighing the relative risks and benefits of these three procedures, the surgeon can choose the optimal approach for each patient.

You are planning a thymectomy in an otherwise healthy 32-year-old woman with known myasthenia gravis. In a patient with myasthenia gravis, what is considered the least invasive approach for performing a thymectomy? A) Full sternotomy B) Partial sternotomy C) Video-assisted thoracoscopic surgery (VATS) D) Robotic assisted E) Transcervical

Key Concept/Objective: Different options for thymectomy and to evaluate the risks and benefits of each approach in patients with myasthenia gravis Answer E is correct VATS thymectomy has been used to treat myasthenia gravis and thymoma. To date, only anecdotal experience has been reported, and no studies examining long-term outcomes have been published. Although a VATS approach to myasthenia gravis may appear attractive at first, it is not the least invasive approach to the thymus. Transcervical thymectomy is a minimally invasive approach to the thymus that does not require a chest incision, does not violate the pleural space, and allows most patients to be discharged home the next day. There is still a degree of controversy as to whether complete thymic resection, including all ectopic thymic tissue, is necessary to obtain clinical remission in patients with myasthenia gravis. For patients with a thymic mass, we prefer a transsternal approach. It is imperative to maintain oncologic surgical principles; accordingly, en bloc resection is emphasized so that pleural seeding is avoided and the risk of incomplete resection is minimized. For patients with myasthenia gravis who do not have a thymoma, however, there is a need for prospective studies comparing VATS with other surgical approaches to thymectomy.

A 62-year-old man developed an empyema after a severe pneumonia. Decortication is planned. In a patient requiring a decortication, which clinical factor is most predictive of the procedure's success or failure? A) Infectious etiology B) Operative approach (VATS versus open) C) Duration of antibiotic therapy D) Ability or inability of the lung to reexpand E) Calculated predictive postoperative forced expiratory volume in 1 second (ppoFEV1)

Key Concept/Objective: Factors that contribute to the effectiveness of lung decortication Answer D is correct The primary indication for decortication in a patient with fibrothorax is symptomatic pulmonary restriction resulting from the development of a fibrinous peel. The timing of the operation is an important consideration. In many cases, pleuropulmonary processes are self-limiting, and the symptoms resolve over time. As a rule, decortication should be considered (1) if pleural thickening has been present for a substantial period (> 4 to 6 weeks), (2) if respiratory symptoms remain disabling, and (3) if there is radiographic evidence of reversible entrapment of the lung. Decortication is often necessary when lesser interventions have not achieved control of a pleural space infection or have not enabled the lung to reexpand. A condition that poses a major challenge to the thoracic surgeon is pleural space infection with underlying parenchymal disease or airway stenosis. In this setting, decortication is destined to fail: the lung parenchyma will not reexpand to fill the pleural space completely, and any surgical intervention carried out on the diseased lung will only aggravate the underlying disease process. Pleuropneumonectomy may be the only option, although it should be considered a last resort, to be used only if there is significant parenchymal destruction. Decortication may be precluded by invasive uncontrolled pulmonary infection, contralateral pulmonary disease, or a chronically debilitated state that results in a significant or even prohibitive level of operative risk. Medical optimization may be required as an initial step. Often, however, the underlying pleural infection process remains unresolved despite optimal medical management, and surgical intervention becomes necessary in a patient who remains medically fragile. Optimally, the patient's nutritional status should be normalized (with forced feedings, if necessary), and sepsis, if present, should be controlled with appropriate antibiotic therapy. Essentially, pleural cavity infection and fibrosis are problems of residual intrapleural space. The lung cannot expand sufficiently to fill the hemithorax, and as a consequence, the residual space becomes or remains infected. These problems often prove difficult for the thoracic surgeon to address; good surgical judgment is crucial for ensuring optimal outcomes. In planning for decortication, a number of technical issues must be considered, including the timing of intervention (taking into account whether the disease process is chronic or subacute), the quality of the underlying pulmonary parenchyma, the expected ability of the lung to reexpand, the possible need to address residual space issues, and the physiologic status of the patient. The goal of the procedure is to remove the peel from the visceral pleura so as to allow the lung to reexpand and, equally important, to ensure that any potential residual space is obliterated.

A 63-year-old woman with chronic obstructive pulmonary disease (COPD) was found to have a 3 cm right upper lobe mass. Lobectomy is being planned. When evaluating a patient for lung resection, a calculated predictive postoperative forced expiratory volume in 1 second (ppoFEV1) of less than 40% is associated with what perioperative mortality? A) 10% B) 20% C) 30% D) 40% E) 50%

Key Concept/Objective: Key factors for risk stratification of patients in the preoperative workup of patients undergoing pulmonary resection Answer E is correct Pulmonary function testing includes pulmonary spirometry, pulmonary hemodynamic response testing, and exercise testing. Pulmonary spirometry is affected by height, age, weight, sex, race, and posture, as well as arterial oxygenation and diffusion capacity. Although pulmonary spirometry and arterial oxygenation help predict mortality, they are not good predictors of postoperative complications. Diffusion capacity of the lung for carbon monoxide (DLCO) is a more sensitive predictor of postoperative complications. DLCO measures the partial pressure difference between inspired and expired carbon monoxide. It relies on the strong affinity and large absorption capacity of erythrocytes for carbon monoxide and thus demonstrates gas uptake by the capillaries that is less dependent on cardiac output. Thus, value is decreased in patients with emphysema, chronic pulmonary hypertension, and interstitial lung disease. DLCO is an important and independent predictor of postoperative complications after major lung resection, even in patients without COPD. Quantitative ventilation/perfusion (/) scan, along with pulmonary spirometry, is useful for predicting postoperative lung function. A calculated ppoFEV1 of less than 40% is associated with 50% mortality. The absolute minimum ppoFEV1 in patients undergoing lobectomy is 800 mL. Pulmonary hemodynamic response testing includes the measurement of pulmonary artery pressure and pulmonary vascular resistance. It is an invasive test and requires right heart catheterization. Systolic pulmonary artery pressure greater than 35 mm Hg is associated with a 10-fold decrease in survival rate, and pulmonary vascular resistance greater than 190 dyne is associated with 90% mortality. Maximum exercise testing is concerned with the amount of arterial desaturation that occurs during exercise. Patients with a maximum oxygen consumption (VO2 max) less than 15 mL/kg/min are considered high risk, whereas those with a VO2 max of 16 to 20 mL/kg/min could probably undergo surgery.

A 72-year old man underwent a left pneumonectomy for lung cancer and subsequently develops a postpneumonectomy empyema. An Eloesser flap is planned. What is an Eloesser flap? A) Incision and drainage of the chest with elimination of dead space by placement of an intercostal muscle flap B) Incision and drainage of the chest with placement of a thoracostomy tube C) Incision and drainage of the chest with elimination of dead space by placement of a latissimus dorsi muscle flap D) Incision and drainage of the chest, rib resection, and marsupialization of the overlying parietal pleura E) Incision and drainage of the chest with elimination of dead space by placement of a rectus abdominis muscle flap

Key Concept/Objective: Open chest drainage procedures Answer D is correct Indications for open chest drainage include postpneumonectomy empyema or bronchopleural fistula, long-standing empyema in a patient who cannot undergo decortication, and chronic bronchopleural fistula in a high-risk patient. The technique currently employed by most thoracic surgeons follows Symbas's modification of Eloesser's open drainage technique. This procedure has come to be known as the Eloesser flap. Preoperative chest computed tomography is essential for identifying the exact location of the empyema, which determines the placement of the incision. Step 1: initial incision and exposure. The patient is placed in the decubitus position, and a 6 to 8 cm incision is made over the area corresponding to the most dependent area of the infected cavity. Symbas employed a U-shaped incision; however, a simple linear incision can also be used, with good results. The subcutaneous tissue and muscle are then divided down to the chest wall with the electrocautery. Step 2: resection of ribs and creation of thoracostomy. The pleural space is opened with the electrocautery, any pus present is drained, and the chest cavity is manually and visually explored. Next, 6 to 8 cm segments of two or three adjacent ribs are resected according to the same principles employed for other chest wall resections. The resulting thoracostomy is large enough to permit drainage and packing. The skin overlying this thoracostomy is then marsupialized to the thickened parietal pleura with absorbable sutures. If the pleura does not possess sufficient integrity to hold the sutures, they can be placed through the periosteum of the ribs. Step 3: packing and drainage. The wound is irrigated with normal saline and packed with saline-moistened gauze. Postoperatively, a chest x-ray should be obtained to rule out pneumothorax, and twice- to thrice-daily packing is initiated. Packing is continued on an outpatient basis, and the wound is monitored. The wound will begin to close over the next several weeks. If the empyema or bronchopleural fistula has not healed by the time the wound starts closing, the thoracostomy will have to be revised. In some cases, this can be accomplished merely by manually dilating the opening in the operating room; in others, the entire thoracostomy must be revised. In either case, the goal is to maintain a large enough opening to allow adequate packing. Step 4: closure of thoracostomy. Once the lung and the pleural space have healed, the thoracostomy is closed. The procedure for closing the thoracostomy depends on the size and nature of the remaining defect. For small defects, simple closure of the skin will suffice. For larger defects or residual spaces in the pleura, however, muscle flap closure will be required. Improvements in radiographic techniques and greater emphasis on early intervention for empyemas have significantly reduced the need for open chest drainage; however, this technique can still be valuable in the appropriate clinical situation.

In a patient with posthepatic jaundice who does not have evidence of choledocholithiasis on abdominal ultrasonography and who has a clinical history suggestive of malignancy, what imaging modality is considered to be the best initial test for diagnosis, staging, and determining resectability? A) Percutaneous transhepatic cholangiography (PTC) B) Hepatoiminodiacetic acid (HIDA) scan C) Endoscopic ultrasonography (EUS) D) Fine-cut multiphase computed tomography (CT) E) 18F-Fluorodeoxyglucose positron emission tomography (FDG-PET)

Key Concept/Objective: The best imaging studies in patients with possibile biliopancreatic malignancy Answer D is correct Many imaging modalities are currently used to determine resectability of biliopancreatic malignancy, and several of these have been established as effective alternatives to direct cholangiography because they involve little, if any, morbidity. Their accuracy varies according to the underlying pathology and the expertise of the user. They have been studied mostly with respect to the staging and diagnosis of pancreatic, periampullary, and biliary hilar cancers. For determining resectability and staging lesions before operation, most surgeons rely mainly on spiral CT. The advent and widespread availability of multidetector CT have made this modality the dominant second-line imaging method in cases of suspected pancreatic masses. For optimal evaluation of the pancreas, a fine-cut dual-phase (arterial phase and portal venous phase) scan should be obtained. At present, spiral CT is considered to be superior for the diagnosis and staging of lesions such as pancreatic cancer. It exhibits a high negative predictive value and has a false positive rate of less than 10%; its sensitivity is optimal for pancreatic lesions larger than 1.5 cm in diameter. Ascites, liver metastases, lymph nodes larger than 2 cm in diameter, and invasion into adjacent organs are all signs of advanced disease. On the basis of these criteria, spiral CT can predict that a lesion will not be resectable with an accuracy approaching 95%; however, as many as 33% of tumors that appear to be resectable on CT are found to be unresectable at operation. Magnetic resonance imaging (MRI)-based staging, along with magnetic resonance cholangiopancreatography (MRCP), can further dictate the subsequent choice of therapy. MRI may be particularly useful for following up patients in whom clip artifacts interfere with a CT image. It also appears to be successful in detecting cholangiocarcinoma spreading along the proximal biliary tree. Only in a few very rare instances is traditional angiography used to assess resectability or stage a hepatobiliary or pancreatic neoplasm. Increasingly, it is being replaced by CT angiography or duplex Doppler ultrasonography, which can confirm the presence of flow in the hepatic arterial or portal venous systems and occasionally can demonstrate invasion of these vessels by tumor. EUS is a highly sensitive method of imaging the pancreas and the duodenum. In two large studies, it was found to be superior to CT and standard ultrasonography in staging pancreatic and ampullary cancers. Subsequent studies indicated that whereas EUS is superior to CT for detection and staging, it provides similar information regarding nodal status and overall assessment of resectability. From a cost minimization point of view, the optimal strategy is to begin with dual-phase CT and to follow up with EUS only in cases in which further information or a tissue diagnosis is required. In the event that none of these modalities point to a diagnosis, the use of FDG-PET may be considered to help differentiate benign pancreatic conditions from malignant ones. Besides facilitating diagnosis, FDG-PET provides information regarding occult metastases and can be useful in detecting recurrent disease. Experience with FDG-PET is growing rapidly as this imaging modality becomes more readily accessible.

Patients with pancreatic or biliary adenocarcinoma may have an elevated serum concentration of the tumor marker CA 19-9. Which of the following is true regarding CA 19-9 levels in patients with pancreatic or biliary adenocarcinoma? A) CA 19-9 is higher in patients with biliary than with pancreatic adenocarcinoma B) CA 19-9 levels always remain elevated even after tumor resection C) Concentrations higher than 10 U/mL are highly suggestive of malignancy D) Serum levels reflect the extent of the tumor E) CA 19-9 levels are rarely associated with pancreatic or biliary adenocarcinoma

Key Concept/Objective: The clinical utility of a CA 19-9 value Answer D is correct Serum concentrations of the tumor marker CA 19-9 are often elevated in patients with pancreatic or biliary adenocarcinomas. The upper limit of the normal range is 37 U/mL. Concentrations higher than 100 U/mL are highly suggestive of malignancy, but elevations between 37 and 100 U/mL are less specific. Serum levels generally reflect the extent of the tumor: small tumors (1 cm in diameter) are rarely associated with levels higher than 100 U/mL, whereas very high concentrations (> 1,000 U/mL) suggest metastatic disease. High levels may also accompany cholangitis, but these levels should subside to normal with relief of obstruction or infection, when malignancy is absent. Measurement of CA 19-9 concentrations may be employed to detect recurrences. In patients who have elevated CA 19-9 levels that return to normal after tumor resection; a second rise in the CA 19-9 level in the follow-up period is indicative of recurrence.

A 62-year-old woman underwent laparoscopic cholecystectomy for presumed chronic cholecystitis. Pathology revealed adenocarcinoma of the gallbladder with tumor invading through the lamina propria. What is the most appropriate next step in management? A) No further surgical therapy is required B) Reoperation to complete an extraserosal resection (removal of the fibrous liver plate) C) Resection of segments 4b/5 of the liver D) Resection of segments 4b/5 of the liver along with portal/celiac lymph nodes E) Resection of segments 4 to 8 of the liver

Key Concept/Objective: The management of gallbladder cancer by stage Answer A is correct The excised specimen should be inked and a frozen section obtained. If there is gallbladder cancer in the specimen but the resection margins are clear and the tumor is a T1 lesion (i.e., has not penetrated the muscularis), the procedure is considered complete in that lymph node metastases are uncommon with T1 tumors (incidence < 10%). However, lymph node metastases are present in 50% of patients with T2 lesions (i.e., tumors that have invaded the muscularis). Therefore, if margins are positive or the tumor is a T2 lesion, resection of segments 4b and 5 of the liver and dissection of the portal and celiac lymph nodes are recommended. Resection of the extrahepatic bile duct and hepaticojejunostomy may be needed in some cases to obtain a complete node clearance, but this is being done less frequently as experience with portal node clearance has been obtained. If it is already clear at the commencement of the operation that the tumor is T2, one should proceed directly to liver, lymph node, and bile duct resection. In more advanced stages of disease (T3 and T4), the aim is still excision with clear margins and resection of portal and celiac lymph nodes. To obtain clear local margins with these tumors, in addition to what is required for T2 tumors, more extensive hepatic resections—up to a trisectionectomy—may be necessary, as well as resection of adjacent organs. Gallbladder cancer may be an incidental finding at laparoscopic cholecystectomy, as it has been at open cholecystectomy. The incidence of this finding ranges from 0.3 to 1.0%. A concern that has arisen in the current era, in which the laparoscopic approach to cholecystectomy is dominant, is the risk of port-site implantation of tumor. Port-site implantation is the result of contact between the malignancy and the tissues surrounding the port site at the time of gallbladder extraction. Therefore, when evidence of gallbladder wall thickening is noted intraoperatively, the gallbladder should be extracted in a sac. The gallbladder should be inspected at the time of extraction, and any questionable areas should undergo biopsy.

A 32-year-old woman presents with intermittent right upper quadrant pain and jaundice. Total bilirubin is 4.3 mg/dL, and a sonogram shows gallstones and intrahepatic ductal dilation. Endoscopic retrograde cholangiopancreatography (ERCP) is planned. In patients undergoing ERCP, what is the approximate risk of postprocedure pancreatitis or cholangitis? A) 0.4 to 0.7% B) 4 to 7% C) 14 to 17% D) 24 to 27% E) 34 to 37%

Key Concept/Objective: The risk associated with endoscopic retrograde cholangiopancreatography (ERCP) Answer B is correct Of the many imaging methods available today, the gold standard for defining the level of a biliary obstruction before operation in a jaundiced patient remains direct cholangiography, which can be performed either via ERCP or via percutaneous transhepatic cholangiography (PTC). Unlike other imaging modalities, direct cholangiography poses significant risks to the patient: there is a 4 to 7% incidence of pancreatitis or cholangitis after ERCP, and there is a 4% incidence of bile leakage, cholangitis, or bleeding after PTC. There are also several risks that are particular to the manipulation of an obstructed biliary system. For these reasons, the role of ERCP and PTC is increasingly a therapeutic one; therefore, it is important to gather as much imaging information as possible on the likely cause of the jaundice before performing either investigation.

A 78-year-old man presents with the complaint of dysphagia. What feature in the history of a patient being evaluated for dysphagia suggests that the underlying cause may be esophageal cancer? A) Food is "sticking" B) Intolerance to liquids and solids C) Quickly progressing course D) Pain localized to the retrosternal region E) Odynophagia

Key Concept/Objective: To recognize a history suggestive of esophageal cancer as a cause of dysphagia Answer C is correct Dysphagia is a common complaint, particularly in patients 50 and older. A thorough history allows for an accurate assessment of the likely etiology. Symptoms that have been present for a short time or symptoms that are rapidly progressive suggest a malignant etiology of dysphagia. A sensation of food "sticking" suggests an esophageal cause of dysphagia, as opposed to an oropharyngeal cause, but does not suggest an etiology. Intolerance to liquids and solids implicates a functional or neuromuscular cause of dysphagia. Although the patient history can help localize a culprit esophageal lesion causing dysphagia to within 4 cm, location alone does not suggest an etiology of dysphagia. Odynophagia is most often caused by infectious or inflammatory etiologies.

A 71-year-old man presents to the emergency department with several days of left lower quadrant pain, fever to 38.5°C (101.4°F), and urinary urgency. On further questioning, he reports episodes of pneumaturia. A diagnosis of diverticulitis with colovesical fistula is suspected. Which of the following statements about colovesical fistula is not true? A) Women are less likely to experience colovesical fistula because the uterus protects the bladder B) Cystoscopy is required to confirm the diagnosis C) Possible CT findings include thickening of the bladder, air in the bladder, and sigmoid diverticula D) Resection of a colovesical fistula is usually undertaken given the associated risk of urinary sepsis E) None of the above

To demonstrate knowledge about the evaluation of colovesical fistulas resulting from diverticulitis Answer B is correct Some diverticular abscesses rupture into adjacent tissues or viscera, resulting in the formation of fistulas. The fistulas most commonly seen in this setting (50 to 65% of cases) are colovesical fistulas. This complication is less common in women because of the protection afforded by the uterus. Symptoms of colovesical fistulas tend to involve the urinary tract (e.g., pneumaturia, hematuria, and urinary frequency). When a colovesical fistula occurs, contrast CT with narrow cuts in the pelvis can be very helpful. The classic findings are sigmoid diverticula, thickening of the bladder and colon, air in the bladder, opacification of the fistula tract and bladder, and, possibly, an abscess. Cystoscopy is less specific, showing possible edema or erythema at the site of the fistula. Colovesical fistulas are usually resected because of the risk of urinary sepsis and the concern that a malignancy might be overlooked. Preferably, the operation is done when the acute inflammation has subsided.

A 23-year-old woman is referred to you with toxic multinodular goiter, and a total thyroidectomy is planned. You decide to initiate medical therapy to prepare her for surgery. Which of the following is true regarding medical therapy for hyperthyroidism? A) PTU also inhibits peripheral conversion of T3 to the metabolically more active T4 B) Agranulocytosis affects 10 to 20% of patients receiving thionamides (PTU and methimazole) C) Most patients are rendered euthyroid within 6 weeks of treatment initiation D) PTU and methimazole limit thyroid hormone synthesis by disrupting the oxidation of iodine E) Beta blockers reduce the effect of thionamides, and the two classes of drugs should not be used in combination

To demonstrate knowledge of the medical treatment for hyperthyroidism Answer C is correct Achieving a euthyroid state preoperatively is of utmost importance as it reduces the likelihood of thyroid storm intra- or postoperatively. The thionamides, PTU and methimazole, limit thyroid hormone synthesis by disrupting organification of iodine. PTU also inhibits peripheral conversion of T4 to the metabolically more active T3. Minor side effects (pruritis, fever, gastrointestinal disturbances, and arthralgia) occur in 5 to 25% of patients receiving thionamides, and agranulocytosis affects 0.2 to 0.5% of patients receiving either medication. Rare cases of fulminant hepatic failure and autoimmune vasculitis have been associated with PTU alone. Beta blockers (atenolol, propranolol) may be added to antithyroid medications for more rapid control of severe adrenergic symptoms when present. Treatment efficacy is monitored by thyroid function tests and clinical evaluation (resolution of tachycardia and tremor). Most patients are rendered euthyroid within 6 weeks of treatment initiation; virtually all are euthyroid at 3 months. PTU is often chosen over methimazole during pregnancy as methimazole is more apt to cross the placenta and has been linked to fetal scalp aplasia cutis.

A 23-year-old woman is referred to you with toxic multinodular goiter, and a total thyroidectomy is planned. You decide to initiate medical therapy to prepare her for surgery. Which of the following is true regarding medical therapy for hyperthyroidism? A) PTU also inhibits peripheral conversion of T3 to the metabolically more active T4 B) Agranulocytosis affects 10 to 20% of patients receiving thionamides (PTU and methimazole) C) Most patients are rendered euthyroid within 6 weeks of treatment initiation D) PTU and methimazole limit thyroid hormone synthesis by disrupting the oxidation of iodine E) Beta blockers reduce the effect of thionamides, and the two classes of drugs should not be used in combination

To demonstrate knowledge of the medical treatment for hyperthyroidism Answer C is correct. Achieving a euthyroid state preoperatively is of utmost importance as it reduces the likelihood of thyroid storm intra- or postoperatively. The thionamides, PTU and methimazole, limit thyroid hormone synthesis by disrupting organification of iodine. PTU also inhibits peripheral conversion of T4 to the metabolically more active T3. Minor side effects (pruritis, fever, gastrointestinal disturbances, and arthralgia) occur in 5 to 25% of patients receiving thionamides, and agranulocytosis affects 0.2 to 0.5% of patients receiving either medication. Rare cases of fulminant hepatic failure and autoimmune vasculitis have been associated with PTU alone. Beta blockers (atenolol, propranolol) may be added to antithyroid medications for more rapid control of severe adrenergic symptoms when present. Treatment efficacy is monitored by thyroid function tests and clinical evaluation (resolution of tachycardia and tremor). Most patients are rendered euthyroid within 6 weeks of treatment initiation; virtually all are euthyroid at 3 months. PTU is often chosen over methimazole during pregnancy as methimazole is more apt to cross the placenta and has been linked to fetal scalp aplasia cutis.

A 30-year-old female experiences 14 hours of right lower quadrant pain and is taken to the operating room with the preoperative diagnosis of appendicitis. Exploratory laparoscopy reveals a 4 cm simple right ovarian cyst, torsion of the right adnexa, and an injected, edematous right ovary What is the most appropriate next step in management? A) Right ovarian cystectomy B) Right ovarian cystectomy and detorsion of the right adnexa C) Right ovarian oopherectomy D) Right ovarian salpingo-oopherectomy E) Detorsion of the right adnexa

To demonstrate understanding of management of adnexal torsion Answer B is correct Abnormal enlargement of the adnexa, either by neoplasms or by cysts, may predispose to torsion of the tube and the ovary around the infundibulopelvic ligament, resulting in vascular compromise. Adnexal torsion is a gynecologic emergency: if untreated, it will result in infarction of the adnexa with eventual peritonitis. Although torsion can occur in postmenopausal women, it is most common in patients of reproductive age. The presenting complaint is severe pelvic pain of abrupt onset, often accompanied by nausea and vomiting of sudden onset. Physical examination often demonstrates an acute abdomen with lower abdominal tenderness and guarding. Pelvic examination usually demonstrates a mass that may be difficult to assess because of tenderness. Ultrasonography demonstrates a mass in nearly all affected patients. Computed tomography may demonstrate ovarian enlargement, smooth wall thickening of the mass or tube, and uterine deviation toward the affected adnexa. In the past, adnexal torsion was commonly treated aggressively, with salpingo-oophorectomy. This approach continues to be warranted in perimenopausal and postmenopausal patients; however, a more conservative approach is now considered preferable for the treatment of women of reproductive age who desire future fertility. The traditional approach was based on two assumptions: first, that the twisted adnexum with vascular compromise was nonviable, and second, that detorsion was potentially dangerous because it might cause thrombus to be released from the clotted veins in the infundibulopelvic ligament. These assumptions proved to be unfounded. Accordingly, current management begins with untwisting the adnexum and assessing its viability. Once the torsion is unwound, the adnexum may demonstrate reperfusion or infarction without vascular recovery. Only a gangrenous adnexum must be completely removed. Patients may be managed by means of either laparoscopy or laparotomy, depending on the clinical circumstances.

A 30-year-old female experiences 14 hours of right lower quadrant pain and is taken to the operating room with the preoperative diagnosis of appendicitis. Exploratory laparoscopy reveals a 4 cm simple right ovarian cyst, torsion of the right adnexa, and an injected, edematous right ovary. What is the most appropriate next step in management? A) Right ovarian cystectomy B) Right ovarian cystectomy and detorsion of the right adnexa C) Right ovarian oopherectomy D) Right ovarian salpingo-oopherectomy E) Detorsion of the right adnexa

To demonstrate understanding of management of adnexal torsion Answer B is correct Abnormal enlargement of the adnexa, either by neoplasms or by cysts, may predispose to torsion of the tube and the ovary around the infundibulopelvic ligament, resulting in vascular compromise. Adnexal torsion is a gynecologic emergency: if untreated, it will result in infarction of the adnexa with eventual peritonitis. Although torsion can occur in postmenopausal women, it is most common in patients of reproductive age. The presenting complaint is severe pelvic pain of abrupt onset, often accompanied by nausea and vomiting of sudden onset. Physical examination often demonstrates an acute abdomen with lower abdominal tenderness and guarding. Pelvic examination usually demonstrates a mass that may be difficult to assess because of tenderness. Ultrasonography demonstrates a mass in nearly all affected patients. CT may demonstrate ovarian enlargement, smooth wall thickening of the mass or tube, and uterine deviation toward the affected adnexa. In the past, adnexal torsion was commonly treated aggressively, with salpingo-oophorectomy. This approach continues to be warranted in perimenopausal and postmenopausal patients; however, a more conservative approach is now considered preferable for the treatment of women of reproductive age who desire future fertility. The traditional approach was based on two assumptions: first, that the twisted adnexum with vascular compromise was nonviable, and second, that detorsion was potentially dangerous because it might cause thrombus to be released from the clotted veins in the infundibulopelvic ligament. These assumptions proved to be unfounded. Accordingly, current management begins with untwisting the adnexum and assessing its viability. Once the torsion is unwound, the adnexum may demonstrate reperfusion or infarction without vascular recovery. Only a gangrenous adnexum must be completely removed. Patients may be managed by means of either laparoscopy or laparotomy depending on the clinical circumstances.

A 44-year-old female is referred to you for eczematoid changes of the nipple that failed to improve with multiple topical treatments, including steroids. On examination, the nipple is beet red and ulcerated. There is no palpable mass. Which of the following statements regarding the management of this problem is false? A) She requires biopsy of the nipple B) She requires imaging of the breast C) Chemotherapy should be the first modality of treatment D) She likely has DCIS within the breast E) She will likely require surgery

To demonstrate understanding of the diagnosis and management of Paget disease of the breast Answer C is correct. An uncommon presentation of breast cancer is that of eczematoid changes directly in the nipple, often with beet red and ulcerated skin in the nipple. This is due to malignant cells with a characteristic "fried egg" appearance occurring in the nipple skin or "Paget cells." This is Paget disease of the nipple. In most cases of Paget disease, there is a cancer in the underlying breast, and the cells in the nipple may represent intraductal spread of the cancer to the nipple, although this process is poorly understood. In most cases, the underlying cancer is relatively close to the nipple and in most cases is DCIS. Women with these changes in the nipple are often misdiagnosed as having benign skin conditions and are treated with topical agents, including steroids. If such treatments are used and the lesion does not resolve, or if there is any clinical suspicion, a biopsy of the affected skin is needed. If the biopsy shows Paget disease, then the breast should be carefully imaged by mammography, supplemented by magnetic resonance imaging (MRI), if the breast is dense to search for primary tumor in the breast. If a breast lesion is found, it, too, should be biopsied. Treatment is generally based on the type (in situ or invasive) of primary tumor in the breast and treatment of the nipple. This may be accomplished in many cases with removal of the nipple in continuity with the breast cancer, followed by radiation. Mastectomy is still often used but may be reserved for cases with more extensive involvement. Lymph node surgery is warranted only with invasive cancer. Adjuvant systemic therapy is administered by standard guidelines.

A 58-year-old woman is referred to you for a 2 cm upper outer quadrant invasive ductal carcinoma. She has no palpable axillary or cervical lymphadenopathy. Which of the following statements regarding this case is false? A) She should undergo lumpectomy with sentinel lymph node biopsy B) The axilla contains an average of 15 to 20 lymph nodes C) The risk of lymphedema is about 10% with axillary dissection D) Women with clinically positive lymph nodes should have full axillary lymph node dissection E) Resection of positive axillary nodes prevents recurrence and prolongs survival

To demonstrate understanding of management of the axilla in the surgical treatment of breast cancer Answer E is correct The presence of metastases in regional lymph nodes is a key prognostic factor used in making treatment decisions in patients with breast cancer. In addition, removing lymph nodes prevents recurrence risk in the regional lymph node basin. However, treatment of lymph nodes does not itself affect survival. Defining the involvement of regional lymph nodes requires surgical resection. No noninvasive test of the regional lymph nodes has proven insufficient sensitivity and specificity to replace surgical resection. Therefore, surgical evaluation of regional nodes remains a key component of breast cancer treatment. The axilla contains an average of 15 to 20 lymph nodes extending under the axillary vein medially toward the thoracic inlet. The risk of lymphedema is about 10% with axillary dissection alone and substantially higher when radiation is also required after node dissection. Sentinal lymph node biopsy is appropriate for women with clinically negative nodes. Those with clinically positive nodes are not candidates for sentinal lymph node biopsy and should have full axillary lymph node dissection. If there is a question about nodes, involvement of nodes may be confirmed before surgery using ultrasound-guided needle biopsy (CNB or fine-needle aspiration). Sentinel lymph node biopsy is performed by injecting radiolabeled tracer and/or vital dyes that drain to and are trapped in the initial lymph nodes in the chain. The injection is generally in the periareolar skin and under the nipple-areola complex. This identifies an average of three lymph nodes, which are then removed and examined for metastases. Sentinal lymph node biopsy is appropriate in virtually all women with invasive breast cancer clinically negative nodes. Sentinal lymph node biopsy may be performed in women with local recurrence after previous breast conservation surgery and radiation, although the accuracy of sentinal lymph node biopsy in this setting has never been tested.

A 56-year-old male with T3N1 low rectal cancer has completed a course of neoadjuvant chemotherapy and radiation and is seen in your office to discuss a planned low anterior resection. Given the low nature of the anastomosis and the radiated field, you have wisely chosen to perform a temporary diverting loop ostomy. Which of the following statements is not true? A) Preoperative enterostomal consultation can correct misconceptions about ostomies and reduce anxiety B) Loop ileostomies are associated with a lower incidence of postoperative intestinal obstruction than are loop colostomies C) Loop ileostomies are associated with a lower incidence of complications than are loop colostomies D) The stoma should be brought out through a separate opening in the abdominal wall and not through the main incision E) Ileostomies are generally sited in the right iliac fossa

To demonstrate understanding of ostomy planning and creation Answer B is correct Defunctioning of a distal anastomosis after rectal excision and anastomosis may be achieved with either a loop ileostomy or a loop transverse colostomy. A number of nonrandomized studies and randomized, controlled trials have been performed in an effort to determine which of these two approaches is superior. Both types of stoma effectively defunction the distal bowel; however, loop ileostomy appears to be associated with a lower incidence of complications related to stoma formation and closure, although it may also carry a higher risk of postoperative intestinal obstruction. The two types of stoma are comparable with respect to patient quality of life, and the degree of subsequent social restriction is influenced more by the number and type of complications than by the type of stoma formed. A stoma should be brought out through a separate opening in the abdominal wall, not through the main incision: there is a high incidence of wound infection and incisional hernia formation if the main incision is used as a stoma site. In general, ileostomies are sited in the right iliac fossa, sigmoid colostomies (loop or end) in the left iliac fossa, and transverse loop colostomies in either the right or the left upper quadrant. These positions are preferred because they are conveniently close to the particular bowel segments to be used for creating the various stomas. As needed, however—as when finding a suitable site proves difficult because of previous scars or deformity—both the ileum and the colon can be mobilized to provide sufficient length to reach most sites on the abdominal wall.

A 72-year-old bed-bound nursing home patient presents with a gangrenous left foot. He has previously undergone a left femoropopliteal bypass, which has occluded. No further revascularization options remain, and amputation is being considered. Which of the following is true regarding lower extremity amputation in this patient? A) Transcutaneous oxygen tension can be used to determine that an amputation will not heal. B) A below-the-knee amputation is ideal for this patient. C) In this patient, a transmetatarsal amputation has a high likelihood of success. D) Guillotine amputation above the ankle followed by definitive above-the-knee amputation is a treatment option. E) Mortality from a lower extremity amputation has a mortality of less than 5%.

To demonstrate understanding of planning for lower extremity amputation Answer D is correct Selecting the appropriate level of amputation is of primary importance for healing and preservation of function. For an ambulatory patient who has either a palpable pulse over the dorsal pedal or posterior tibial artery or a functioning infrainguinal arterial bypass graft, amputation on the foot (either toe amputation or transmetatarsal amputation) is appropriate. For an ambulatory patient who has a palpable femoral pulse and a patent deep femoral (profunda femoris) artery, whose skin is warm at least to the level of the ankle, and who has no skin lesions on the proposed amputation flaps, amputation below the knee is appropriate. For a nonambulatory patient who has ischemic rest pain, ulceration, or gangrene, amputation above the knee is appropriate. Arterial reconstruction is not indicated if the extremity is nonfunctional. Below-the-knee amputation does not offer nonambulatory patients any functional advantage; moreover, it is less likely to heal and often results in a flexion contracture at the knee that leads to pressure ulceration of the stump. Above-the-knee amputation depends on pulsatile flow into the ipsilateral internal iliac artery for successful healing. Above-the-knee amputation is also necessary for a patient whose skin is cool at or above the midcalf or who has skin lesions at or proximal to the midcalf. Several adjunctive measurements (e.g., transcutaneous oxygen tension and segmental arterial pressure) have been used to select the level of amputation but have not proved particularly helpful. Generally, these adjuncts can reliably determine a level of amputation at which healing is virtually ensured, but they cannot reliably determine the level at which an amputation will not heal. Consequently, reliance on such measures to select the level of amputation will result in an unnecessarily high percentage of more proximal amputations. In most cases, definitive amputation can be accomplished in a single stage. Local cellulitis can usually be controlled beforehand with bed rest and systemic administration of antibiotics. Undrained pus or recalcitrant cellulitis, however, must be treated with débridement and drainage in advance of definitive amputation. This can be accomplished with local soft tissue débridement, single-toe open amputation, or guillotine amputation across the ankle. Careful preoperative medical assessment is essential. Lower extremity amputation for ischemia is associated with a mortality of 4.5 to 18% owing to the poor overall condition of the patient population. Accordingly, optimization of cardiac and pulmonary function and control of systemic infection are mandatory.

A 72-year-old bed-bound nursing home patient presents with a gangrenous left foot. He has previously undergone a left femoropopliteal bypass, which has occluded. No further revascularization options remain, and amputation is being considered. Which of the following is true regarding lower extremity amputation in this patient? A) Transcutaneous oxygen tension can be used to determine that an amputation will not heal B) A below-the-knee amputation is ideal for this patient. C) In this patient, a transmetatarsal amputation has a high likelihood of success. D) Guillotine amputation above the ankle followed by definitive below-the-knee amputation is a treatment option. E) Mortality from a lower extremity amputation has a mortality of less than 5%.

To demonstrate understanding of planning for lower extremity amputation Answer D is correct Selecting the appropriate level of amputation is of primary importance for healing and preservation of function. For an ambulatory patient who has either a palpable pulse over the dorsal pedal or posterior tibial artery or a functioning infrainguinal arterial bypass graft, amputation on the foot (either toe amputation or transmetatarsal amputation) is appropriate. For an ambulatory patient who has a palpable femoral pulse and a patent deep femoral (profunda femoris) artery, whose skin is warm at least to the level of the ankle, and who has no skin lesions on the proposed amputation flaps, amputation below the knee is appropriate. For a nonambulatory patient who has ischemic rest pain, ulceration, or gangrene, amputation above the knee is appropriate. Arterial reconstruction is not indicated if the extremity is nonfunctional. Below-the-knee amputation does not offer nonambulatory patients any functional advantage; moreover, it is less likely to heal and often results in a flexion contracture at the knee that leads to pressure ulceration of the stump. Above-the-knee amputation depends on pulsatile flow into the ipsilateral internal iliac artery for successful healing. Above-the-knee amputation is also necessary for a patient whose skin is cool at or above the midcalf or who has skin lesions at or proximal to the midcalf. Several adjunctive measurements (e.g., transcutaneous oxygen tension and segmental arterial pressure) have been used to select the level of amputation but have not proved particularly helpful. Generally, these adjuncts can reliably determine a level of amputation at which healing is virtually ensured but cannot reliably determine the level at which an amputation will not heal. Consequently, reliance on such measures to select the level of amputation will result in an unnecessarily high percentage of more proximal amputations. In most cases, definitive amputation can be accomplished in a single stage. Local cellulitis can usually be controlled beforehand with bed rest and systemic administration of antibiotics. Undrained pus or recalcitrant cellulitis, however, must be treated with débridement and drainage in advance of definitive amputation. This can be accomplished with local soft tissue débridement, single-toe open amputation, or guillotine amputation across the ankle. Careful preoperative medical assessment is essential. Lower extremity amputation for ischemia is associated with a mortality of 4.5 to 18% owing to the poor overall condition of the patient population. Accordingly, optimization of cardiac and pulmonary function and control of systemic infection are mandatory.

A 48-year-old woman complains of a new neck mass. Ultrasonography reveals a 3 cm solid mass in the left thyroid lobe. Which of the following is the most common type of malignant thyroid cancer? A) Follicular thyroid carcinoma (FTC) B) Anaplastic thyroid carcinoma (ATC) C) Primary thyroid lymphoma (PTL) D) Papillary thyroid carcinoma (PTC) E) Medullary thyroid carcinoma (MTC)

To demonstrate understanding of the common forms of malignant thyroid cancer Answer D is correct PTC is the most common thyroid cancer and carries an excellent prognosis when treated appropriately (> 90% 10-year survival overall). Poor prognostic factors include age less than 20 or more than 60 years, tumor greater than 4 cm, extrathyroidal extension, and the presence of distant metastases. FTC comprises 10 to 15% of thyroid malignancies. In contrast to PTC, FTC is typically solitary and metastasis is more often hematogenous than lymphatic, favoring lung and bone. Poor prognostic factors include age over 50 years and tumors over 3.5 cm. ATC is rare in the United States, representing less than 2% of thyroid malignancies. The median survival for patients with ATC is approximately 6 months. ATC is suspected when older patients present with rapidly progressive, bulky, fixed tumors. MTC is a neuroendocrine tumor that arises from the calcitonin-secreting parafollicular cells. MTC constitutes about 5% of thyroid cancers; most (75%) cases are sporadic, whereas the remainder are hereditary. PTL accounts for approximately 1% of thyroid malignancies and 1% of extranodal lymphomas.

A chest x-ray is obtained in an elderly gentleman who was in a motor vehicle collision. No trauma-related injury is observed, but the chest x-ray demonstrates a lesion in the left lower lobe. Which of the following is a characteristic of a solitary pulmonary nodule? A) Size greater than 3 cm in diameter B) Associated adenopathy C) Completely surrounded by pulmonary parenchyma D) Associated atelectasis E) Pleural effusion

To demonstrate understanding of the definition of a solitary pulmonary nodule Answer C is correct. A solitary pulmonary nodule is a common finding. It is defined as a single radiographically visible pulmonary lesion that is less than 3 cm in diameter, is completely surrounded by pulmonary parenchyma, and is not associated with atelectasis or adenopathy. Any pulmonary lesion larger than 3 cm is considered a mass.

A 44-year-old female is referred to you for a 1.4 cm solid left adrenal lesion that was found incidentally for the workup of fibroids. She has no other known medical problems and complains only of headache and fatigue. In your office, her blood pressure is 181/90 mm Hg and her serum potassium is 2.8 mEq/L. Which of the following statements regarding her condition is true? A) The most likely diagnosis is Cushing syndrome B) The most likely diagnosis is secondary hyperaldosteronism C) She should undergo bilateral adrenal vein sampling of aldosterone and cortisol D) Alkalosis is common in this scenario E) She should undergo right adrenalectomy if her plasma aldosterone-to-renin ratio is significantly elevated

To demonstrate understanding of the diagnosis and management of primary hyperaldosteronism (Conn syndrome) Answer D is correct Primary hyperaldosteronism (Conn syndrome) occurs in as many as 8 to 12% of hypertensive patients. The diagnosis of primary hyperaldosteronism should be considered in any patient who has hypertension of early onset, hypertension that is difficult to control, or hypertension with hypokalemia. The pathophysiology primarily consists of increased aldosterone secretion by the adrenal gland, which leads to increased sodium retention, expansion of intravascular fluid volume, and suppression of renin secretion by the kidney. Aldosterone also promotes the exchange of sodium for potassium and hydrogen ion in the distal tubule, which leads to potassium depletion and alkalosis. The alkalosis may be aggravated by the movement of hydrogen ion into the cells to replace intracellular potassium. The initial biochemical evaluation for suspected primary hyperaldosteronism should consist of measurement of the plasma aldosterone concentration (PAC) and assessment of plasma renin activity (PRA). A PAC-to-PRA ratio greater than 25 to 30, in conjunction with suppressed PRA (< 0.2 to 0.5 ng/mL/hr) and a PAC higher than 15 ng/dL, is consistent with primary hyperaldosteronism. Once the diagnosis of primary hyperaldosteronism has been confirmed biochemically, the next step is to identify the cause or determine the subtype. Historically, aldosterone-secreting adenomas (aldosteronomas) have accounted for approximately 60% of cases, and idiopathic hyperaldosteronism from bilateral cortical hyperplasia has accounted for the bulk of the remaining cases. More recent data, however, suggest that only 30 to 50% of patients with primary hyperaldosteronism had aldosteronomas, whereas 50 to 70% had bilateral adrenal hyperplasia. It is important to distinguish between these two conditions because aldosteronomas are treated surgically and idiopathic hyperaldosteronism is treated medically.

You are referred a 51-year-old female with an 11 cm right adrenal mass that was discovered on a CT scan for the workup of right flank pain. On closer review of the imaging study, the mass appears to invade the kidney and possibly the vena cava. Which of the following statements regarding this disease is false? A) This disease is rarely associated with cortisol or androgen hypersecretion B) Mitotane is the most specific chemotherapeutic agent available for this disease C) Complete surgical resection is the only treatment modality that can provide cure D) The clinical presentation of this disease is often associated with the large size of the mass E) The presence of tumor thrombus in the renal vein or inferior vena cava is not a contraindication for resection

To demonstrate understanding of the diagnosis and management of adrenocortical carcinoma Answer A is correct Adrenocortical carcinomas are typically large (> 6 to 8 cm) at presentation, and most patients present with advanced disease (i.e., stage III or IV). The clinical presentation may be related to the large size of the tumor (e.g., abdominal or back pain or other mass effects) or to increased secretion of one or more steroid hormones. Approximately 60% of adrenocortical carcinomas are hyperfunctioning; about 30% secrete cortisol and give rise to Cushing syndrome, and 20% secrete androgen and have virilizing effects. A mixed hormone secretion pattern is frequently present. Adrenocortical carcinoma is an aggressive tumor that may spread locally to regional lymph nodes, adjacent organs, and distant sites (including the liver, the lungs, and bones). Complete surgical resection offers the only chance for a cure and is the best predictor of clinical outcome. Overall 5-year actuarial survival rates after complete resection range from 32 to 48%. The presence of tumor thrombus in the renal vein or inferior vena cava is not a contraindication for surgical resection provided that the tumor can be completely excised. Treatment options for patients with unresectable or recurrent adrenocortical carcinomas are limited. Mitotane is toxic for adrenocortical cells and is the most specific chemotherapeutic agent available for the treatment of adrenocortical carcinomas.

A 29-year-old sexually active female is seen in the emergency department with 4 hours of acute lower abdominal pain. Her blood pressure is 78/40 mm Hg. She is pale, and her bilateral lower abdomen is tender to palpation without rebound or guarding. Her hematocrit returns at 20.0 and a serum β-human chorionic gonadotropin (β-hCG) is negative. A computed tomography (CT) scan reveals what appears to be a ruptured ovarian cyst with mild to moderate hemoperitoneum. Following 2,000 cc of normal saline and transfusion of 2 units of packed red blood cells, her blood pressure is 79/42 mm Hg. What is the most appropriate immediate management strategy? A) Operative management with unilateral salpingo-oopherectomy B) Operative management with oopherectomy C) Operative management with ovarian cystectomy D) Continued resuscitation with intravenous (IV) fluid E) Continued resuscitation with blood products

To demonstrate understanding of the diagnosis and management of bleeding from ovarian cysts Answer C is correct Each month, the ovary of a premenopausal woman undergoes a cycle of hormonally driven changes characterized by follicular development, the emergence of a dominant follicle, ovulation, and the formation of a corpus luteum. Cystic enlargement of one of these physiologic structures is relatively common and is considered functional rather than neoplastic. Approximately 7% of asymptomatic women between the ages of 25 and 40 years have ovarian cysts larger than 2.5 cm. Functional ovarian cysts should not cause pain unless accompanied by bleeding, rupture, or torsion. Follicular cysts are fluid-filled structures that arise from a normal follicle that fails to ovulate or does not undergo atresia. Rupture of follicular cysts may result in acute pain, which is usually of brief duration. Corpus luteum cysts arise from a mature, well-vascularized corpus luteum and may be associated with prolonged secretion of progesterone. They tend to be larger than follicular cysts and are more likely to cause clinical symptoms. Corpus luteum cysts range in severity from masses that cause no symptoms to ruptured cysts that cause catastrophic bleeding. Dull, unilateral lower abdominal and pelvic pain is the typical presenting complaint. Pelvic examination may show an enlarged, tender ovary. Unruptured corpus luteum cysts are followed conservatively. However, events such as coitus, exercise, or trauma may rupture the cyst, resulting in slight to significant intraperitoneal bleeding, in which case, the patient usually experiences the sudden onset of severe lower abdominal or pelvic pain. The acute pain of a bleeding corpus luteum cyst is indistinguishable from that of a ruptured ectopic pregnancy. A negative serum β-hCG determination rules out an ectopic pregnancy. In hemodynamically stable patients, pelvic ultrasonography may establish a diagnosis and permit expectant management even if some intraperitoneal bleeding has occurred. In an acute setting, CT typically demonstrates a cystic adnexal mass with areas of high attenuation at intramural and intracystic sites. A hemoperitoneum may also be present, with a high-attenuation clot and blood accumulating in the dependent pelvis. Laparoscopy may be useful when the diagnosis is uncertain or when a stable patient has a moderate hemoperitoneum. In hemodynamically unstable patients, adequate volume and blood replacement are initiated and emergency laparotomy is performed. Once it is confirmed that the bleeding is from a ruptured cyst, conservative therapy, consisting of removing the cyst and achieving hemostasis, is initiated.

A 68-year-old active female is found to have repeated episodes of low-volume vaginal bleeding. A focused history is negative for an obvious cause, and her physical and pelvic examinations are unremarkable. A pelvic sonogram is also unrevealing for pathology. What is the next most appropriate step in management? A) CT scan of the pelvis with IV and enteral contrast B) Endometrial biopsy C) Dilatation and curettage D) Hysterectomy E) Hormone replacement therapy

To demonstrate understanding of the diagnosis and management of endometrial cancer Answer B is correct Endometrial carcinoma is the most common gynecologic malignancy and accounts for more than 95% of uterine cancers. Although the majority of these carcinomas occur on a sporadic basis, an endometrial cancer can also be the initial manifestation of Lynch syndrome (hereditary nonpolyposis colorectal cancer [HNPCC]). In the United States, approximately 40,000 new cases of endometrial cancer are diagnosed each year, of which 7,000 prove fatal. About 75% of these cases involve postmenopausal women, with the remaining 25% involving premenopausal women; only about 5% of cases involve women younger than 40 years. Endometrial cancers are adenocarcinomas and usually resemble the glands of the endometrial lining; however, approximately 10% have a nonendometrioid (i.e., papillary serous or clear cell) histology, a pattern associated with a biologically aggressive behavior. Abnormal vaginal bleeding or postmenopausal bleeding is the most common presenting complaint. Outpatient endometrial biopsy with aspiration of tissue from the uterine cavity often establishes the diagnosis. If the tissue obtained is insufficient or the patient cannot tolerate outpatient evaluation, dilatation and curettage (often with hysteroscopy) is performed with the patient under anesthesia. Approximately 75% of patients with endometrial cancer have early disease. Therapy typically consists of hysterectomy, bilateral salpingo-oophorectomy, procurement of peritoneal specimens for cytologic examination, and staging. The regional lymph nodes are the most frequent site of occult metastatic disease. Accordingly, the surgical staging criteria emphasize the pathologic status of the regional lymph nodes with selective pelvic and para-aortic lymphadenectomy.

A 68-year-old active female is found to have repeated episodes of low-volume vaginal bleeding. A focused history is negative for an obvious cause, and her physical and pelvic examinations are unremarkable. A pelvic sonogram is also unrevealing for pathology. What is the next most appropriate step in management? A) CT scan of the pelvis with intravenous and enteral contrast. B) Endometrial biopsy C) Dilatation and curettage D) Hysterectomy E) Hormone replacement therapy

To demonstrate understanding of the diagnosis and management of endometrial cancer Answer B is correct. Endometrial carcinoma is the most common gynecologic malignancy and accounts for more than 95% of uterine cancers. Although the majority of these carcinomas occur on a sporadic basis, an endometrial cancer can also be the initial manifestation of Lynch syndrome (HNPCC). In the United States, approximately 40,000 new cases of endometrial cancer are diagnosed each year, of which 7,000 prove fatal. About 75% of these cases involve postmenopausal women, with the remaining 25% involving premenopausal women; only about 5% of cases involve women younger than 40 years. Endometrial cancers are adenocarcinomas and usually resemble the glands of the endometrial lining; however, approximately 10% have a nonendometrioid (i.e., papillary serous or clear cell) histology, a pattern associated with a biologically aggressive behavior. Abnormal vaginal bleeding or postmenopausal bleeding is the most common presenting complaint. Outpatient endometrial biopsy with aspiration of tissue from the uterine cavity often establishes the diagnosis. If the tissue obtained is insufficient or the patient cannot tolerate outpatient evaluation, dilatation and curettage (often with hysteroscopy) is performed with the patient under anesthesia. Approximately 75% of patients with endometrial cancer have early disease. Therapy typically consists of hysterectomy, bilateral salpingo-oophorectomy, procurement of peritoneal specimens for cytologic examination, and staging. The regional lymph nodes are the most frequent site of occult metastatic disease. Accordingly, the surgical staging criteria emphasize the pathologic status of the regional lymph nodes with selective pelvic and para-aortic lymphadenectomy.

A 44-year-old female complains of pain in her right breast for several days. Physical examination reveals a tender and inflamed cord palpated over the lateral aspect of the right breast. She has no cervical or axillary lymphadenopathy. Which of the following is the most appropriate treatment strategy? A) Reassurance and observation B) Nonsteroidal antiinflammatory drugs (NSAIDs) and antibiotics C) Ultrasonography D) Core-needle biopsy E) Operative biopsy

To demonstrate understanding of the diagnosis and treatment of Mondor disease Answer B is correct Mondor disease is thrombophlebitis of the superficial veins of the breast. It is characterized by the finding of a tender and often inflamed cord palpated on the patient's breast. After a thorough history and physical examination of the patient's breasts, treatment should consist of NSAIDs, analgesics, and antibiotics. If there is evidence of infection or should the condition fail to improve, surgical excision is appropriate for definitive management and diagnosis.

A 17-year-old male is seen in the emergency department following a minor sports injury. Physical examination reveals discoid, subareolar, rubbery thickening of the bilateral breast tissue. The rest of the physical examination, including a testicular examination, is normal. His past medial history is negative, he does not take any medications, and he admits to regular use of marijuana. Regarding the findings of breast examination, which of the following statements is false? A) This is usually a benign condition B) Biopsy will be required C) This condition is the result of medication in 20 to 25% of cases D) This condition can be the result of elevated prolactin levels E) Tamoxifen, raloxifene, and aromatase inhibitors can be used to treat this problem

To demonstrate understanding of the diagnosis and management of gynecomastia Answer B is correct Gynecomastia, or a breast mass in male breast tissue, is usually a benign condition in adolescent males and presents as discoid, subareolar, rubbery thickening of the breast tissue. In the absence of any other findings, such as testicular pathology, or a history of ingestion or use of substances associated with gynecomastia, these patients should be reassured and reexamined. In the case of especially prominent, large, or symptomatic gynecomastia, surgical excision is reasonable. In the mature male population, a physical examination and history are vitally important. Soft, diffuse enlargement is a result of medication in 20 to 25% of cases. Gynecomastia can also be due to hormonal, physiologic, or idiopathic factors and can usually be managed nonoperatively and with serial examinations. Many medications resulting in gynecomastia are felt to do so by elevating prolactin levels comparatively. If the gynecomastia is attributable to medication, switching the patient off the medication will often result in regression over the course of a few months. For men who would like to avoid surgery, some success has been noted in treating gynecomastia with tamoxifen, raloxifene, and aromatase inhibitors. However, a finding of a solitary hard mass, especially with findings of adenopathy and skin changes, requires aggressive workup, including mammogram and tissue diagnosis, usually with CNB.

A 65-year-old female is found to have unilateral skin edema and erythema over half of her breast. A skin biopsy was performed and shows invasion of the dermal lymphatics by tumor cells. What is the next best course of action? A) Systemic chemotherapy B) Local radiotherapy C) Breast conservation therapy D) Total mastectomy E) Modified radical mastectomy

To demonstrate understanding of the diagnosis and management of inflammatory breast cancer Answer A is correct Inflammatory breast cancer is an infiltrating cancer characterized by skin edema and erythema encompassing over half the breast and associated with an especially high likelihood of subsequent metastases and death. It often correlates with the histologic finding of dermal lymphatics clogged by tumor cells, but the diagnosis of inflammatory cancer is only a clinical diagnosis. Skin biopsy showing dermal lymphatic involvement is not necessary to classify a cancer as inflammatory. Neoadjuvant chemotherapy is the treatment of choice for those with cancer that involves the skin or chest wall (T4), those with extensive lymph node involvement clinically, or those who have inflammatory cancer.

A 54-year-old male is seen in your office with the complaint of a lump in his breast. Physical examination reveals an overweight gentleman with a 2 cm hard and fixed mass below the right areolar complex. There is marked erosion of the skin, dimpling, and a significant area of erythema directly overlying the mass. There is no palpable axillary lymphadenopathy. A CNB is performed in your office, and the pathology is consistent with a high-grade invasive ductal adenocarcinoma. Which of the following regarding the management of this patient is false? A) He will require mastectomy B) He will require radiation C) Sentinel lymph node biopsy and axillary lymph node dissection should be performed using the same indications as in women D) Most male breast cancers are hormone receptor negative E) Breast cancer in a younger man may be indicative of inherited susceptibility within that family particularly related to BRCA2

To demonstrate understanding of the diagnosis and management of male breast cancer Answer D is correct Breast cancer occurs in men with an incidence less than 1% of that of women. No screening for men is warranted, but a mass in the breast should be fully evaluated. Most masses proved to be gynecomastia. Breast cancer in a younger man may be indicative of inherited susceptibility within that family particularly related to BRCA2. Careful family history and counseling regarding this risk should be provided. The treatment of breast cancer in men follows the same principles as in women. Surgery is generally mastectomy because the tumor is directly under the nipple complex. Wide margins should be obtained. Radiation may be administered if the tumor is larger or involves the skin and/or chest wall. Sentinel lymph node biopsy and axillary lymph node dissection should be performed using the same indications as in women. Men should also receive adjuvant systemic therapy using the same criteria as women. Most are hormone receptor positive, and endocrine therapy is effective.

A 28-year-old sexually active female is seen in the emergency department with 2 days of acute lower abdominal pain. On examination, her temperature is 38.6°C (101.6°F) and she has diffuse tenderness of the bilateral lower quadrants with rebound tenderness on the left lower quadrant. A pelvic examination reveals purulent endocervical discharge with cervical motion tenderness. Which of the following statements is true? A) The etiology is a sexually transmitted disease B) The etiology is a polymicrobial infection C) Broad-spectrum, empirical antibiotics should be started D) Many tubo-ovarian abscesses can be successfully managed with antibiotics alone E) All of the above are true

To demonstrate understanding of the diagnosis and management of pelvic inflammatory disease (PID) Answer E is correct Pelvic inflammatory disease (PID) is an acute infection of the upper female genital tract that involves the uterus, the fallopian tubes, and the ovaries. It is a community-acquired disease and is initiated by a sexually transmitted agent that results in the ascent of vaginal flora into the upper genital tract. The resulting polymicrobial infection (caused by a mixture of anaerobes, facultative anaerobes, and aerobic organisms) can lead to a wide spectrum of disorders, including endometritis, salpingitis, tubo-ovarian abscess, and pelvic peritonitis. Lower abdominal pain is the most frequent complaint. The pain usually is bilateral, rarely is of more than 2 weeks' duration, and often begins with or follows menses. Approximately half of the patients are febrile. Abdominal palpation demonstrates diffuse tenderness that is more pronounced in the two lower quadrants. Rebound tenderness and diminished bowel sounds are common. Pelvic examination reveals a purulent endocervical discharge with cervical motion and adnexal tenderness. Empirical treatment of PID should be initiated in sexually active young women or women at risk in the appropriate clinical setting. PID treatment regimens are designed to provide broad-spectrum empirical therapy of likely pathogens, including N.gonorrhoeae, C. trachomatis, anaerobes, and gram-negative facultative bacteria. Tubo-ovarian abscess is the most serious manifestation of PID. Fevers and pelvic pain are typical presenting complaints. Pelvic examination usually demonstrates a mass or extreme adnexal tenderness. Many unruptured tubo-ovarian abscesses, unlike unruptured abscesses elsewhere in the body, can be successfully managed with antibiotics alone. Ultrasonography is useful for diagnosis and follow-up of cases that are managed conservatively. Indications for surgical intervention include a questionable diagnosis, rupture of the tubo-ovarian abscess (a true surgical emergency), and failure of medical (i.e., antibiotic) therapy.

A 22-year-old sexually active female is seen in the emergency department with 1 day of acute lower abdominal pain. On examination, her temperature is 38.7°C (101.8°F), and she has diffuse tenderness of the bilateral lower quadrants with rebound tenderness on the left lower quadrant. A pelvic examination reveals purulent endocervical discharge with cervical motion tenderness. A CT scan reveals a 3.5 cm left tubo-ovarian abscess. What is the most appropriate treatment strategy? A) 14- to 21-day course of oral antibiotics and outpatient follow-up B) Intravenous antibiotics and hospital admission C) Percutaneous drainage D) Laparoscopy and drainage E) Laparoscopy and unilateral ovarian cystectomy

To demonstrate understanding of the diagnosis and management of pelvic inflammatory disease (PID) and tubo-ovarian abscess Answer B is correct Pelvic inflammatory disease (PID) is an acute infection of the upper female genital tract that involves the uterus, the fallopian tubes, and the ovaries. It is a community-acquired disease and is initiated by a sexually transmitted agent that results in the ascent of vaginal flora into the upper genital tract. The resulting polymicrobial infection (caused by a mixture of anaerobes, facultative anaerobes, and aerobic organisms) can lead to a wide spectrum of disorders, including endometritis, salpingitis, tubo-ovarian abscess, and pelvic peritonitis. Lower abdominal pain is the most frequent complaint. The pain usually is bilateral, rarely is of more than 2 weeks' duration, and often begins with or follows menses. Approximately half of the patients are febrile. Abdominal palpation demonstrates diffuse tenderness that is more pronounced in the two lower quadrants. Rebound tenderness and diminished bowel sounds are common. Pelvic examination reveals a purulent endocervical discharge with cervical motion and adnexal tenderness. Empirical treatment of PID should be initiated in sexually active young women or women at risk in the clinical setting of lower abdominal tenderness, adnexal tenderness, and cervical motion tenderness. Additional criteria supporting the diagnosis include fever (oral temperature > 38.3°C [101°F]), abnormal cervical or vaginal mucopurulent discharge, the presence of abundant white cells on saline microscopy of vaginal secretions, an elevated sedimentation rate, an elevated C-reactive protein level, and documentation of cervical infection with Neisseria gonorrhoeae or Chlamydia trachomatis. Pertinent laboratory studies include determination of the serum β-hCG level (to rule out an ectopic pregnancy), microscopic examination of the vaginal discharge, a complete blood count, endocervical nucleic acid amplification tests for N. gonorrhoeae and C. trachomatis, sedimentation rate, and urinalysis. Unfortunately, many episodes of PID go unrecognized and subsequently give rise to infertility, ectopic pregnancy, chronic pelvic pain, or tubo-ovarian abscesses. PID treatment regimens are designed to provide broad-spectrum empirical therapy of likely pathogens, including N. gonorrhoeae, C. trachomatis, anaerobes, and gram-negative facultative bacteria. Criteria for inpatient therapy include an inability to rule out a surgical emergency (e.g., appendicitis), severe symptoms, high fevers, failure to respond to oral antibiotics, and the presence of a tubo-ovarian abscess. Patients who are initially treated with parenteral antibiotics are usually switched to oral therapy within 24 hours after clinical improvement. Tubo-ovarian abscess is the most serious manifestation of PID. Fevers and pelvic pain are typical presenting complaints. Pelvic examination usually demonstrates a mass or extreme adnexal tenderness. Many unruptured tubo-ovarian abscesses, unlike unruptured abscesses elsewhere in the body, can be successfully managed with antibiotics alone. Ultrasonography is useful for diagnosis and follow-up of cases that are managed conservatively. Medical management of an unruptured abscess consists of administration of appropriate antibiotics, close monitoring, and, possibly, drainage via colpotomy. Indications for surgical intervention include a questionable diagnosis, rupture of the tubo-ovarian abscess (a true surgical emergency), and failure of medical (i.e., antibiotic) therapy.

A 22-year-old sexually active female is seen in the emergency department with 1 day of acute lower abdominal pain. On examination, her temperature is 38.7°C (101.8°F) and she has diffuse tenderness of the bilateral lower quadrants with rebound tenderness on the left lower quadrant. A pelvic examination reveals purulent endocervical discharge with cervical motion tenderness. A CT scan reveals a 3.5 cm left tubo-ovarian abscess. What is the most appropriate treatment strategy? A) 14- to 21-day course of oral antibiotics and outpatient follow-up B) IV antibiotics and hospital admission C) Percutaneous drainage D) Laparoscopy and drainage E) Laparoscopy and unilateral ovarian cystectomy

To demonstrate understanding of the diagnosis and management of pelvic inflammatory disease (PID) and tubo-ovarian abscess Answer B is correct Pelvic inflammatory disease (PID) is an acute infection of the upper female genital tract that involves the uterus, the fallopian tubes, and the ovaries. It is a community-acquired disease and is initiated by a sexually transmitted agent that results in the ascent of vaginal flora into the upper genital tract. The resulting polymicrobial infection (caused by a mixture of anaerobes, facultative anaerobes, and aerobic organisms) can lead to a wide spectrum of disorders, including endometritis, salpingitis, tubo-ovarian abscess, and pelvic peritonitis. Lower abdominal pain is the most frequent complaint. The pain usually is bilateral, rarely is of more than 2 weeks' duration, and often begins with or follows menses. Approximately half of the patients are febrile. Abdominal palpation demonstrates diffuse tenderness that is more pronounced in the two lower quadrants. Rebound tenderness and diminished bowel sounds are common. Pelvic examination reveals a purulent endocervical discharge with cervical motion and adnexal tenderness. Empirical treatment of PID should be initiated in sexually active young women or women at risk in the clinical setting of lower abdominal tenderness, adnexal tenderness, and cervical motion tenderness. Additional criteria supporting the diagnosis include fever (oral temperature > 38.3° C [101° F]), abnormal cervical or vaginal mucopurulent discharge, the presence of abundant white cells on saline microscopy of vaginal secretions, an elevated sedimentation rate, an elevated C-reactive protein level, and documentation of cervical infection with Neisseria gonorrhoeae or Chlamydia trachomatis. Pertinent laboratory studies include determination of the serum β -hCG level (to rule out an ectopic pregnancy), microscopic examination of the vaginal discharge, a complete blood count, endocervical nucleic acid amplification tests for N. gonorrhoeae and C. trachomatis, sedimentation rate, and urinalysis. Unfortunately, many episodes of PID go unrecognized and subsequently give rise to infertility, ectopic pregnancy, chronic pelvic pain, or tubo-ovarian abscesses. PID treatment regimens are designed to provide broad-spectrum empirical therapy of likely pathogens, including N. gonorrhoeae, C. trachomatis, anaerobes, and gram-negative facultative bacteria. Criteria for inpatient therapy include an inability to rule out a surgical emergency (e.g., appendicitis), severe symptoms, high fevers, failure to respond to oral antibiotics, and the presence of a tubo-ovarian abscess. Patients who are initially treated with parenteral antibiotics are usually switched to oral therapy within 24 hours after clinical improvement. Tubo-ovarian abscess is the most serious manifestation of PID. Fevers and pelvic pain are typical presenting complaints. Pelvic examination usually demonstrates a mass or extreme adnexal tenderness. Many unruptured tubo-ovarian abscesses, unlike unruptured abscesses elsewhere in the body, can be successfully managed with antibiotics alone. Ultrasonography is useful for diagnosis and follow-up of cases that are managed conservatively. Medical management of an unruptured abscess consists of administration of appropriate antibiotics, close monitoring, and, possibly, drainage via colpotomy. Indications for surgical intervention include a questionable diagnosis, rupture of the tubo-ovarian abscess (a true surgical emergency), and failure of medical (i.e., antibiotic) therapy.

A 44-year-old female is referred to you for a 1.4 cm solid left adrenal lesion that was found incidentally for the workup of fibroids. She has no other known medical problems and complains only of headache and fatigue. In your office, her blood pressure is 181/90 mm Hg and her serum potassium is 2.8 mEq/L. Which of the following statements regarding her condition is true? A) The most likely diagnosis is Cushing syndrome B) The most likely diagnosis is secondary hyperaldosteronism C) She should undergo bilateral adrenal vein sampling of aldosterone and cortisol D) Alkalosis is common in this scenario E) She should undergo right adrenalectomy if her plasma aldosterone-to-renin ratio is significantly elevated

To demonstrate understanding of the diagnosis and management of primary hyperaldosteronism (Conn syndrome) Answer D is correct Primary hyperaldosteronism (Conn syndrome) occurs in as many as 8 to 12% of hypertensive patients. The diagnosis of primary hyperaldosteronism should be considered in any patient who has hypertension of early onset, hypertension that is difficult to control, or hypertension with hypokalemia. The pathophysiology primarily consists of increased aldosterone secretion by the adrenal gland, which leads to increased sodium retention, expansion of intravascular fluid volume, and suppression of renin secretion by the kidney. Aldosterone also promotes the exchange of sodium for potassium and hydrogen ion in the distal tubule, which leads to potassium depletion and alkalosis. The alkalosis may be aggravated by the movement of hydrogen ion into the cells to replace intracellular potassium. The initial biochemical evaluation for suspected primary hyperaldosteronism should consist of measurement of the plasma aldosterone concentration (PAC) and assessment of plasma renin activity (PRA). A PAC-to-PRA ratio greater than 25 to 30, in conjunction with suppressed PRA (< 0.2 to 0.5 ng/mL/hr) and a PAC higher than 15 ng/dL, is consistent with primary hyperaldosteronism. Once the diagnosis of primary hyperaldosteronism has been confirmed biochemically, the next step is to identify the cause or determine the subtype. Historically, aldosterone-secreting adenomas (aldosteronomas) have accounted for approximately 60% of cases, and idiopathic hyperaldosteronism from bilateral cortical hyperplasia has accounted for the bulk of the remaining cases. More recent data, however, suggest that only 30 to 50% of patients with primary hyperaldosteronism had aldosteronomas, whereas 50 to 70% had bilateral adrenal hyperplasia. It is important to distinguish between these two conditions because aldosteronomas are treated surgically and idiopathic hyperaldosteronism is treated medically.

A 31-year-old woman is referred to you for a serum calcium level of 11.8 mg/dL and a serum parathyroid hormone (PTH) level of 80 ng/L (normal 10 to 65 ng/L). She denies depression, cognitive symptoms, musculoskeletal pain, or nephrolithiasis. Her physical examination is unrevealing. Which of the following statements concerning her management is true? A) Either the serum calcium or the serum PTH level is spurious B) Imaging studies are unlikely to be of value C) She would achieve most benefit from a targeted resection D) She would achieve most benefit from four-gland parathyroidectomy with autotransplantation E) Workup for malignancy should be immediately pursued

To demonstrate understanding of the diagnosis and management of primary hyperparathyroidism Answer C is correct. The most likely diagnosis in this case is a hyperfunctioning parathyroid adenoma. Primary hyperparathyroidism results from the inappropriately high secretion of PTH. This may be caused by dysfunction in one or multiple parathyroid glands. Approximately 90% of primary hyperparathyroidism is caused by a parathyroid adenoma, which can be multiple in approximately 3 to 8% of cases. In contrast, 9% of primary hyperparathyroidism is attributable to four-gland hyperplasia, and approximately 1% is the result of parathyroid carcinoma. In general, primary hyperparathyroidism is treated surgically with removal of all hyperfunctioning parathyroid tissue. There is agreement that all symptomatic disease should be treated surgically. However, the indications for the treatment of asymptomatic disease are less clear-cut. Currently, surgery is recommended for asymptomatic patients with serum calcium levels 1.0 mg/dL above the upper limit of normal, a glomerular filtration rate less than 60 mL/min, T scores less than − 2.5 at any site and/or previous fracture fragility, or age less than 50 years. Localization studies are helpful for preoperative planning for parathyroid adenomas and have improved dramatically over recent years, allowing for progressively less invasive surgical options. Because 80% of patients with primary hyperparathyroidism have a single adenoma, preoperative localization studies allow far less extensive neck dissection. The mainstays of localization studies are ultrasonography and sestamibi scanning. Ultrasonography has a sensitivity of 79%, whereas sestamibi scanning has a sensitivity of 88%. When these two modalities are combined, sensitivity increases to greater than 90%. Surgical treatment entails resection of the abnormal, hyperfunctioning parathyroid gland.

A 31-year-old woman is referred to you for a serum calcium level of 11.8 mg/dL and an serum PTH level of 80 ng/L (normal 10 to 65 ng/L). She denies depression, cognitive symptoms, musculoskeletal pain, or nephrolithiasis. Her physical examination is unrevealing. Which of the following statements concerning her management is true? A) Either the serum calcium or the serum PTH level is spurious B) Imaging studies are unlikely to be of value C) She would achieve most benefit from a targeted resection D) She would achieve most benefit from four-gland parathyroidectomy with autotransplantation E) Workup for malignancy should be immediately pursued

To demonstrate understanding of the diagnosis and management of primary hyperparathyroidism Answer C is correct. The most likely diagnosis in this case is a hyperfunctioning parathyroid adenoma. Primary hyperparathyroidism results from the inappropriately high secretion of PTH. This may be caused by dysfunction in one or multiple parathyroid glands. Approximately 90% of primary hyperparathyroidism is caused by a parathyroid adenoma, which can be multiple in approximately 3 to 8% of cases. In contrast, 9% of primary hyperparathyroidism is attributable to four-gland hyperplasia, and approximately 1% is the result of parathyroid carcinoma. In general, primary hyperparathyroidism is treated surgically with removal of all hyperfunctioning parathyroid tissue. There is agreement that all symptomatic disease should be treated surgically. However, the indications for the treatment of asymptomatic disease are less clear-cut. Currently, surgery is recommended for asymptomatic patients with serum calcium levels 1.0 mg/dL above the upper limit of normal, a glomerular filtration rate less than 60 mL/min, T scores less than −2.5 at any site and/or previous fracture fragility, or age less than 50 years. Localization studies are helpful for preoperative planning for parathyroid adenomas and have improved dramatically over recent years, allowing for progressively less invasive surgical options. Because 80% of patients with primary hyperparathyroidism have a single adenoma, preoperative localization studies allow far less extensive neck dissection. The mainstays of localization studies are ultrasonography and sestamibi scanning. Ultrasonography has a sensitivity of 79%, whereas sestamibi scanning has a sensitivity of 88%. When these two modalities are combined, sensitivity increases to greater than 90%. Surgical treatment entails resection of the abnormal, hyperfunctioning parathyroid gland.

Two years following breast conservation therapy for a 1 cm invading ductal carcinoma, a 54-year-old woman feels a mass in the surgical resection site. A 1.5 cm solid lesion with irregular borders is discerned on MRI, and a CNB confirms ductal adenocarcinoma. What treatment course should immediately follow? A) Reexcision with negative margins B) Reexcision with negative margins and radiation C) Mastectomy D) Local radiation E) Systemic chemotherapy

To demonstrate understanding of the diagnosis and management of recurrent invasive breast cancer following breast conservation therapy Answer C is correct. Local recurrence after surgical treatment of breast cancer occurs with both mastectomy and breast conservation therapy. When performed by experienced surgeons with careful pathologic control, appropriate radiation, and stage-appropriate adjuvant systemic therapy, the rate of local recurrence with breast conservation therapy is as low as 1 to 5%. Local recurrences are also a significant issue among women treated with mastectomy. This risk is about 2 to 5% for women with negative lymph nodes and ranges as high as 25% for women with multiple positive lymph nodes. Local recurrence is a serious event whether it occurs after breast conservation therapy or mastectomy. These women have a higher risk of developing distant metastases compared with women with similar-stage cancer who do not suffer local recurrence. The accepted treatment for local recurrence after breast conservation therapy is mastectomy. This is because the recurrence may be multifocal and because full-dose radiation cannot be administered safely a second time because of tissue toxicity. Local recurrence after mastectomy is more difficult to treat and carries a more serious prognosis. Treatment usually includes resection of a local recurrence in the skin, muscle, or regional nodes if possible, followed by radiation to the chest wall and regional nodes. The role for additional adjuvant systemic therapy after local recurrence is uncertain, and there are no clinical trial data supporting its use. However, many oncologists will deliver additional systemic treatment based on the type of previous treatment and the nature and extent of the local recurrence.

A 58-year-old woman with recurrent pelvic pain is found to have a large uterine leiomyoma. Which of the following statements regarding uterine leiomyomas is false? A) Uterine leiomyomas are malignant tumors of the myometrium. B) Uterine leiomyomas are the most common tumors of the uterus. C) The majority of women with uterine leiomyomas are asymptomatic. D) Symptomatic uterine leiomyomas can be treated with myomectomy. E) Symptomatic uterine leiomyomas can be treated with uterine artery embolization.

To demonstrate understanding of the diagnosis and management of uterine leiomyomas Answer A is correct Leiomyomas, or myomas, arise as benign tumors from the myometrium and are the most common tumors of the uterus. Leiomyomas are often multiple and are grouped according to their location within the uterus; intramural, subserosal, and submucosal sites are the most common locations. Symptoms include pressure from an enlarging uterus, abnormal uterine bleeding, dysmenorrhea, and adverse pregnancy outcomes. The majority of women with myomas, however, are asymptomatic. In general, the severity of the symptoms is determined by the size and number of the fibroids, as well as their location. The diagnosis of myomas is typically made through palpation of an irregular, enlarged uterus, although other causes of uterine enlargement or asymmetry must also be considered. Difficult cases may be resolved by employing ultrasonography or magnetic resonance imaging (MRI) to examine the uterus. Small, asymptomatic leiomyomas may be managed with observation once a stable uterine size has been established. The need for intervention is determined on the basis of interval growth, lesion location, and patient symptoms. Rapid growth or enlargement of the uterus is another indication for surgical treatment because of the possibility of a uterine sarcoma, as well as an increased risk of operative complications. Women with symptomatic leiomyomas may be treated with myomectomy, hysterectomy, or, as is now becoming more common, uterine artery embolization. The choice between myomectomy and hysterectomy is usually determined by the patient's age, parity, and, most important, desire for future childbearing. The therapeutic effect of uterine artery embolization is thought to result from irreversible postembolic ischemia that causes necrosis and shrinkage of the myomas.

On a routine pelvic examination, a 58-year-old woman is found to have a fungating mass of the cervix. A punch biopsy is performed and reveals carcinoma. Which of the following statements regarding cervical cancer is false? A) Cervical cancer is the most common genital tract cancer in the world B) This disease commonly presents with postcoital bleeding or bloody vaginal discharge C) Colposcopy or conization may be required for diagnosis of early lesions D) For young patients with microinvasive cancers, the treatment of choice is radical hysterectomy E) Adjuvant chemotherapy and radiation are offered to patients with advanced disease

To demonstrate understanding of the diagnosis and treatment of cervical cancer Answer D is correct Although cervical cancer is only the third most common gynecologic malignancy in the United States, it is the most common genital tract cancer in the world. Patients with early invasive carcinomas may present with nothing more than an abnormal Papanicolaou smear, but most complain of postcoital bleeding or a bloody vaginal discharge. With early disease, colposcopy or conization may be required to establish the diagnosis; however, the majority of patients have a gross cervical lesion, and punch biopsy is usually performed to establish the diagnosis. Treatment of cervical cancer is predicated on the patient's suitability for surgery and on the stage and bulk of the tumor. Young patients with early, genuine microinvasive carcinomas and a desire for future childbearing may be followed conservatively after conization with negative margins. For the majority of patients with microinvasive cancers, definitive treatment consists of simple abdominal or vaginal hysterectomy; the risk of pelvic nodal metastasis is lower than 1%. Patients with stage IA2, IB, and IIA disease may be treated with radical hysterectomy. This surgical procedure differs from simple abdominal hysterectomy in several respects. In a simple hysterectomy for benign disease, the vascular pedicles are placed close to the uterus and the cervix. A radical hysterectomy dissects out the lower aspect of the ureters, allowing wider clearance of the central specimen and the adjacent adventitial tissues (e.g., cardinal ligaments and parametria). Once the distal ureters have been mobilized, clamps are placed into the paravaginal tissues, and an extra margin of lateral tissue is removed. The upper 2 cm of the vagina is removed in continuity with the rest of the specimen. A systematic pelvic lymph node dissection is then performed, with skeletonization of the common iliac artery, the external iliac artery, the external iliac vein, and the obturator nerve. Patients who have more advanced disease (stage IIB or higher) or are unable to undergo surgery are treated with radiotherapy and chemotherapy as a radiation sensitizer.

A 62-year-old woman was found to have a linear grouping of calcifications on mammography and therefore underwent a stereotactic core biopsy of this lesion. The pathology revealed the presence of malignant-appearing cells confined to the lumen of the ductal system. Which of the following represents the best management option? A) Observation and radiographic surveillance B) Tamoxifen alone C) Lumpectomy D) Lumpectomy and radiation E) Modified radical mastectomy

To demonstrate understanding of the diagnosis and treatment of ductal carcinoma in situ (DCIS) Answer D is correct The histologic hallmark of DCIS is the presence of malignant-appearing cells confined to the lumen of the ductal system in the breast. DCIS is most often diagnosed because of the presence of calcifications clustered in one area of the breast. The calcifications suspicious for DCIS are often linear (not round), growing in a ductal distribution. DCIS is generally considered a precursor to invasive cancer. The purpose of treatment of DCIS is to prevent progression to invasive cancer with the sequelae of metastatic disease and death. All women diagnosed with DCIS should undergo appropriate treatment. Mastectomy is effective in DCIS, but in most cases, a breast-conserving approach is possible. Most women with DCIS may be treated by surgical excision ("lumpectomy" or "wide excision") of the DCIS with some surrounding normal breast tissue to achieve a "negative margin." The choice between breast-conserving surgery and mastectomy may be influenced by the size of the area of involvement, the size of the breast, and the location of the DCIS. Large-scale randomized clinical trials in North America and Europe have demonstrated the effectiveness of breast conservation therapy and the role of radiation in DCIS. The National Surgical Adjuvant Breast and Bowel Project (NSABP), the European Organization for Research and Treatment of Cancer (EORTC), and others conducted trials comparing wide excision alone versus wide excision plus radiation therapy. Every study demonstrated long-term overall survival approaching 100% irrespective of treatment. However, women treated with breast conservation therapy had a substantial risk of recurrence in the same breast, and radiation reduced this risk. The rate of local recurrence in the NSABP study was 30% without radiation and about 12% with radiation.

A 31-year-old woman presents to the emergency department with lower abdominal cramping and vaginal bleeding. A serum β-human chorionic gonadotropin test is positive. While awaiting transvaginal ultrasonography, she develops a rigid abdomen and her blood pressure decreases to 75/40 mm Hg. What is the most appropriate next step in management? A) Emergent transvaginal ultrasonography B) Emergent transabdominal ultrasonography C) Admission to the intensive care unit for resuscitation and serial abdominal examinations D) Computed tomography of the abdomen and pelvis E) Surgical exploration

To demonstrate understanding of the diagnosis and treatment of ectopic pregnancy Answer E is correct Pelvic pain and vaginal bleeding in the first trimester are the complaints most commonly associated with ectopic pregnancy. Although hypotension, tachycardia, and guarding may be noted and alert the clinician to impending tubal rupture, most patients present with less alarming findings. Patients with a suspected ectopic pregnancy are initially assessed by means of transvaginal ultrasonography; the first task in the diagnostic evaluation is to exclude an intrauterine pregnancy. The clinical scenario of hypovolemic shock, an acute abdomen, and a positive pregnancy test usually establishes the diagnosis of a ruptured ectopic pregnancy. The rupture of the ectopic gestation through the tube is followed by the abrupt onset of pelvic pain lateralized to the affected adnexa. The clinical manifestations of hemoperitoneum ensue, including abdominal pain, diaphragmatic irritation, and syncope. Once the diagnosis of a ruptured ectopic pregnancy is made, treatment consists of surgical removal of the ectopic gestation; there is no role for medical management (e.g., methotrexate) in the treatment of this condition. It should be kept in mind that not all ruptured ectopic gestations necessitatelaparotomy. Hemodynamically stable patients can often be treated laparoscopically, depending on the clinical situation, the physical characteristics of the patient, and the equipment available. Diagnostic laparoscopy is an excellent tool for both diagnosis and treatment in such patients. The patient's desire for future fertility must be considered before surgical intervention is initiated.

A 31-year-old woman presents to the emergency department with lower abdominal cramping and vaginal bleeding. A serum β-human chorionic gonadotropin β-hCG) test is positive. While awaiting transvaginal ultrasonography, she develops a rigid abdomen and her blood pressure decreases to 75/40 mm Hg. What is the most appropriate next step in management? A) Emergent transvaginal ultrasonography B) Emergent transabdominal ultrasonography C) Admission to the intensive care unit for resuscitation and serial abdominal examinations D) Computed tomography (CT) of the abdomen and pelvis E) Surgical exploration

To demonstrate understanding of the diagnosis and treatment of ectopic pregnancy Answer E is correct. Pelvic pain and vaginal bleeding in the first trimester are the complaints most commonly associated with ectopic pregnancy. Although hypotension, tachycardia, and guarding may be noted and alert the clinician to impending tubal rupture, most patients present with less alarming findings. Patients with a suspected ectopic pregnancy are initially assessed by means of transvaginal ultrasonography; the first task in the diagnostic evaluation is to exclude an intrauterine pregnancy. The clinical scenario of hypovolemic shock, an acute abdomen, and a positive pregnancy test usually establishes the diagnosis of a ruptured ectopic pregnancy. The rupture of the ectopic gestation through the tube is followed by the abrupt onset of pelvic pain lateralized to the affected adnexa. The clinical manifestations of hemoperitoneum ensue, including abdominal pain, diaphragmatic irritation, and syncope. Once the diagnosis of a ruptured ectopic pregnancy is made, treatment consists of surgical removal of the ectopic gestation; there is no role for medical management (e.g., methotrexate) in the treatment of this condition. It should be kept in mind that not all ruptured ectopic gestations necessitate laparotomy. Hemodynamically stable patients can often be treated laparoscopically depending on the clinical situation, the physical characteristics of the patient, and the equipment available. Diagnostic laparoscopy is an excellent tool for both diagnosis and treatment in such patients. The patient's desire for future fertility must be considered before surgical intervention is initiated.

A 32-year-old female with no past medical history underwent FNA of a 14 mm solid thyroid nodule. The pathology was consistent with papillary thyroid cancer. Which of the following statements regarding this malignancy is false? A) She should be offered hemithyroidectomy B) Papillary thyroid cancer is the most common thyroid cancer C) Papillary thyroid cancer carries a greater than 90% 10-year survival D) The most common mode of spread of papillary thyroid cancer is lymphatic E) The majority of papillary thyroid cancers are multicentric

To demonstrate understanding of the diagnosis and treatment of papillary thyroid cancer Answer A is correct Papillary thyroid cancer is the most common thyroid cancer and carries an excellent prognosis when treated appropriately (> 90% 10-year survival overall). Poor prognostic factors include age less than 20 or more than 60 years, tumor greater than 4 cm, extrathyroidal extension, and the presence of distant metastases. Lymphatic spread worsens the prognosis and increases local recurrence risk for patients over 45 years and those with clinically apparent nodal involvement. One quarter of papillary thyroid cancers are multicentric, and lymph nodes are involved at the initial presentation in about one third. Total thyroidectomy should be performed for most papillary thyroid cancers exceeding 1 cm; this procedure addresses multicentricity, maximizes the therapeutic impact of postoperative radioactive iodine, and facilitates surveillance for recurrences. Lobectomy suffices for small, intrathyroidal papillary thyroid cancers discovered incidentally in glands resected for other diagnoses when radiation exposure, nodal disease, and poor prognostic variants are absent. Central neck dissection is added based on FNA or operative findings suggesting lymphatic spread. The favorable natural history of papillary thyroid cancer and the morbidity of lateral cervical lymphadenectomy suggest that this procedure be added only for pathologically proven lateral nodal disease.

In the office, you are evaluating a 39-year-old male patient who complains of worsening forgetfulness and muscle weakness and was recently discharged from the emergency department for nephrolithiasis. A physical examination reveals a 2 cm fixed mass in the left neck, approximately 3 cm from the midline. The serum calcium level is 13.1 mg/dL, and the serum parathyroid hormone (PTH) level is elevated outside the range of the assay. What is the most likely diagnosis? A) Primary hyperparathyroidism B) Secondary hyperparathyroidism C) Tertiary hyperparathyroidism D) Parathyroid carcinoma E) Thyroid carcinoma

To demonstrate understanding of the diagnosis and treatment of parathyroid carcinoma Answer D is correct The most likely diagnosis in this case is parathyroid carcinoma. Parathyroid carcinoma is a rare cause of hyperparathyroidism. It often results in markedly elevated levels of both PTH and calcium. Complications of hyperparathyroidism are more common with parathyroid carcinoma. The incidence of kidney stones is greater than 50%, whereas severe bone disease is observed in as many as 90% of patients. These lesions are more likely palpable than benign hypersecreting glands, and on gross inspection, they exhibit invasive features. Of course, the hallmark of parathyroid carcinoma is its invasive nature. Parathyroid lesions that grossly extend into neighboring structures should prompt suspicion of malignancy. Parathyroid carcinoma absolutely requires surgery if cure is to be achieved. In this case, the operation of choice is en bloc resection of the malignant parathyroid tumor with the overlying musculature and ipsilateral thyroid lobe. Additionally, local recurrences and localized distant metastases should be treated surgically. Systemic chemotherapy and radiation have minimal efficacy and are considered palliative interventions in this context. Thirty percent of patients will have nodal metastases on presentation, and the 5-year survival rate approximates 60% with treatment.

A 39-year-old male was seen in the emergency department with a severe headache in the setting of a blood pressure of 210/120 mm Hg. He was admitted to the medical intensive care unit (ICU) and treated with several antihypertensive agents that are modestly controlling his blood pressure. A CT scan of the abdomen revealed a 2 cm mass in the left adrenal gland. The next appropriate step in management is described by which of the following? A) Magnetic resonance imaging (MRI) with T2-weighted images B) Plasma metanephrine determination C) Percutaneous biopsy of the adrenal lesion D) Left adrenalectomy E) Treatment with propylthiouracil

To demonstrate understanding of the diagnosis and treatment of pheochromocytoma Answer B is correct Pheochromocytomas are catecholamine-producing tumors that arise from chromaffin tissue. The majority (85 to 90%) develop in the adrenal gland, but some occur at extra-adrenal sites. Several features of pheochromocytomas have been characterized by a 10% frequency of distribution: 10% are extra-adrenal, 10% are bilateral, 10% occur in children, 10% are familial, and 10% are malignant. Some more recent series, however, have reported lower percentages of malignant pheochromocytomas. The clinical features of pheochromocytoma are related to the effects of increased secretion of norepinephrine and epinephrine. Hypertension is the most consistent feature and may be either paroxysmal or sustained or present as hypertensive emergency. The diagnosis of pheochromocytoma is established by demonstrating elevated levels either of catecholamines and metabolites (metanephrines) in urine or of fractionated metanephrines in plasma. Once the diagnosis of pheochromocytoma has been established biochemically and radiographically, the next step is to prepare the patient for adrenalectomy pharmacologically to prevent an intraoperative hypertensive crisis. Preoperative alpha blockade with phenoxybenzamine is initiated and usually increased until the patient is mildly orthostatic. As alpha blockade progresses, fluids should be administered liberally to fill the expanded intravascular space. If the patient has marked tachycardia or dysrhythmia, beta blockade may also be required. Biopsy can instigate hypertensive crisis and is not indicated.

Two hours following total thyroidectomy for Graves disease, a 31-year-old woman is noted to have a distinct, moderate-size area of swelling and bruising at the surgical site. Initially, she is asymptomatic, but she suddenly experiences neck pain accompanied by dyspnea and stridor. Her O2 saturation remains above 92% on 4 L oxygen via nasal cannula. What is the most appropriate immediate step in treatment? A) Increased oxygen administration and immediate chest x-ray B) Racemic epinephrine nebulizer and admission to the intensive care unit C) Return to the operating room for neck exploration D) Opening of skin incision at the bedside E) Opening of skin incision and strap muscle layer at the bedside

To demonstrate understanding of the diagnosis and treatment of postoperative hematoma following thyroidectomy Answer E is correct Complications specific to thyroidectomy include neck hematoma or seroma, recurrent or external laryngeal nerve injury, and transient or permanent hypoparathyroidism. Postoperative bleeding into the neck can produce life-threatening airway compromise, and any postoperative respiratory distress should raise concern for neck hematoma. Urgent bedside decompression by reopening the wound down through the strap muscles is lifesaving when symptoms are sudden and severe. The patient is then returned to the operating room for wound exploration and reclosure under anesthesia. Neck hematoma development with few or no symptoms merits intensive care unit observation for at least 24 hours. Most bleeding occurs within 4 hours of operation, and virtually all occurs within 24 hours.

You are asked to evaluate a 31-year-old woman on postoperative day 2 following resection of a parathyroid adenoma. She is complaining of progressive perioral numbness and tingling in her fingertips. Three other parathyroid glands were identified as normal glands during the operation. Which of the following statements is false? A) Physical examination may reveal a positive Chvostek sign B) Postoperative hyperparathyroidism may be responsible for these symptoms C) Maintaining a normal serum magnesium level is important in the treatment of this problem D) Hungry bone syndrome could account for this problem E) Treatment includes calcium supplementation

To demonstrate understanding of the diagnosis and treatment of postoperative hypocalcemia following parathyroid resection Answer B is correct Hypocalcemia after parathyroidectomy has a number of causes, including permanent hypoparathyroidism, transient hypoparathyroidism, and a condition referred to as hungry bone syndrome. Symptoms generally do not occur unless serum calcium is less than 8.0 mg/dL. Mild hypocalcemia may present with perioral numbness and tingling in the fingertips, whereas severe hypocalcemia may present with muscle cramping, spasm, and tetany. Signs of hypocalcemia include Chvostek sign, a twitching of the facial muscles when the cheek is tapped over the facial nerve. Trousseau sign is elicited by inflating a sphygmomanometer above systolic blood pressure, which causes muscular contractions with hyperextension of the fingers in some hypocalcemic patients. Hypoparathyroidism may arise as the result of removal, bruising, ischemic injury, or devascularization of the parathyroid glands. Hypocalcemia from hypoparathyroidism may be transient or permanent if all four glands are permanently damaged. Laboratory values in postoperative patients with hypoparathyroidism are likely to show low serum calcium, high phosphorus, and low or even undetectable PTH levels. It is also important to note magnesium levels as calcium replacement can be dependent on maintaining normal magnesium levels as well. Hungry bone syndrome is a phenomenon that occurs after the PTH stimulus for bone resorption has been taken away and the bones suddenly begin to sequester calcium. The consequence of this sequestration is hypocalcemia. Additionally, phosphorus will be absorbed as well, giving low serum phosphorus levels. This phenomenon may last days or, sometimes, even weeks. A severe manifestation of this condition occurs in patients with osteitis fibrosa cystica. In addition to the profound hypocalcemia that may occur after parathyroidectomy, these patients may have very high alkaline phosphatase levels. Patients treated for a single adenoma are more likely to have hypocalcemia secondary to hungry bone syndrome. This generally presents 2 to 3 days postoperatively. Patients being treated with four-gland exploration with multiple resections are more likely to experience permanent hypocalcemia. This can present within 2 hours of surgery. Treatment and prophylaxis for hypocalcemia associated with both problems are similar; in particular, treatment consists of oral calcium, calcitriol, and magnesium when it is also low. It is common practice to prophylax every patient in the postoperative period with daily calcium citrate and vitamin D.

You are asked to evaluate a 31-year-old woman on postoperative day 2 following resection of a parathyroid adenoma. She is complaining of progressive perioral numbness and tingling in her fingertips. Three other parathyroid glands were identified as normal glands during the operation. Which of the following statements is false? A) Physical examination may reveal a positive Chvostek sign B) Postoperative hyperparathyroidism may be responsible for these symptoms C) Maintaining a normal serum magnesium level is important in the treatment of this problem D) Hungry bone syndrome could account for this problem E) Treatment includes calcium supplementation

To demonstrate understanding of the diagnosis and treatment of postoperative hypocalcemia following parathyroid resection Answer B is correct Hypocalcemia after parathyroidectomy has a number of causes, including permanent hypoparathyroidism, transient hypoparathyroidism, and a condition referred to as hungry bone syndrome. Symptoms generally do not occur unless serum calcium is less than 8.0 mg/dL. Mild hypocalcemia may present with perioral numbness and tingling in the fingertips, whereas severe hypocalcemia may present with muscle cramping, spasm, and tetany. Signs of hypocalcemia include Chvostek sign, a twitching of the facial muscles when the cheek is tapped over the facial nerve. Trousseau sign is elicited by inflating a sphygmomanometer above systolic blood pressure, which causes muscular contractions with hyperextension of the fingers in some hypocalcemic patients. Hypoparathyroidism may arise as the result of removal, bruising, ischemic injury, or devascularization of the parathyroid glands. Hypocalcemia from hypoparathyroidism may be transient or permanent if all four glands are permanently damaged. Laboratory values in postoperative patients with hypoparathyroidism are likely to show low serum calcium, high phosphorus, and low or even undetectable PTH levels. It is also important to note magnesium levels as calcium replacement can be dependent on maintaining normal magnesium levels as well. Hungry bone syndrome is a phenomenon that occurs after the PTH stimulus for bone resorption has been taken away and the bones suddenly begin to sequester calcium. The consequence of this sequestration is hypocalcemia. Additionally, phosphorus will be absorbed as well, giving low serum phosphorus levels. This phenomenon may last days or, sometimes, even weeks. A severe manifestation of this condition occurs in patients with osteitis fibrosa cystica. In addition to the profound hypocalcemia that may occur after parathyroidectomy, these patients may have very high alkaline phosphatase levels. Patients treated for a single adenoma are more likely to have hypocalcemia secondary to hungry bone syndrome. This generally presents 2 to 3 days postoperatively. Patients being treated with four-gland exploration with multiple resections are more likely to experience permanent hypocalcemia. This can present within 2 hours of surgery. Treatment and prophylaxis for hypocalcemia associated with both problems are similar; in particular, treatment consists of oral calcium, calcitriol, and magnesium when it is also low. It is common practice to prophylax every patient in the postoperative period with daily calcium citrate and vitamin D.

A 29-year-old female is seen in your office in consultation for surgical management of her chronic UC. She has been plagued for more than 10 years with chronic recalcitrant colitis flares that require frequent admissions to the hospital and high-dose steroids. She has been treated unsuccessfully with a number of immunosuppressants, topical and enteral steroids, and antiinflammatory agents. Which of the following is not true? A) She should be offered a total proctocolectomy with ileal pull-through anastomosis B) Extraintestinal manifestations of UC is an indication for surgery C) Colonic mucosal dysplasia is an indication for surgery in UC D) The estimated risk of colon cancer in a patient with UC is 10 to 15% at 10 years E) Continent ileostomy or Kock pouch is associated with a high rate of pouch dysfunction

To demonstrate understanding of the elective surgical management of ulcerative colitis (UC) Answer D is correct The indications for elective surgery in UC include (1) failure of medical management to control symptoms; (2) complications associated with side effects of medications; (3) stricture formation; (4) mucosal dysplasia, dysplasia-associated lesion or mass (DALM), or malignancy; (5) extraintestinal manifestations of UC; and (6) growth retardation in children. The development of malignancy in the setting of UC has been well described. The estimated risk of colon cancer in a patient with UC has been estimated to be anywhere from 2% at 20 years after onset of UC to 43% at 35 years. A patient's individual risk for colon cancer is likely increased if there is evidence of high-grade dysplasia on random colon biopsies or if there is a DALM. The presence of low-grade dysplasia on random biopsy is a more difficult clinical situation; most clinicians recommend increasing the frequency of surveillance colonoscopies and not surgery. However, some preliminary reports indicate that the presence of any degree of dysplasia not associated with a mass lesion should be viewed with a very high degree of suspicion, and surgery should be recommended. Currently, there are two widely accepted surgical options for the treatment of UC: a total proctocolectomy with an end (Brooke) ileostomy and total proctocolectomy with IPAA. A third option, total abdominal colectomy with an ileorectal anastomosis, may be considered in a highly select group of patients: those who have relatively mild disease and who have a contraindication to an ostomy, such as portal hypertension or ascites, or those in whom an ileal pouch procedure would be technically impossible, and the patient refuses to have an ileostomy. Previously, a fourth option was employed, proctocolectomy with construction of a continent ileostomy or Kock pouch. However, there was such a high rate of pouch dysfunction that often required surgical correction in the majority of patients that this operation is no longer routinely offered or performed. Also, given the outstanding functional results of the IPAA procedure, the continent ileostomy is rarely used today.

Three months following a renal transplantation, you are asked to evaluate a 66-year-old man with elevated serum calcium levels and elevated serum PTH levels. He has had severe constipation and worsening reflux symptoms. Which of the following statements is false? Which of the following statements is false? A) Elevated serum calcium levels are typical of secondary hyperparathyroidism B) Secondary hyperparathyroidism usually does not require surgical management C) Chronic renal failure is the most common cause of secondary hyperparathyroidism D) Tertiary hyperparathyroidism is the long-term consequence of prolonged secondary hyperparathyroidism after the cause is corrected E) Tertiary hyperthyroidism is primarily a surgical disease

To demonstrate understanding of the diagnosis and treatment of secondary and tertiary hyperparathyroidism Answer A is correct Secondary hyperparathyroidism occurs from the physiologic response to low serum calcium levels of nonparathyroid origin. Although the pathophysiology of secondary hyperparathyroidism is complex, it is most commonly caused by chronic renal failure, which interferes with vitamin D metabolism, which in turn leads to decreased calcium absorption from nutritional sources. Other causes include " hungry bone syndrome," sprue, chronic vitamin D deficiency, and aluminum toxicity from hemodialysis. Generally, hypercalcemia is not a characteristic of this disease. Secondary hyperparathyroidism generally is not considered a surgical disease. Instead, these patients are treated with calcimimetics, calcium, and vitamin D replacement. Rarely, with failure of medical management, surgery may be considered. In this case, indications for surgery may include severe pruritis, musculoskeletal pain, renal osteodystrophy, and calciphylaxis. Tertiary hyperparathyroidism is the long-term consequence of prolonged secondary hyperparathyroidism after the cause is corrected. It results in autonomously elevated PTH concentrations as a result of the loss of calcium-sensing receptors in parathyroid tissue. The net result of tertiary hyperparathyroidism is elevated serum calcium. The typical scenario is the patient with secondary hyperparathyroidism attributable to end-stage renal disease that undergoes renal transplantation. In this condition, all four glands are treated as hyperplastic glands. Tertiary hyperthyroidism is primarily a surgical disease. Because all parathyroid tissue is hyperfunctioning in these patients, the preferred operation is four-gland parathyroidectomy with autotransplantation of half of a single gland or subtotal parathyroidectomy. In the latter case, a parathyroid remnant roughly the size of a normal parathyroid gland is left in situ.

Three months following a renal transplantation, you are asked to evaluate a 66-year-old man with elevated serum calcium levels and elevated serum PTH levels. He has had severe constipation and worsening reflux symptoms. Which of the following statements is false? A) Elevated serum calcium levels are typical of secondary hyperparathyroidism B) Secondary hyperparathyroidism usually does not require surgical management C) Chronic renal failure is the most common cause of secondary hyperparathyroidism D) Tertiary hyperparathyroidism is the long-term consequence of prolonged secondary hyperparathyroidism after the cause is corrected E) Tertiary hyperthyroidism is primarily a surgical disease

To demonstrate understanding of the diagnosis and treatment of secondary and tertiary hyperparathyroidism Answer A is correct Secondary hyperparathyroidism occurs from the physiologic response to low serum calcium levels of nonparathyroid origin. Although the pathophysiology of secondary hyperparathyroidism is complex, it is most commonly caused by chronic renal failure, which interferes with vitamin D metabolism, which in turn leads to decreased calcium absorption from nutritional sources. Other causes include "hungry bone syndrome," sprue, chronic vitamin D deficiency, and aluminum toxicity from hemodialysis. Generally, hypercalcemia is not a characteristic of this disease. Secondary hyperparathyroidism generally is not considered a surgical disease. Instead, these patients are treated with calcimimetics, calcium, and vitamin D replacement. Rarely, with failure of medical management, surgery may be considered. In this case, indications for surgery may include severe pruritis, musculoskeletal pain, renal osteodystrophy, and calciphylaxis. Tertiary hyperparathyroidism is the long-term consequence of prolonged secondary hyperparathyroidism after the cause is corrected. It results in autonomously elevated PTH concentrations as a result of the loss of calcium-sensing receptors in parathyroid tissue. The net result of tertiary hyperparathyroidism is elevated serum calcium. The typical scenario is the patient with secondary hyperparathyroidism attributable to end-stage renal disease that undergoes renal transplantation. In this condition, all four glands are treated as hyperplastic glands. Tertiary hyperthyroidism is primarily a surgical disease. Because all parathyroid tissue is hyperfunctioning in these patients, the preferred operation is four-gland parathyroidectomy with autotransplantation of half of a single gland or subtotal parathyroidectomy. In the latter case, a parathyroid remnant roughly the size of a normal parathyroid gland is left in situ.

A 33-year-old female with a long history of UC is nearing the end of a steroid taper for a disease flare. She presents to the emergency department of a university hospital complaining of 6 hours of severe abdominal pain. Her heart rate is 122 beats/min and blood pressure is 89/44 mm Hg. She is diaphoretic and has diffuse tenderness to tap palpation and involuntary guarding. A kidney-ureter-bladder film shows dilation of the colon from the cecum to and including the descending colon with a maximum diameter of 12 cm at the transverse colon. The next appropriate step in management includes: A) CT of the abdomen and pelvis with enteric contrast B) Noncontrast CT of the abdomen and pelvis C) Intravenous steroids D) Admission for bowel rest, intravenous fluids, and serial abdominal examinations E) Total abdominal colectomy with end ileostomy

To demonstrate understanding of the diagnosis and treatment of toxic megacolon Answer E is correct This patient has toxic megacolon and peritonitis and requires immediate laparotomy. Emergency surgery is performed in those patients with UC who present with fulminant disease that cannot be treated medically, toxic megacolon, or massive hemorrhage. In these situations, the primary surgical strategy is to remove the bulk of the diseased colon to allow the patient's medical condition to improve. An important secondary consideration is to perform an operation that will not preclude a future restorative procedure such as IPAA. Toxic megacolon may be the initial presentation of UC or may represent a flare in a patient with long-standing disease. The entire colon or an isolated segment of the colon (usually transverse or the left colon) is involved. Although there is a radiographic definition of toxic megacolon being dilatation of the transverse colon greater than 5.5 cm on a supine abdominal film, toxic megacolon is truly a clinical diagnosis. The medical treatment of toxic megacolon is similar to that used for patients with fulminant colitis, namely, nothing given by mouth, nasogastric tube decompression, correction of fluid deficits and electrolyte abnormalities, high-dosage steroids, and antibiotics if there is fever or an elevated leukocyte count. Emergency surgery is indicated if the patient's clinical or radiographic status worsens, if there is evidence of perforation, or if there is no improvement 24 to 36 hours after beginning aggressive medical therapy. Delaying surgery increases the risk of perforation, which raises the mortality from less than 5% to nearly 30%. Fortunately, fewer patients are seen with the severe complications of UC today because of improved medical therapies and earlier surgical intervention in patients with UC.

A 33-year-old female with a long history of ulcerative colitis (UC) is nearing the end of a steroid taper for a disease flare. She presents to the emergency department complaining of 6 hours of severe abdominal pain. Her heart rate is 122 beats/min and blood pressure is 89/44 mm Hg. She is diaphoretic and has diffuse tenderness to tap palpation and involuntary guarding. A kidney-ureter-bladder film shows dilation of the colon from the cecum to and including the descending colon with a maximum diameter of 12 cm at the transverse colon. The next appropriate step in management includes: A) Computed tomography (CT) of the abdomen and pelvis with enteric contrast B) Noncontrast CT of the abdomen and pelvis C) IV steroids D) Admission for bowel rest, IV fluids, and serial abdominal examinations E) Total abdominal colectomy with end ileostomy

To demonstrate understanding of the diagnosis and treatment of toxic megacolon Answer E is correct This patient has toxic megacolon and peritonitis and requires immediate laparotomy. Emergency surgery is performed in those patients with UC who present with fulminant disease that cannot be treated medically, toxic megacolon, or massive hemorrhage. In these situations, the primary surgical strategy is to remove the bulk of the diseased colon to allow the patient's medical condition to improve. An important secondary consideration is to perform an operation that will not preclude a future restorative procedure such as ileal pouch-anal anastomosis (IPAA). Toxic megacolon may be the initial presentation of UC or may represent a flare in a patient with long-standing disease. The entire colon or an isolated segment of the colon (usually transverse or the left colon) is involved. Although there is a radiographic definition of toxic megacolon as dilatation of the transverse colon greater than 5.5 cm on a supine abdominal film, toxic megacolon is truly a clinical diagnosis. The medical treatment of toxic megacolon is similar to that used for patients with fulminant colitis, namely, nothing given by mouth, nasogastric tube decompression, correction of fluid deficits and electrolyte abnormalities, high-dosage steroids, and antibiotics if there is fever or an elevated leukocyte count. Emergency surgery is indicated if the patient's clinical or radiographic status worsens, if there is evidence of perforation, or if there is no improvement 24 to 36 hours after beginning aggressive medical therapy. Delaying surgery increases the risk of perforation, which raises the mortality from less than 5% to nearly 30%. Fortunately, fewer patients are seen with the severe complications of UC today because of improved medical therapies and earlier surgical intervention in patients with UC.

A 78-year-old female with a strong family history of breast cancer who has a 2.5 cm breast mass is referred to you. Ultrasonography reveals the mass to be solid, and biopsy is recommended. Which of the following statements regarding breast biopsy is true? A) Fine-needle aspiration (FNA) is the diagnostic procedure of choice for solid breast lesions B) FNA cannot discriminate ductal carcinoma in situ from invasive carcinoma C) FNA is typically performed with a 12- to 14-gauge needle D) Malignancies identified by core-needle biopsy should be confirmed by FNA E) A negative result from FNA essentially excludes cancer

To demonstrate understanding of the different modalities of breast biopsy Answer B is correct Cytologic or tissue diagnosis of a palpable breast mass may be obtained by means of FNA biopsy, CNB, or open incisional or excisional biopsy. Currently, most solid lesions are initially diagnosed by means of CNB, which is less invasive, less costly, and more expeditious than open biopsy while achieving comparable accuracy. FNA biopsy is unable to discriminate carcinoma in situ from invasive carcinoma and therefore is incapable of establishing whether axillary node staging is needed. Furthermore, because of the smaller quantity of tissue extracted, FNA biopsy is a less reliable means of assessing receptor status than CNB is. Finally, in 1 to 2% of cases, FNA biopsy may yield false positive results, potentially leading to an unnecessary cancer operation. For these reasons, it is recommended that malignancies identified by FNA biopsy be confirmed by means of CNB or open biopsy (preferably the former) before definitive therapy is provided. Cytologic analysis that is diagnostic of a specific benign lesion (e.g., a fibroadenoma) may generally be relied on if it is in concordance with the clinical features of the lesion. However, a negative result from FNA biopsy does not exclude cancer: this procedure fails to diagnose as many as 40% of breast malignancies. Although smaller needles may be used, a 21-gauge needle is optimal for FNA biopsy because it can be effectively used for both aspiration of potentially viscous fluid and procurement of sufficient cellular material from solid masses.

A 29-year-old female is seen in your office in consultation for surgical management of her chronic UC. She has been plagued for more than 10 years with chronic recalcitrant colitis flares that require frequent admissions to the hospital and high-dose steroids. She has been treated unsuccessfully with a number of immunosuppressants, topical and enteral steroids, and antiinflammatory agents. Which of the following is not true? A) She should be offered a total proctocolectomy with ileal pull-through anastomosis B) Extraintestinal manifestations of UC are an indication for surgery C) Colonic mucosal dysplasia is an indication for surgery in UC D) The estimated risk of colon cancer in a patient with US is 10 to 15% at 10 years E) Continent ileostomy or Kock pouch is associated with a high rate of pouch dysfunction

To demonstrate understanding of the elective surgical management of ulcerative colitis (UC) Answer D is correct The indications for elective surgery in UC include (1) failure of medical management to control symptoms; (2) complications associated with side effects of medications; (3) stricture formation; (4) mucosal dysplasia, dysplasia-associated lesion or mass (DALM), or malignancy; (5) extraintestinal manifestations of UC; and (6) growth retardation in children. The development of malignancy in the setting of UC has been well described. The estimated risk of colon cancer in a patient with UC has been estimated to be anywhere from 2% at 20 years after onset of UC to 43% at 35 years. A patient's individual risk for colon cancer is likely increased if there is evidence of high-grade dysplasia on random colon biopsies or if there is a DALM. The presence of low-grade dysplasia on random biopsy is a more difficult clinical situation; most clinicians recommend increasing the frequency of surveillance colonoscopies and not surgery. However, some preliminary reports indicate that the presence of any degree of dysplasia not associated with a mass lesion should be viewed with a very high degree of suspicion, and surgery should be recommended. Currently, there are two widely accepted surgical options for the treatment of UC: a total proctocolectomy with an end (Brooke) ileostomy and total proctocolectomy with IPAA. A third option, total abdominal colectomy with an ileorectal anastomosis, may be considered in a highly select group of patients: those who have relatively mild disease and who have a contraindication to an ostomy, such as portal hypertension or ascites, or those in whom an ileal pouch procedure would be technically impossible, and the patient refuses to have an ileostomy. Previously, a fourth option was employed, proctocolectomy with construction of a continent ileostomy or Kock pouch. However, there was such a high rate of pouch dysfunction that often required surgical correction in the majority of patients that this operation is no longer routinely offered or performed. Also, given the outstanding functional results of the IPAA procedure, the continent ileostomy is rarely used today.

A 54-year-old female with a 7 cm right adnexal mass is referred to you. A percutaneous biopsy reveals ovarian cancer. Which of the following statements regarding her diagnosis is false? A) Approximately 5 to 10% of ovarian cancers have a hereditary component B) The majority of patients with ovarian cancer have localized disease (stage I or II) disease C) The CA 125 antigen is not a useful screening test in patients with ovarian cancer D) The staging of ovarian cancer is based on the pathologic findings at surgery E) Patients with advanced disease of the peritoneal cavity at operation should undergo surgical debulking

To demonstrate understanding of the staging and treatment of ovarian cancer Answer B is correct Approximately 5 to 10% of these cancers have a hereditary component; women with BRCA1/BRCA2 or Lynch syndrome (hereditary nonpolyposis colorectal cancer) germline mutations are at increased risk. Ovarian epithelial carcinomas spread primarily through exfoliation of cells, which places the entire peritoneal cavity at risk for metastases. Moreover, early recognition is difficult because the majority of patients have only vague and nonspecific complaints. Not surprisingly, 60 to 70% of patients have advanced (stage III or IV) disease at diagnosis. Detection of ovarian cancer is further hindered by the fact that there is no effective screening test. The tumor-associated antigen CA 125 is expressed by more than 80% of ovarian epithelial cancers; however, although CA 125 levels are employed in following up patients with documented disease, they are not useful for screening purposes. Surgical intervention serves three purposes in the setting of ovarian epithelial cancer: diagnosis, staging, and tumor debulking (cytoreduction). The staging of ovarian cancer is based on the pathologic findings at surgery. Only a minority of women have stage I disease at diagnosis, and the decision whether to employ paclitaxel and carboplatinum chemotherapy is predicated on the surgical extent of the disease. In patients who appear to have early rather than advanced disease, the emphasis is on accurate staging. Because the majority of patients with ovarian epithelial cancers present with advanced disease, the main goal of surgical intervention is often tumor debulking. The purpose of cytoreduction is to remove the primary cancer and any metastases. Optimal cytoreduction enhances the response to IV paclitaxel and carboplatinum chemotherapy, and patients with minimal residual disease have a distinct survival advantage over those with more substantial residual disease.

During surgical exploration for a parathyroid adenoma, you are having difficulty locating the lesional gland. Which of the following is false regarding the parathyroid glands? A) During development, the superior gland descends into the neck from branchial pouch IV and the inferior gland descends into the neck from branchial pouch III B) The most common ectopic location of a parathyroid gland is within the thyroid itself C) All four parathyroid glands derive their blood supply from the inferior parathyroid artery D) The incidence of supernumerary glands is about 10 to 15% E) In general, the superior parathyroid glands can be found about 1 cm superior to the inferior thyroid artery

To demonstrate understanding of the embryology and anatomy of the parathyroid glands . Answer B is correct The parathyroid glands arise during the fourth and fifth weeks of gestation. The superior glands descend into the neck from branchial pouch IV, whereas the inferior glands descend into the neck from branchial pouch III. In general, the superior glands settle 1 cm superior to the inferior thyroid artery in a position posterior to the thyroid gland, and the inferior glands settle at the lower pole of the thyroid gland in a posterolateral position. The superior and inferior parathyroids are consistently located posterior and anterior to the recurrent laryngeal nerve, respectively. However, the parathyroid glands can be found in ectopic locations. Approximately 60% of parathyroids on reoperation are not located in the typical locations. The inferior glands have a higher propensity for this in comparison with the upper glands. Common ectopic locations of the inferior glands at reoperation include the thymus (46%), within the thyroid itself (7%), undescended (4%), and the carotid sheath (1%). Common ectopic locations of the superior glands include the posterior superior mediastinum (40%), the tracheoesophageal groove (17%), within the thyroid (1.5%), and the carotid sheath (1.5%). Most commonly, there are four parathyroid glands; however, the incidence of supernumerary glands is around 13%. All four glands derive their blood supply from the inferior thyroid artery. Venous drainage is by the venous network of the thyroid capsule and/or venous pedicles of the thyroid body.

During surgical exploration for a parathyroid adenoma, you are having difficulty locating the lesional gland. Which of the following is false regarding the parathyroid glands? A) During development, the superior gland descends into the neck from branchial pouch IV and the inferior gland descends into the neck from branchial pouch III B) The most common ectopic location of a parathyroid gland is within the thyroid itself C) All four parathyroid glands derive their blood supply from the inferior parathyroid artery D) The incidence of supernumerary glands is about 10 to 15% E) In general, the superior parathyroid glands can be found about 1 cm superior to the inferior thyroid artery

To demonstrate understanding of the embryology and anatomy of the parathyroid glands Answer B is correct The parathyroid glands arise during the fourth and fifth weeks of gestation. The superior glands descend into the neck from branchial pouch IV, whereas the inferior glands descend into the neck from branchial pouch III. In general, the superior glands settle 1 cm superior to the inferior thyroid artery in a position posterior to the thyroid gland, and the inferior glands settle at the lower pole of the thyroid gland in a posterolateral position. The superior and inferior parathyroids are consistently located posterior and anterior to the recurrent laryngeal nerve, respectively. However, the parathyroid glands can be found in ectopic locations. Approximately 60% of parathyroids on reoperation are not located in the typical locations. The inferior glands have a higher propensity for this in comparison with the upper glands. Common ectopic locations of the inferior glands at reoperation include the thymus (46%), within the thyroid itself (7%), undescended (4%), and the carotid sheath (1%). Common ectopic locations of the superior glands include the posterior superior mediastinum (40%), the tracheoesophageal groove (17%), within the thyroid (1.5%), and the carotid sheath (1.5%). Most commonly, there are four parathyroid glands; however, the incidence of supernumerary glands is around 13%. All four glands derive their blood supply from the inferior thyroid artery. Venous drainage is by the venous network of the thyroid capsule and/or venous pedicles of the thyroid body.

68-year-old man presents to his primary care physician with a cough that has been present for approximately 4 weeks. What detail of the history suggests that a nonmalignant cause of cough is more likely? A) Patient's sputum is bloody B) Patient has a 50-pack-year smoking history C) Patient worked in a shipyard D) Patient reports lower extremity edema E) Patient has melanotic stools

To demonstrate understanding of the history features suggestive of malignant as opposed to nonmalignant lung disease Answer D is correct A cough requires diagnostic attention when it persists for 3 weeks or longer, at which time, it is deemed chronic. Careful diagnostic evaluation is required to identify potentially life-threatening causes of chronic coughing. Tobacco use, occupational exposure, and bloody sputum all should raise concern for an underlying malignancy. As metastatic tumors to the lung are more frequent than primary lung cancers, signs and symptoms of cancers likely to metastasize to the lung, such as colon cancer, should also be noted. Congestive heart failure, which may also give rise to lower extremity edema, is a nonmalignant cause of chronic cough.

In which of the following situations is partial mastectomy not indicated? A) Previous partial mastectomy for infiltrating ductal adenocarcinoma with 4 mm margins B) Previous partial mastectomy for infiltrating ductal adenocarcinoma with 0.5 mm margins C) Core-needle biopsy (CNB) consistent with ductal ectasia D) CNB consistent with radial scar E) CNB consistent with papilloma

To demonstrate understanding of the indications for partial mastectomy and the significance of margins Answer C is correct. Partial mastectomy—also referred to as wide local excision or lumpectomy involves excision of all cancerous tissue to microscopically clear margins. Although 1 cm margins are the goal, many surgeons consider 2 mm margins to be adequate for reducing the risk of local recurrence. Hence, reexcision is indicated whenever margins are either positive or too close (< 2 mm). Partial mastectomy is commonly performed with the patient under local anesthesia, with or without sedation. The addition of an axillary staging procedure (a common event) usually necessitates general anesthesia, but in select circumstances, local or epidural anesthesia may suffice. CNB is a highly accurate diagnostic tool: the false negative rate is only 1 to 2%, which is comparable to that of open wire-localized biopsy. When pathologic evaluation reveals fibroadenoma, microcalcifications within benign fibrocystic tissue, or other comparably benign pathologic conditions, there is no need for any special follow-up, and routine screening mammography may be resumed. However, when biopsy of the targeted mass lesion fails to yield a mass diagnosis or a biopsy specimen from a group of clustered calcifications is devoid of microcalcifications on pathologic review, the discordant result should be viewed with some suspicion and should be considered an indication for open biopsy. Subsequent excisional biopsy is also indicated when CNB reveals atypical hyperplasia, radial scar, lobular carcinoma in situ, or papilloma. The rationale is that the excisional biopsy may result in a pathologic upgrade to cancer. False positive results are rare with CNB; therefore, a diagnosis of malignancy may be believed, and a one-stage definitive surgical procedure may then be planned without further biopsy. Ductal ectasia is a benign condition characterized by dilated ducts resulting from obstruction with keratin plugs and subsequent inflammation, secretions, and galactorrhea.

A 44-year-old female is seen in your office with a 1 cm, biopsy-proven, poorly differentiated adenocarcinoma of the hepatic flexure of the colon. Positron emission tomography-CT reveals no evidence of metastatic disease. Which of the following regarding oncologic principles of colon resection is not true? A) At least 8 to 10 lymph nodes should be harvested to ensure a high degree of staging accuracy B) This patient should be offered an extended right hemicolectomy C) The involved mesenteric vessels should be ligated at their point of origin D) If the tumor is equidistant from two named mesenteric vessels, both vessels should be ligated E) Wound recurrence rates for laparoscopic resections are no different from those with open operations

To demonstrate understanding of the surgical principles of colon resection for cancer Answer A is correct Colon resections for malignancy should include radical en bloc removal of the draining lymphovascular complex, with bowel margins wide enough to limit intraluminal and pericolic (lymphatic) recurrence. The drainage of the lymphatic system mirrors that of the vascular system. In the case of colon carcinomas, there are two possible directions for lymphatic drainage: (1) paraintestinal (along the intestine) and (2) central (along named mesenteric vessels). To prevent regional lymphatic recurrence, the major draining mesenteric vessel should be divided at the point of origin, together with the accompanying lymphatic network. If the tumor is equidistant from two named mesenteric vessels, both vessels should be ligated proximally. Although, in most cases, intramural spreading of cancer should not exceed 2 cm, an oncologic resection of the abdominal colon should aim at achieving proximal and distal margins of at least 5 to 10 cm to ensure adequate procurement of the epicolic and pericolic lymph nodes. The two exceptions to this rule are in resection of rectal cancer, where a margin of 1 to 2 cm is accepted as part of a sphincter-saving operation, and in resection of a cecal carcinoma, where the ileum can be divided close to the ileocecal valve without compromising the oncologic outcome, provided that the lymphatic vessels along the ileocolic pedicle are removed. The ultimate length of the resected bowel segment is dictated by the lymphovascular resection. An adequate lymphadenectomy is undoubtedly important for accurate staging, and the current recommendation is that at least 13 lymph nodes should be harvested to ensure a high degree of staging accuracy. Fear of port-site recurrences initially kept laparoscopy from being widely accepted in the treatment of colorectal carcinoma, but several randomized, controlled studies reported that the wound recurrence rates with laparoscopic resections were no different from those with open resections. Particularly significant were the long-awaited results of the randomized, prospective trial carried out by the Clinical Outcomes of Surgical Therapy (COST) Study Group, which found the oncologic outcomes of open and laparoscopic surgery to be similar after a median follow-up period of 4.4 years.

An 81-year-old male has a well-differentiated rectal cancer that is 4 cm from the rectal verge. An endorectal ultrasound reveals a T1N0 tumor. Which of the following represents the best management option? A) Definitive chemotherapy and radiation B) AFulguration C) Transanal local excision D) Low anterior resection E) Abdominoperineal resection

To demonstrate understanding of the management of a low rectal cancer Answer C is correct The histologic hallmark of DCIS is the presence of malignant-appearing cells confined to the lumen of the ductal system in the breast. DCIS is most often diagnosed because of the presence of calcifications clustered in one area of the breast. The calcifications suspicious for DCIS are often linear (not round), growing in a ductal distribution. DCIS is generally considered a precursor to invasive cancer. The purpose of treatment of DCIS is to prevent progression to invasive cancer with the sequelae of metastatic disease and death. All women diagnosed with DCIS should undergo appropriate treatment. Mastectomy is effective in DCIS, but in most cases, a breast-conserving approach is possible. Most women with DCIS may be treated by surgical excision ("lumpectomy" or "wide excision") of the DCIS with some surrounding normal breast tissue to achieve a "negative margin." The choice between breast-conserving surgery and mastectomy may be influenced by the size of the area of involvement, the size of the breast, and the location of the DCIS. Large-scale randomized clinical trials in North America and Europe have demonstrated the effectiveness of breast conservation therapy and the role of radiation in DCIS. The National Surgical Adjuvant Breast and Bowel Project (NSABP), the European Organization for Research and Treatment of Cancer (EORTC), and others conducted trials comparing wide excision alone versus wide excision plus radiation therapy. Every study demonstrated long-term overall survival approaching 100% irrespective of treatment. However, women treated with breast conservation therapy had a substantial risk of recurrence in the same breast, and radiation reduced this risk. The rate of local recurrence in the NSABP study was 30% without radiation and about 12% with radiation.

A fit 29-year-old woman with no medical history is seen in your office with the complaint of a lump in her neck. On examination, she has a 2.5 cm nodule in her right anterior neck at the level of the thyroid cartilage. It is soft, mobile, and nontender. There is no associated lymphadenopathy. Her serum TSH and T3 and T4 are within normal limits. Ultrasonography confirms a single 2.5 cm solid mass of the thyroid gland. Which of the following statements is false? A) Thin-walled cysts on a sonogram are most likely benign B) Malignant nodules are more often hypoechoic and often irregular in shape on a sonogram C) A "benign" FNA diagnosis should be confirmed with a repeat FNA D) A "nondiagnostic" result on repeated FNA should prompt thyroid lobectomy E) A "suspicious for malignancy" result on FNA requires thyroid lobectomy as minimal treatment

To demonstrate understanding of the management of a solitary euthyroid thyroid nodule Answer C is correct. Thyroid nodules are very common and represent a wide spectrum of thyroid disease, both benign and malignant. Further assessment is accomplished with ultrasonography, a safe, noninvasive, cost-effective means of imaging thyroid parenchyma and the cervical nodal basins. Benign nodules typically are round, homogeneous, slightly hypoechoic, and easily separable from surrounding tissue. Malignant nodules are more often irregular in shape, heterogeneous, markedly hypoechoic, and poorly demarcated. Thin-walled cysts without internal echoes are overwhelmingly benign; mixed cystic-solid nodules are indeterminate and grouped with solid lesions for further evaluation. FNA provides the most accurate, cost-effective assessment of thyroid nodules. There are several possible results of FNA: 1. Nondiagnostic. About 10 to 15% of aspirates are nondiagnostic, most often attributable to technical error, inadequate sampling, or a substantial cystic component. Less than 5% of these are malignant. Repeat FNA often yields adequate and definitive material, and persistently insufficient results should prompt lobectomy. 2. Benign. A benign diagnosis is rendered in nearly 70% of all specimens, and no intervention is required. The risk of malignancy is 0.5 to 3%. 3. Atypia or follicular lesion of undetermined significance. This is a heterogeneous subset of cytologically indeterminate nodules that are managed like in the same manner as a nondiagnostic FNA: repeat FNA and then lobectomy if a definitive diagnosis remains elusive. Cancer is present in 15 to 30% of cases "suspicious for follicular neoplasm." Furthermore, the distinction between benign follicular adenoma and follicular carcinoma cannot be made by FNA cytology. Assessment for malignancy is based on capsular or vascular invasion, and lobectomy is indicated for tissue diagnosis. Completion thyroidectomy is performed for final diagnoses of follicular cancer or follicular variant of papillary cancer. Some patients opt for total thyroidectomy initially to avoid the need for reoperation; this strategy is reasonable when complication rates are kept low. 4. FNA suspicious for malignancy. A small number of abnormal cells within a sparsely cellular or more benign-looking background characterize lesions in this category. Malignancy rates are 60 to 75%. Thyroid lobectomy is the minimum adequate treatment, and total thyroidectomy is preferable when complication rates are low. 5. Malignant. Roughly 5% of cytologic diagnoses are conclusively malignant, and the positive predictive value of FNA for cancer is 97 to 99%. Comments about the cell type are usually provided. Total thyroidectomy is the preferred treatment for well-differentiated thyroid cancer and medullary thyroid carcinoma, whereas lobectomy suffices for unilateral metastases from other primary sites. Resections for primary thyroid lymphoma and anaplastic thyroid cancer are tailored to the extent of disease, although resection for anaplastic thyroid cancer is usually aimed at palliation only.

A 41-year-old male patient was found to have a 5.2 cm solid lesion within his right adrenal gland on a noncontrast computed tomographic (CT) scan for nephrolithiasis. Regarding the management of this lesion, which of the following statements is false? A) Most adrenal incidentalomas do not secrete excess adrenal hormones B) Needle biopsy should be performed before considering operative management C) Right adrenalectomy is a reasonable treatment option D) Biochemical screening should be performed and should include a dexamethasone test and measurement of plasma metanephrines E) Cortical adenoma is the most common adrenal incidentaloma

To demonstrate understanding of the management of an adrenal incidentaloma Answer B is correct Most adrenal incidentalomas are clinically silent, do not secrete excess adrenal hormones, and do not cause pain. The differential diagnosis of an adrenal incidentaloma includes all of the functioning adrenal tumors, primary or metastatic adrenal malignancy, and various nonfunctioning lesions. Nonfunctioning cortical adenoma is the most common adrenal incidentaloma, accounting for approximately 60% of cases. Other miscellaneous adrenal lesions that are sometimes seen in this setting are myelolipomas and cysts. For all patients with adrenal incidentalomas, biochemical screening should include, at a minimum, an overnight dexamethasone test to exclude hypercortisolism and measurement of the concentrations of either plasma metanephrines or urinary catecholamines and metabolites to test for pheochromocytoma. Measurement of plasma aldosterone and renin levels is indicated for hypertensive or hypokalemic patients with an incidentally discovered adrenal mass. On imaging studies, size is the variable that should be addressed first; a lesion smaller than 4 cm in a patient without a known malignancy is very unlikely to be malignant. Needle biopsy of adrenal masses is rarely indicated. It does not often yield diagnostic information, it cannot differentiate benign from malignant adrenal cortical tumors, and it is associated with a not insignificant risk of complications. Moreover, if the lesion is an undiagnosed pheochromocytoma, needle biopsy may lead to hypertensive crisis and sudden death. Most adrenal surgeons recommend removing nonfunctioning adrenal masses larger than 5 cm and observing masses smaller than 3 cm unless they are obvious myelolipomas or cysts. Management of masses 4 to 5 cm in size should be individualized, with adrenalectomy favored if the patient is young or if the imaging features are at all atypical for an adenoma. Patients for whom resection is not appropriate should be followed with repeat imaging at 4 to 6 months and with both repeat imaging and biochemical testing at 1 year. If the lesion is nonfunctioning and stable in size throughout the follow-up period, no further monitoring is required.

A 41-year-old male patient was found to have a 5.2 cm solid lesion within his right adrenal gland on a noncontrast computed tomographic (CT) scan for nephrolithiasis. Regarding the management of this lesion, which of the following statements is false? A) Most adrenal incidentalomas do not secrete excess adrenal hormones B) Needle biopsy should be performed before considering operative management C) Right adrenalectomy is a reasonable treatment option D) Biochemical screening should be performed and should include a dexamethasone test and measurement of plasma metanephrines E) Cortical adenoma is the most common adrenal incidentaloma

To demonstrate understanding of the management of an adrenal incidentaloma Answer B is correct Most adrenal incidentalomas are clinically silent, do not secrete excess adrenal hormones, and do not cause pain. The differential diagnosis of an adrenal incidentaloma includes all of the functioning adrenal tumors, primary or metastatic adrenal malignancy, and various nonfunctioning lesions. Nonfunctioning cortical adenoma is the most common adrenal incidentaloma, accounting for approximately 60% of cases. Other miscellaneous adrenal lesions that are sometimes seen in this setting are myelolipomas and cysts. For all patients with adrenal incidentalomas, biochemical screening should include, at a minimum, an overnight dexamethasone test to exclude hypercortisolism and measurement of the concentrations of either plasma metanephrines or urinary catecholamines and metabolites to test for pheochromocytoma. Measurement of plasma aldosterone and renin levels is indicated for hypertensive or hypokalemic patients with an incidentally discovered adrenal mass. On imaging studies, size is the variable that should be addressed first; a lesion smaller than 4 cm in a patient without a known malignancy is very unlikely to be malignant. Needle biopsy of adrenal masses is rarely indicated. It does not often yield diagnostic information, it cannot differentiate benign from malignant adrenal cortical tumors, and it is associated with a not insignificant risk of complications. Moreover, if the lesion is an undiagnosed pheochromocytoma, needle biopsy may lead to hypertensive crisis and sudden death. Most adrenal surgeons recommend removing nonfunctioning adrenal masses larger than 5 cm and observing masses smaller than 3 cm unless they are obvious myelolipomas or cysts. Management of masses 4 to 5 cm in size should be individualized, with adrenalectomy favored if the patient is young or if the imaging features are at all atypical for an adenoma. Patients for whom resection is not appropriate should be followed with repeat imaging at 4 to 6 months and with both repeat imaging and biochemical testing at 1 year. If the lesion is nonfunctioning and stable in size throughout the follow-up period, no further monitoring is required.

A 32-year-old woman is in the fifth month of her pregnancy. In the workup of a 2.5 cm left breast mass, a CNB is performed and reveals invasive ductal carcinoma. Which of the following statements regarding breast cancer during pregnancy is false? A) Mammography and ultrasonography can be safely performed in pregnancy B) Chemotherapy can be safely delivered during this stage of her pregnancy C) Surgery can be safely performed during this stage of her pregnancy D) Radiation can be safely performed during this stage of her pregnancy E) Sentinel lymph node biopsy can be safely performed during this stage of her pregnancy

To demonstrate understanding of the management of breast cancer during pregnancy Answer D is correct Breast cancer is uncommon in young women but may occur during the childbearing years. Therefore, breast cancer may occur during pregnancy and, in fact, is the second most common cancer associated with pregnancy. Women in this age group are generally not undergoing routine breast cancer screening, and changes in the breast associated with pregnancy can mask the presence of a mass. Therefore, breast cancer diagnosis is often delayed in this situation. Women with a suspicious breast mass detected during pregnancy should be evaluated aggressively. Imaging can generally be done with judicious use of mammography, ultrasonography, and biopsy performed as in any woman with a suspicious breast lesion. The principles of treatment are the same as in any woman with breast cancer. The only limitations are that chemotherapy cannot be administered during the first trimester and radiation cannot be administered until after delivery. However, neither of these issues limits aggressive treatment of women with breast cancer during pregnancy. Termination of the pregnancy is not necessary, although some women diagnosed early in pregnancy may choose to do so. Older texts suggested that mastectomy is the preferred treatment in women with breast cancer during pregnancy. However, the same choices available to nonpregnant women with cancer are available to pregnant women with cancer because the radiation that is administered is generally delayed until after chemotherapy and therefore is timed to occur after delivery of the child. Therefore, breast conservation can be used safely during pregnancy. Similarly, chemotherapy may be used safely during pregnancy. There are extensive data demonstrating the safety of chemotherapy for both the mother and the fetus. Sentinel lymph node biopsy may also be performed during pregnancy using radioactive tracers. The estimated radiation dose to the fetus is exceedingly low and is considered safe. The vital dyes isosulfan blue and methylene blue have potential teratogenic effects and are contraindicated during pregnancy.

A 44-year-old female is seen in your office with a 1 cm, biopsy-proven, poorly differentiated adenocarcinoma of the hepatic flexure of the colon. Positron emission tomography-CT reveals no evidence of metastatic disease. Which of the following regarding oncologic principles of colon resection is not true? A) At least eight to 10 lymph nodes should be harvested to ensure a high degree of staging accuracy B) This patient should be offered an extended right hemicolectomy C) The involved mesenteric vessels should be ligated at their point of origin D) If the tumor is equidistant from two named mesenteric vessels, both vessels should be ligated E) Wound recurrence rates for laparoscopic resections are no different from those with open operations

To demonstrate understanding of the surgical principles of colon resection for cancer Answer A is correct Colon resections for malignancy should include radical en bloc removal of the draining lymphovascular complex, with bowel margins wide enough to limit intraluminal and pericolic (lymphatic) recurrence. The drainage of the lymphatic system mirrors that of the vascular system. In the case of colon carcinomas, there are two possible directions for lymphatic drainage: (1) paraintestinal (along the intestine) and (2) central (along named mesenteric vessels). To prevent regional lymphatic recurrence, the major draining mesenteric vessel should be divided at the point of origin, together with the accompanying lymphatic network. If the tumor is equidistant from two named mesenteric vessels, both vessels should be ligated proximally. Although, in most cases, intramural spreading of cancer should not exceed 2 cm, an oncologic resection of the abdominal colon should aim at achieving proximal and distal margins of at least 5 to 10 cm to ensure adequate procurement of the epicolic and pericolic lymph nodes. The two exceptions to this rule are in resection of rectal cancer, where a margin of 1 to 2 cm is accepted as part of a sphincter-saving operation, and in resection of a cecal carcinoma, where the ileum can be divided close to the ileocecal valve without compromising the oncologic outcome, provided that the lymphatic vessels along the ileocolic pedicle are removed. The ultimate length of the resected bowel segment is dictated by the lymphovascular resection. An adequate lymphadenectomy is undoubtedly important for accurate staging, and the current recommendation is that at least 13 lymph nodes should be harvested to ensure a high degree of staging accuracy. Fear of port-site recurrences initially kept laparoscopy from being widely accepted in the treatment of CRC, but several randomized, controlled studies reported that the wound recurrence rates with laparoscopic resections were no different from those with open resections. Particularly significant were the long-awaited results of the randomized, prospective trial carried out by the Clinical Outcomes of Surgical Therapy (COST) Study Group, which found the oncologic outcomes of open and laparoscopic surgery to be similar after a median follow-up period of 4.4 years.

A 51-year-old woman is found to have a 4 cm, centrally located mass for which you are planning a mastectomy. Which of the following statements regarding mastectomy is true? A) She is a good candidate for nipple-sparing mastectomy B) Skin-sparing mastectomy is oncologically safe and not associated with an increased incidence of local recurrence C) During mastectomy, all breast tissue is removed down to the level of, but not including, the pectoral fascia D) A modified radical mastectomy consists of wide local excision and axillary node dissection E) The blood supply to the pedicled latissimus dorsi flap reconstruction is the thoracoacromial artery

To demonstrate understanding of the technical aspects of mastectomy Answer B is correct The goal of a mastectomy is to remove all breast tissue—including the nipple, the areola, and the pectoral fascia— while leaving viable skin flaps and a smooth chest wall for application of prosthesis. This should be the objective whether the mastectomy is performed for cancer treatment or for prophylaxis. Modified radical mastectomy essentially consists of an axillary node dissection added to a simple mastectomy. At the lateral edge of the dissection, the border of the latissimus dorsi is exposed, as is the lateral border of the pectoral muscle. Skin-sparing mastectomy, which consists of resection of the nipple-areola complex, any existing biopsy scar, and the breast parenchyma, followed immediately by breast reconstruction, has become an increasingly popular approach for women requiring mastectomy. With this approach, the inframammary fold and contour of the breast are preserved, and a generous skin envelope remains in situ for reconstruction; cosmetic results are thereby optimized. In addition, skin-sparing mastectomy is oncologically safe and is not associated with an increased incidence of local recurrence. Following mastectomy or skin-sparing mastectomy, many patients report dissatisfaction with the reconstruction of their nipple-areola complex using skin grafting or tattooing. Nipple-sparing mastectomy eliminates the need for reconstruction by preservation of the dermis and epidermis of the nipple-areola complex during the initial procedure. Several initial studies have shown nipple-sparing mastectomy to be an oncologically safe procedure in those patients appropriately selected. Contraindications to nipple-sparing mastectomy include larger tumors (> 2 cm), centrally located lesions with a small tumor-to-nipple distance, and lymphovascular invasion. In a pedicled latissimus dorsi myocutaneous flap reconstruction, the ipsilateral latissimus dorsi is transferred along with overlying skin and fat based on the thoracodorsal vessels to create a breast mound.

A 61-year-old female patient undergoes a core-needle biopsy for suspicious calcifications seen on screening mammography. She does not have a family history of breast cancer or other risk factors that place her at high risk for breast cancer. The pathology is consistent with lobular carcinoma in situ (LCIS). Which of the following statements regarding LCIS is false? A) LCIS is itself a benign lesion B) LCIS conveys an increased lifelong risk of subsequent invasive cancer C) This patient should undergo surgical excision of the surrounding tissue D) Chemoprevention with tamoxifen or raloxifene is a reasonable treatment option for this patient E) Bilateral mastectomy is a reasonable treatment option for this patient

To demonstrate understanding of the management of lobular carcinoma in situ (LCIS) Answer E Small uniform cells confined to the lobule of the breast characterize LCIS. It is generally a clinically and mammographically occult lesion that is identified only incidentally when a biopsy is performed for calcifications or a mass that proves to be some other benign lesion. LCIS is actually not cancer but rather is a benign lesion and does not require cancer treatment per se. The primary issue with LCIS is that it conveys an increased lifelong risk of subsequent invasive cancer quantified at 0.5 to 0.75% per year. In addition, when LCIS is identified on a core-needle biopsy, there is a 10 to 20% chance of ductal carcinoma in situ (DCIS) or invasive cancer in the surrounding tissue; therefore, surgical excision is warranted. Long-term follow-up shows that the large majority of women with LCIS never develop invasive breast cancer. Therefore, ablative surgical therapy and radiation for LCIS are not necessary. Previously, LCIS was considered in and of itself an indication to consider bilateral mastectomy. However, mastectomy is generally not indicated in women with LCIS and should be performed only in the context of risk reduction for those at very high risk related to factors such as inherited susceptibility. Women with a biopsy showing LCIS may also consider risk-reducing chemoprevention with one of the selective estrogen receptor modulators (SERMs), tamoxifen or raloxifene. These reduce the risk of subsequent invasive cancer by about 50%, with an acceptable toxicity profile. Raloxifene is the preferred agent in postmenopausal women.

A 58-year-old female is found to have a 6 cm right ovarian complex mass on a pelvic sonogram and CT scan. A percutaneous biopsy is performed and reveals a mucinous ovarian cancer. She undergoes laparotomy with the intention of resection and is found to have a firm, fleshy mass of the right ovary extending along the right fallopian tube with many small (1 to 5 mm) tumor implants on the serosal surfaces of the small bowel, omentum, and liver. What is the most appropriate treatment strategy? A) Close and refer for systemic chemotherapy B) Close and refer for systemic chemotherapy and radiation C) Total abdominal hysterectomy and bilateral oopherectomy D) Total abdominal hysterectomy and bilateral oopherectomy, resection of intestinal serosal metastases, omentectomy, and pelvic lymph node dissection E) Total abdominal hysterectomy and bilateral oopherectomy and intraperitoneal chemotherapy

To demonstrate understanding of the management of ovarian cancer Answer D is correct The peak incidence of invasive ovarian epithelial cancers is between 56 and 60 years of age. Approximately 5 to 10% of these cancers have a hereditary component; women with BRCA1/BRCA2 or Lynch syndrome (HNPCC) germline mutations are at increased risk. Ovarian epithelial carcinomas spread primarily through exfoliation of cells, which places the entire peritoneal cavity at risk for metastases. Moreover, early recognition is difficult because the majority of patients have only vague and nonspecific complaints. Not surprisingly, 60 to 70% of patients have advanced (stage III or IV) disease at diagnosis. Surgical intervention serves three purposes in the setting of ovarian epithelial cancer: diagnosis, staging, and tumor debulking (cytoreduction). The staging of ovarian cancer is based on the pathologic findings at surgery. In stage I, tumor growth is limited to the ovaries. In stage II, the tumor involves one or both ovaries, with pelvic extension. In stage III, the tumor involves one or both ovaries with peritoneal implants outsidef the pelvis. In stage IV, there are distant metastases. Only a minority of women have stage I disease at diagnosis, and the decision whether to employ paclitaxel and carboplatinum chemotherapy is predicated on the surgical extent of the disease. In patients who appear to have early rather than advanced disease, the emphasis is on accurate staging. A midline incision is made, and on entry into the peritoneal cavity, any free abdominal fluid is submitted for cytologic evaluation. If no abdominal fluid is present, pelvic washings are done with 50 to 100 mL of saline, and the saline is recovered and submitted for cytologic evaluation. The ovarian tumor should be removed intact (if possible) and submitted for frozen-section analysis. The intra-abdominal surfaces and the viscera are systematically inspected. Any suspicious areas or adhesions are subjected to biopsy. Random peritoneal biopsies are performed on the lateral upper and lower paracolic gutters, the bladder flap, the cul-de-sac, and the pelvic sidewalls. Sterile tongue depressors are used to obtain cytologic specimens from both infradiaphragmatic surfaces. An infracolic omentectomy is performed. The regional nodes are palpated, and any enlarged lymph nodes are removed. Finally, the pelvic and para-aortic nodes are sampled to allow detection of any occult spread. Because the majority of patients with ovarian epithelial cancers present with advanced disease, the main goal of surgical intervention is often tumor debulking. The purpose of cytoreduction is to remove the primary cancer, as well as any metastases. If complete resection of metastases is not feasible, the aim is to reduce the bulk of the residual disease to, ideally, a diameter of 1 cm or less (so-called optimal debulking). Optimal cytoreduction enhances the response to intravenous (IV) paclitaxel and carboplatinum chemotherapy, and patients with minimal residual disease have a distinct survival advantage over those with more substantial residual disease. Furthermore, current data indicate that patients who have undergone optimal debulking may be candidates for intraperitoneal, as opposed to IV, chemotherapy. There is evidence to suggest that the use of intraperitoneal chemotherapy may improve survival rates.

A 47-year-old woman with achalasia is undergoing laparoscopic Heller myotomy. The esophagogastric junction has been exposed. Which is not a technical step in creating the myotomy itself? A) The esophagus is pulled downward and to the left B) The myotomy is started 3 cm below the esophagogastric junction C) The submucosal plane is entered at a single point D) Muscle fibers are separated from the mucosa for transection E) The myotomy is extended 2 cm distally and 6 cm proximally

To demonstrate understanding of the operative technique of laparoscopic Heller myotomy Answer B is correct After the fat pad has been removed to expose the esophagogastric junction, a Babcock clamp is applied over the junction, and the esophagus is pulled downward and to the left to expose the right side of the esophagus. The myotomy is performed at the 11 o'clock position. The myotomy is started 3 cm above the esophagogastric junction because at this point, the esophageal wall is usually normal. The proper submucosal plane should be reached at a single point to prevent mucosal perforation. Once the mucosa has been exposed, the myotomy is extended for about 2 cm onto the gastric wall distally and 6 cm proximally above the esophagogastric junction by separating muscle fibers from the mucosa for transection.

A 31-year-old female is referred to your clinic for evaluation of asymptomatic cholelithiasis. On examination, you notice central obesity, obvious facial hair, prominent fat overlying the upper back, fragile skin, and multiple ecchymoses over the upper extremities. She endorses recent weight gain, amenorrhea, mood swings, and depression. The next appropriate step in management is A) Laparoscopic cholecystectomy B) Combined laparoscopic cholecystectomy and adrenalectomy C) Random plasma cortisol D) 24-hour free urine cortisol level and dexamethasone suppression test E) CT of the abdomen and pelvis

To demonstrate understanding of the presentation and diagnosis of Cushing syndrome Answer D is correct Cushing syndrome may be caused by adrenocorticotropic hormone (ACTH)-secreting pituitary tumors, by ectopic ACTH-secreting tumors, or by cortisol-producing adrenal tumors. Of these conditions, ACTH-secreting tumors of the pituitary are the most common cause of Cushing syndrome, accounting for approximately 65 to 70% of cases. Ectopic production of ACTH accounts for between 10 and 15% of cases; small cell lung carcinomas are the most common ectopic ACTH-secreting tumors (50% of ectopic cases), and adrenal tumors account for about 20% of cases of Cushing syndrome. Cortisol-producing adenomas are the most common cause of adrenal Cushing syndrome, followed by adrenocortical carcinomas. A number of characteristic clinical symptoms and signs are associated with Cushing syndrome and include central obesity, prominent supraclavicular fat pads and fat over the upper back (so-called buffalo hump), skin striae, easy bruising, facial plethora, hirsutism in women, and testicular atrophy. The initial goal in the evaluation of a person who may have Cushing syndrome is to determine whether a state of increased cortisol production exists. If it does, subsequent testing should be undertaken (1) to determine whether the cause is ACTH dependent (a pituitary or ectopic ACTH-producing tumor) or ACTH independent (a primary adrenal lesion) and (2) to locate the source. When Cushing syndrome is suspected on clinical grounds, the initial diagnostic tests should be measurement of urinary free cortisol and an overnight dexamethasone suppression test. Once the diagnosis of Cushing syndrome is established, a plasma ACTH level should be obtained to differentiate ACTH-dependent causes of the syndrome (i.e., pituitary and ectopic ACTH-secreting tumors) from ACTH-independent causes (i.e., adrenal lesions). In patients with pituitary Cushing syndrome, ACTH levels are typically normal or only moderately elevated (> 20 pg/mL). In patients with ectopic ACTH-secreting tumors, ACTH levels are usually higher, although there is some overlap with the levels measured in patients with Cushing disease. In patients with Cushing syndrome caused by an adrenocortical tumor, plasma ACTH levels are suppressed (< 5 pg/mL). If it appears that the syndrome has an ACTH-dependent cause, the next step is to determine whether the cause is of pituitary or ectopic origin. Localization is performed once biochemical evaluation is complete; the next step is radiographic localization of the source of the Cushing syndrome. If adrenal Cushing syndrome is suspected, abdominal cross-sectional imaging with CT is recommended. If a pituitary source is suspected, magnetic resonance imaging of the pituitary should be the initial imaging test.

A 31-year-old female is referred to your clinic for evaluation of asymptomatic cholelithiasis. On examination, you notice central obesity, obvious facial hair, prominent fat overlying the upper back, fragile skin, and multiple ecchymoses over the upper extremities. She endorses recent weight gain, amenorrhea, mood swings, and depression. The next appropriate step in management is A) Laparoscopic cholecystectomy B) Combined laparoscopic cholecystectomy and adrenalectomy C) Random plasma cortisol D) 24-hour free urine cortisol level and dexamethasone suppression test E) CT of the abdomen and pelvis

To demonstrate understanding of the presentation and diagnosis of Cushing syndrome Answer D is correct Cushing syndrome may be caused by adrenocorticotropic hormone (ACTH)-secreting pituitary tumors, by ectopic ACTH-secreting tumors, or by cortisol-producing adrenal tumors. Of these conditions, ACTH-secreting tumors of the pituitary are the most common cause of Cushing syndrome, accounting for approximately 65 to 70% of cases. Ectopic production of ACTH accounts for between 10 and 15% of cases; small cell lung carcinomas are the most common ectopic ACTH-secreting tumors (50% of ectopic cases), and adrenal tumors account for about 20% of cases of Cushing syndrome. Cortisol-producing adenomas are the most common cause of adrenal Cushing syndrome, followed by adrenocortical carcinomas. A number of characteristic clinical symptoms and signs are associated with Cushing syndrome and include central obesity, prominent supraclavicular fat pads and fat over the upper back (so-called buffalo hump), skin striae, easy bruising, facial plethora, hirsutism in women, and testicular atrophy. The initial goal in the evaluation of a person who may have Cushing syndrome is to determine whether a state of increased cortisol production exists. If it does, subsequent testing should be undertaken (1) to determine whether the cause is ACTH dependent (a pituitary or ectopic ACTH-producing tumor) or ACTH independent (a primary adrenal lesion) and (2) to locate the source. When Cushing syndrome is suspected on clinical grounds, the initial diagnostic tests should be measurement of urinary free cortisol and an overnight dexamethasone suppression test. Once the diagnosis of Cushing syndrome is established, a plasma ACTH level should be obtained to differentiate ACTH-dependent causes of the syndrome (i.e., pituitary and ectopic ACTH-secreting tumors) from ACTH-independent causes (i.e., adrenal lesions). In patients with pituitary Cushing syndrome, ACTH levels are typically normal or only moderately elevated (> 20 pg/mL). In patients with ectopic ACTH-secreting tumors, ACTH levels are usually higher, although there is some overlap with the levels measured in patients with Cushing disease. In patients with Cushing syndrome caused by an adrenocortical tumor, plasma ACTH levels are suppressed (< 5 pg/mL). If it appears that the syndrome has an ACTH-dependent cause, the next step is to determine whether the cause is of pituitary or ectopic origin. Localization is performed once biochemical evaluation is complete; the next step is radiographic localization of the source of the Cushing syndrome. If adrenal Cushing syndrome is suspected, abdominal cross-sectional imaging with CT is recommended. If a pituitary source is suspected, MRI of the pituitary should be the initial imaging test.

A 31-year-old female is referred to your clinic for evaluation of a 1 cm breast mass. A core-needle biopsy was performed and is consistent with ductal carcinoma. Because of her strong family history, genetic testing was performed, and she is found to have a BRCA1 mutation. Which of the following statements regarding this case is false? A) This patient has a higher risk of developing a second breast cancer B) A reasonable management strategy would include mastectomy C) A reasonable management strategy would include bilateral mastectomy D) BRCA1 mutations are also associated with male breast cancer E) BRCA2 mutations are also associated with ovarian cancer

To demonstrate understanding of the presentation and management of breast cancer in patients with inherited susceptibility Answer D is correct Hereditary breast cancer is characterized by early age at onset, bilateral disease, and disease in other organ sites. Approximately 10% of all breast cancers are hereditary. Women with inherited susceptibility to breast cancer, most notably those with mutations in BRCA1 and BRCA2, are generally younger at breast cancer diagnosis and have a higher risk of developing a second cancer. This risk appears to be dependent on the age at which they are diagnosed with their first breast cancer: 25 to 50% for those under age 50 at the initial diagnosis and 15 to 20% for those over age 50. BRCA1 and BRCA2 account for the majority of hereditary breast cancers. The lifetime risk of breast cancer in these individuals can be as high as 85%. Mutations in these genes are associated with other malignancies, most notably ovarian, pancreatic, and prostate cancer. Specific founder mutations in BRCA1 and BRCA2 occur within certain ethnic groups (e.g., Ashkenazi Jews). Surgeons should also be aware of mutations in other genes that are commonly associated with an increased risk of breast cancer, including mutations in p53 (Li-Fraumeni syndrome), mutations in the PTEN gene (Cowden syndrome), and hereditary diffuse gastric cancer syndrome (CHD1). Tests that detect mutations in these genes are commercially available. Clinicians should consult a licensed genetic counselor to determine appropriate candidates for testing. When confronted with the choice of breast conservation therapy or mastectomy for small breast tumors, women who have inherited susceptibility with a mutation known in BRCA1 or BRCA2 also often choose mastectomy or bilateral mastectomy. The overall survival for women treated with breast conservation therapy in this situation is the same as for women without inherited susceptibility. However, they are at increased risk for developing a second primary cancer in the same breast, a chance that approaches 50%, especially in women under age 50 at the initial diagnosis. BRCA2, not BRCA1, mutations are associated with male breast cancer.

A 66-year-old female is referred to you for a 3 cm infiltrating ductal adenocarcinoma of the breast. On examination, she has a hard, fixed lymph node in the axilla. Which of the following statements regarding staging of the axilla and lymph node metastases in breast cancer is false? A) The risk of lymph node metastases is related to the size of the primary tumor B) In patients with clinically negative nodes, axillary lymph node dissection is at least equivalent to radiation for local control C) Axillary lymph node dissection is associated with significant long-term morbidity D) Inflammatory breast carcinoma is a contraindication to sentinel lymph node biopsy E) Previous excisional breast biopsy is a contraindication to sentinel lymph node biopsy

To demonstrate understanding of the rationale for sentinel lymph node biopsy in breast cancer Answer E is correct In early breast cancer, the pathologic status of the regional lymph nodes is the most important predictor of outcome. Axillary lymph node dissection in the setting of breast cancer is superior to observation and at least equivalent to irradiation for regional disease control in clinically node-negative patients. There is some evidence that adequate regional disease control may confer a small survival benefit. Invasive breast cancer has a relatively high rate of nodal metastasis in clinically node-negative patients. The risk of metastasis is clearly related to the size of the primary tumor, but it is significant (15% or higher) even in patients with early (T1a) lesions. The primary nodal drainage basin for the breast is the ipsilateral axilla. Axillary lymph node dissections are associated with a significant risk of long-term morbidity, primarily related to the risk of lymphedema in the affected arm. For this reason, sentinel node biopsy was developed and investigated as a possible substitute for standard axillary lymph node dissection in the treatment of breast cancer patients with clinically uninvolved axillary lymph nodes. All clinically node-negative patients with a diagnosis of invasive breast cancer are potential candidates for sentinel node biopsy. Ideal candidates are those patients with unifocal lesions less than 5 cm in greatest dimension who have no history of previous axillary surgery or previous cancer treatment. Performing a sentinel node biopsy after a previous excisional biopsy is technically feasible. Patients who have undergone extensive previous breast procedures (e.g., breast reduction, placement of breast implants, or multiple open biopsies) may have significant alterations in the lymphatic pathways; however, accurate identification of the sentinel node can still be accomplished with proper planning of the injection sites. For patients who will be undergoing mastectomy with immediate breast reconstruction, performing sentinel node biopsy as an outpatient procedure prior to the planned mastectomy may facilitate surgical planning depending on the need for postmastectomy radiation or axillary lymph node dissection. However, the false negative rate of sentinel node biopsy may be higher in cases with large additive tumor burden. For patients who present with inflammatory breast carcinoma or who present with palpable adenopathy and pathologically confirmed metastatic nodal disease, there is absolutely no role for sentinel node biopsy at the time of presentation.

A 54-year-old female with a 7 cm right adnexal mass is referred to you. A percutaneous biopsy reveals ovarian cancer. Which of the following statements regarding her diagnosis is false? A) Approximately 5 to 10% of ovarian cancers have a hereditary component.. B) The majority of patients with ovarian cancer have localized disease (stage I or II) disease. C) The CA 125 antigen is not a useful screening test in patients with ovarian cancer. D) The staging of ovarian cancer is based on the pathologic findings at surgery. E) Patients with advanced disease of the peritoneal cavity at operation should undergo surgical debulking.

To demonstrate understanding of the staging and treatment of ovarian cancer Answer B is correct Approximately 5 to 10% of these cancers have a hereditary component; women with BRCA1/BRCA2 or Lynch syndrome (hereditary nonpolyposis colorectal cancer [HNPCC]) germline mutations are at increased risk. Ovarian epithelial carcinomas spread primarily through exfoliation of cells, which places the entire peritoneal cavity at risk for metastases. Moreover, early recognition is difficult because the majority of patients have only vague and nonspecific complaints. Not surprisingly, 60 to 70% of patients have advanced (stage III or IV) disease at diagnosis. Detection of ovarian cancer is further hindered by the fact that there is no effective screening test. The tumor-associated antigen CA 125 is expressed by more than 80% of ovarian epithelial cancers; however, although CA 125 levels are employed in following up patients with documented disease, they are not useful for screening purposes. Surgical intervention serves three purposes in the setting of ovarian epithelial cancer: diagnosis, staging, and tumor debulking (cytoreduction). The staging of ovarian cancer is based on the pathologic findings at surgery. Only a minority of women have stage I disease at diagnosis, and the decision whether to employ paclitaxel and carboplatinum chemotherapy is predicated on the surgical extent of the disease. In patients who appear to have early rather than advanced disease, the emphasis is on accurate staging. Because the majority of patients with ovarian epithelial cancers present with advanced disease, the main goal of surgical intervention is often tumor debulking. The purpose of cytoreduction is to remove the primary cancer and any metastases. Optimal cytoreduction enhances the response to IV paclitaxel and carboplatinum chemotherapy, and patients with minimal residual disease have a distinct survival advantage over those with more substantial residual disease.

You are planning a modified radical mastectomy for a 57-year-old female with infiltrating ductal carcinoma of the breast. During axillary lymph node dissection, all the following must be preserved except for which structure? A) Intercostobrachial nerve B) Medial pectoral nerve C) Thoracodorsal nerve D) Long thoracic nerve E) Thoracodorsal artery

To demonstrate understanding of the technique of axillary lymph node dissection Answer A is correct Axillary dissection includes resection of level I and level II lymph nodes The superior border of the dissection is formed by the axillary vein; the lateral border of the dissection is formed by the latissimus dorsi; the medial border is formed by the pectoral muscles and the anterior serratus muscle; and the inferior border is formed by the tail of the breast. Level II nodes are easily removed by retracting the pectoralis major and the pectoralis minor medially; it is not necessary to divide or remove the pectoralis minor. Level III nodes are not removed unless palpable disease is present. A number of vascular structures and nerves passing through the axilla must be preserved during axillary dissection. These structures include the axillary vein and artery; the brachial plexus; the long thoracic nerve, which innervates the anterior serratus muscle; the thoracodorsal nerve, artery, and vein, which supply the latissimus dorsi; and the medial pectoral nerve, which innervates the lateral portion of the pectoralis major. The axillary artery and the brachial plexus should not be exposed during axillary dissection. If they are, the dissection has been carried too far superiorly, and proper orientation at a more inferior position should be established. Medial to the thoracodorsal nerve and adherent to the chest wall is the long thoracic nerve of Bell. The medial pectoral nerve runs from superior to the axillary vein to the undersurface of the pectoralis major, passing through the axillary fat pad and across the level II nodes; it has an accompanying vein whose blue color may be used to identify the nerve. The intercostobrachial nerve provides sensation to the posterior portion of the upper arm. Sacrificing this nerve generally leads to numbness over the triceps region. In many women, the intercostobrachial nerve measures 2 mm in diameter and takes a fairly cephalad course near the axillary vein; when this is the case, preservation of the nerve will not interfere with node dissection. Sometimes, however, the nerve is tiny, has multiple branches, and is intermingled with nodal tissue that should be removed; when this is the case, one should not expend a great deal of time on attempting to preserve the nerve. If the intercostobrachial nerve is sacrificed, it should be transected with a knife or scissors rather than with electrocautery, and the ends should be buried to reduce the likelihood of postoperative causalgia.

A 47-year-old female is found on physical examination to have a small lump in the upper outer quadrant of the left breast. Ultrasonography confirmed its solid, noncystic nature, and a CNB demonstrated infiltrating ductal adenocarcinoma. Which of the following statements concerning the surgical management of this patient is true? A) The addition of radiation to lumpectomy does not increase long-term survival B) At least a 2 mm margin is necessary on lumpectomy C) Previous chest radiation is not a contraindication for breast conservation therapy D) Breast conservation therapy is not possible for pregnant patients E) If a negative margin cannot be obtained with lumpectomy, mastectomy is necessary

To demonstrate understanding of the treatment of small breast tumors (T1/T2) Answer A is correct The National Surgical Adjuvant Breast and Bowel Project (NSABP) B-06 trial compared mastectomy with lumpectomy alone and with lumpectomy plus radiation therapy. Long-term follow-up on this trial demonstrated no difference in overall survival between the three groups. However, with lumpectomy alone, the cancer recurred in the same breast in as many as 40% of women. Radiation reduced this rate of local recurrence to approximately 12%. On the basis of this finding, lumpectomy plus radiation therapy became an accepted standard in the late 1980s and became widely used through the 1990s. Indications for mastectomy include the presence of multicentric disease in the breast—separate cancers in distinct parts of the breast separated by at least 5 cm of normal breast tissue, occurring in 2 to 5% of cases; a very large tumor in relation to the size of the breast that is not amenable to lumpectomy or does not respond sufficiently to neoadjuvant systemic therapy; and previous radiation treatments to the breast, such as with a prior breast cancer or for treatment of other cancers, such as lymphoma. Radiation may not be administered during pregnancy, but with standard sequencing of systemic chemotherapy and radiation after breast conservation therapy, radiation is usually timed such that it is administered after delivery of the baby; therefore, breast conservation therapy is possible for women presenting with breast cancer during pregnancy. In addition, mastectomy is indicated after completion of systemic chemotherapy for inflammatory breast cancer because of the likelihood of the microscopic extent of disease both in the skin and within the breast beyond the area of known cancer.

A 61-year-old female has undergone lumpectomy and radiation for a 2 cm moderately differentiated invasive ductal carcinoma. A sentinel lymph node biopsy was negative. The tumor is estrogen receptor positive, progesterone receptor positive, and HER-2/neu negative. Which of the following statements regarding systemic chemotherapy of breast cancer is false? A) Women with negative lymph nodes do not require chemotherapy B) Tamoxifen therapy is effective only for women whose tumors express hormone receptors C) Aromatase inhibitors are superior to tamoxifen in preventing the recurrence of breast cancer in postmenopausal women D) With only a few exceptions, all women with hormone receptor-positive breast cancers should receive prolonged endocrine therapy E) Breast cancers that express the HER-2/neu protein have a higher risk of developing distant metastases

To demonstrate understanding of the use of endocrine chemotherapy in the treatment of breast cancer Answer A is correct Every woman with invasive cancer must consider whether to receive systemic adjuvant therapy to prevent the growth of this microscopic metastatic disease. It must be recognized that a substantial fraction of women with negative lymph nodes still develop metastatic disease. Treatment entails two major classes of drugs: endocrine drugs, which block the effect of circulating estrogen on breast cancers that express estrogen and progesterone receptors, and systemic cytotoxic chemotherapy. Endocrine therapy is effective only for women whose tumors express hormone receptors. This includes premenopausal women and postmenopausal women as they have low levels of circulating estrogens from nonovarian sources. There are two primary classes of endocrine drugs: selective endocrine receptor modulators (SERMs), of which the primary agent used in breast cancer is tamoxifen, and aromatase inhibitors, which block peripheral synthesis of estrogen and are effective only in postmenopausal women. Recent clinical trials demonstrated that aromatase inhibitors are superior to tamoxifen in preventing the recurrence of breast cancer, with a slightly improved safety profile in postmenopausal women. Aromatase inhibitors are not effective in premenopausal women, for whom tamoxifen remains the drug of choice. With only a few exceptions, all women with hormone receptor-positive breast cancers should receive prolonged endocrine therapy. At least 5 years of therapy is warranted. Women with hormone receptor-positive breast cancers should consider whether to also receive systemic chemotherapy. A large body of evidence from clinical trials shows that survival may be improved by the addition of chemotherapy. Endocrine therapy is not effective in women with hormone receptor-negative breast cancer. For these women, the only treatment to reduce the risk of distant metastases is cytotoxic chemotherapy. Unless otherwise contraindicated by serious comorbidities, women with cancers over 1 cm and negative hormone receptors should receive chemotherapy. Those with tumors under 1 cm have a lower risk of distant metastases and may derive substantially less benefit from chemotherapy. Breast cancers that express the HER-2/neu protein have a higher risk of developing distant metastases. However, the monoclonal antibody trastuzumab (Herceptin) directed against HER-2 has proven to be safe and effective when given in combination with chemotherapy.

A 42-year-old woman with no significant medical history is accompanying her mother to your office for her postoperative visit following mastectomy for breast cancer. She begins to ask you about her risks of breast malignancy and screening guidelines. Which of the following is false? A) Breast Imaging Reporting and Data System (BI-RADS) score of 4 or greater requires biopsy B) Concerning findings on mammography include "clustered" calcifications C) The majority of breast cancers are detected by screening mammography D) Most organizations recommend annual screening mammography at age 50 E) Mammography has a relatively low specificity, with about 60 to 70% of all lesions biopsied proving benign

To demonstrate understanding of the use of screening mammography Answer D is correct The majority of breast cancers are detected by screening mammography. Controversy continues over the exact populations that benefit from screening mammography, but most organizations recommend mammography annually beginning at age 40 and at a younger age for women at substantial risk for increased risk of cancer. Concerning findings include new masses, asymmetry, distortion in the architecture of the breast tissue, and the presence of calcifications grouped ("clustered") in an area of breast tissue. Any finding should be compared with previous mammograms to determine if it is new. Mammogram findings are assigned a score of 0 to 5 using the BI-RADS, with BI-RADS 4 and 5 findings requiring biopsy. Mammography detects only 80 to 90% of breast cancers and is less sensitive in women with dense glandular tissue (more common in younger women) and for lobular cancer. Suspicious physical findings (e.g., a lump or discharge) should be evaluated and biopsied even if breast imaging is normal. Mammography also has a relatively low specificity, with about 60 to 70% of all lesions requiring biopsy proving benign.

A 53-year-old woman presents with a chest wall mass. Biopsy confirms a chest wall sarcoma. Which of the following is the strongest predictor of disease-free survival after resection? A) Patient age B) Preoperative symptoms C) Chest wall size D) Histologic grade of the tumor E) Patient gender

To learn the predictors of outcome of chest wall sarcomas after resection Answer D is correct Sarcomas are the most common primary malignancy of the chest wall. With wide surgical resection, disease-free survival can be achieved in more than 50% of patients even after 10 years. The histologic grade of the tumor is an independent prognostic factor for disease-free survival. The type of surgical resection is likewise predictable. Conversely, age, gender, symptom, and size of the sarcoma do not significantly impact survival.

A patient whose father had colon cancer comes to your office concerned about whether he should undergo a screening colonoscopy. According to screening guidelines, when and how often should patients at high risk for developing colorectal cancer (CRC) (e.g., those with a family history of CRC) undergo colonoscopy? A) At age 50 or 5 years younger than the age of the affected first-degree relative at 5-year intervals B) At age 50 or 15 years younger than the age of the affected first-degree relative at 10-year intervals C) At age 40 or 5 years younger than the age of the affected first-degree relative at 10-year intervals D) At age 40 or 10 years younger than the age of the affected first-degree relative at 5-year intervals E) At age 40 or 5 years younger than the age of the affected first-degree relative at 10-year intervals

To describe the screening guidelines for patients at high risk for developing colorectal cancer (CRC) Answer D is correct Many groups have advocated CRC screening, and published guidelines are available from several organizations, including a collaborative guideline published by the American Cancer Society, the U.S. Preventive Services Task Force, and the American College of Radiology. Others include the American College of Gastroenterology and the National Comprehensive Cancer Network (NCCN). All of these guidelines recommend that screening begin at age 50 for average-risk patients, with colonoscopy at 10-year intervals. The recommended screening options in patients unwilling or unable to undergo colonoscopy are fairly consistent among the various organizations and include (1) fecal occult blood testing (FOBT) yearly, (2) flexible sigmoidoscopy every 5 years, (3) yearly FOBT and flexible sigmoidoscopy every 5 years, (4) double-contrast barium enema every 5 years, and (5) CT colonography every 5 years. In high-risk patients (e.g., those with a family history of CRC), screening may begin at an earlier age—generally at age 40 or 10 years younger than the age of the affected first-degree relative and with shorter, 5-year intervals. There are also specific intensive screening and follow-up regimens for patients with known or suspected familial cancer syndromes.

A 57-year-old man with a history of diabetes mellitus and two previous attacks of diverticulitis requiring hospitalization presents to the emergency department with left lower quadrant pain, diarrhea, and a temperature of 37°C (100.3°F). On physical examination, he is tender to palpation in the left lower quadrant with mild rebound tenderness and guarding. Computed tomography (CT) is performed and reveals multiple sigmoid diverticula with bowel wall thickening and inflammation of the adjacent pericolic fat. What distinguishes a diagnosis of complicated diverticulitis from that of simple diverticulitis? A) The previous number of attacks of diverticulitis a patient has experienced B) The patient's comorbid conditions (i.e., degree of organ failure, immunosuppression, age) C) Diverticulitis involving the cecum D) The presence or absence of obstruction, abscess, fistula, or free perforation E) All of the above

To distinguish simple diverticulitis from complicated diverticulitis Answer D is correct Some cases of diverticulitis are classified as complicated, meaning that the disease process has progressed to obstruction, abscess or fistula formation, or free perforation. Complicated diverticulitis may be particularly challenging to manage, especially because patients may have no known history of diverticular disease. Lower gastrointestinal bleeding is also a complication of diverticular disease in 30 to 50% of cases; in fact, diverticula are the most common colonic cause of lower gastrointestinal bleeding.

A 63-year-old man undergoes a low anterior resection for a rectal cancer 8 cm from the anal verge. In patients undergoing surgical treatment of rectal cancer, which of the following is not considered an adequate resection margin? A) Radial margin less than 2 mm B) Histologically negative margin in patients receiving adjuvant chemoradiation therapy C) Distal margin less than 5 cm D) Distal margin of 2 cm E) Radial margin of 5 mm

To identify adequate resection margins in rectal cancer and understand the importance of an adequate radial margin Answer A is correct There has been a great deal of debate about what constitutes an adequate margin of resection in surgical treatment of rectal cancer. With respect to distal margins, 2 to 5 cm has traditionally been considered to be the minimum necessary for curative resection. Growing interest in sphincter preservation has led investigators to consider smaller distal margins (i.e., < 2 cm). Studies have shown that clear margins smaller than 2 cm are not associated with higher local recurrence rates or reduced survival. Subsequent reports have suggested that even smaller histologically negative margins (i.e., < 1 cm) may be adequate in patients receiving adjuvant chemoradiation therapy. The importance of radial margin involvement after rectal cancer resection was not recognized until comparatively recently. Radial margins are assessed by means of serial slicing and evaluation of multiple coronal sections of the tumor and the mesorectum. Involvement of radial margins is a predictor of both local recurrence and survival after potentially curative rectal cancer surgery and may be associated with an increased risk of distant metastases. Radial margins smaller than 2 mm are associated with increased local recurrence rates. Adjuvant radiation therapy does not compensate for the adverse impact of positive margins on local recurrence rates.

A 48-year-old man presents with a single 3 cm metastatic lesion in the left lateral segment of the liver 4 years after right colectomy for colon cancer. In patients with isolated liver metastases from CRC, what is the approximate 5-year survival rate after resection? A) 5% B) 15% C) 20% D) 30% E) 50%

To identify approximate 5-year survival after hepatic metasectomy in patients with colorectal cancer (CRC) Answer D is correct Numerous studies have addressed the treatment of patients with isolated liver metastases from CRC. Resection of isolated hepatic metastases has been reported to yield 5-year survival rates higher than 30%, with acceptable surgical morbidity and mortality. Investigators from the Memorial Sloan-Kettering Cancer Center developed a staging system known as the clinical score in an attempt to predict which patients are likely to benefit from aggressive surgical resection. This system used five factors that were found to be independent predictors of poor outcome: (1) node-positive primary disease, (2) a disease-free interval shorter than 12 months, (3) the presence of more than one hepatic tumor, (4) a maximum hepatic tumor size exceeding 5 cm, and (5) a carcinoembryonic antigen level higher than 200 ng/mL. Patients who met no more than two of these criteria generally had good outcomes, whereas those who met three or more were recommended for inclusion in adjuvant therapy trials.

The barium swallow of a patient with dysphagia shows an area of obstruction. Which of the following is not a potential cause of this radiographic finding? A) Esophageal ring B) Esophageal web C) Peptic stricture D) Achalasia E) Esophageal cancer

To recognize etiologies of mechanical esophageal obstruction Answer D is correct A patient with a fixed mechanical obstruction on barium swallow may have an esophageal web or ring, sequelae of gastroesophageal reflux disease (Barrett esophagus or peptic stricture), or esophageal cancer. On barium enema, achalasia is demonstrated by narrowing at the esophagogastric junction; however, achalasia is not a mechanical obstruction of the esophagus but rather is a motor disorder.

A patient presents to your office with a cancerous rectal polyp and is interested in transanal excision. Which of the following criteria does not preclude a transanal excision of a rectal cancer? A) Tumor greater than 3 cm in diameter B) Tumor encompassing greater than 30% circumference of the rectum C) Tumor greater than 8 cm from the anal verge D) Tumor invading the submucosa E) Regional lymph node involvement

To identify indications and contraindications to local excision of rectal cancer Answer D is correct T1 disease is, by definition, tumor invading the submucosa without evidence of regional lymph node involvement or metastatic disease. T1 disease is not a contraindication to transanal excision of a rectal cancer. Several criteria have been established to identify patients who may be candidates for transanal excision. According to guidelines published by the NCCN, the lesion must be no more than 3 cm in diameter, must encompass no more than 30% of the circumference of the rectum, and must be less than 8 cm from the anal verge. With the advent of transanal endoscopic microsurgery (TEMS), these criteria have been expanded to include patients with more proximal lesions. Poorly differentiated tumors and the presence of lymphovascular invasion may also be associated with increased nodal involvement and higher recurrence rates.

A 52-year-old man presents with a hard, raised lesion adjacent to the anus. Biopsy reveals squamous cell carcinoma. Which of the following is the most appropriate management plan for a patient with squamous cell carcinoma of the anal margin? A) Wide local excision B) Low anterior resection C) End colostomy D) APR E) Mucosal stripping

To identify the appropriate initial management strategy for management of squamous cell carcinoma of the anal margin Answer A is correct Squamous cell curcinoma of the anal margin acts in a manner similar to that of lesions occurring in other cutaneous areas of the body. The lesions appear as raised, hard, flat masses that may ulcerate. The appropriate therapy is wide local excision with clear margins.

A 42-year-old man presents with an anal lesion. Biopsy of this shows squamous cell carcinoma. In a patient with squamous cell carcinoma of the anal canal, what is the most appropriate initial form of treatment? A) Low anterior resection B) Abdominoperineal resection (APR) C) 5-Fluorouracil (5-FU), leucovorin, and oxaliplatin D) 5-FU, mitomycin C, and radiotherapy E) Local excision

To identify the initial treatment modality in cancer of the anal canal Answer D is correct Inadequate results from surgery or radiotherapy alone led Nigro at Wayne State University to propose initial chemotherapy (5-FU, mitomycin C) and radiotherapy followed by APR. Subsequent experience demonstrated that chemoradiotherapy was curative in most patients, and the subsequent APR has been abandoned unless the tumor persists or recurs. The reported experience with multimodality treatment for anal carcinoma continues to expand. The radiotherapy entails 30 to 60 Gy (given over 3 to 5 weeks) using apposed fields. Chemotherapy is given at the same time as the radiotherapy according to the following scheme: 5-FU (1,000 mg/m2/day) on days 1 to 5 and days 31 to 35 and mitomycin C (15 mg/m2) on day 1.

A 42-year-old man presents with an anal lesion. Biopsy of this shows squamous cell carcinoma. In a patient with squamous cell carcinoma of the anal canal, what is the most appropriate initial form of treatment? A) Low anterior resection B) Abdominoperineal resection (APR) C) 5-Fluorouracil (5-FU), leucovorin, and oxaliplatin D) 5-FU, mitomycin C, and radiotherapy E) Local excision

To identify the initial treatment modality in cancer of the anal canal Answer D is correct Inadequate results from surgery or radiotherapy alone led Nigro at Wayne State University to propose initial chemotherapy (5-FU, mitomycin C) and radiotherapy followed by APR. Subsequent experience demonstrated that chemoradiotherapy was curative in most patients, and the subsequent APR has been abandoned unless the tumor persists or recurs. The reported experience with multimodality treatment for anal carcinoma continues to expand. The radiotherapy entails 30 to 60 Gy (given over 3 to 5 weeks) using apposed fields. Chemotherapy is given at the same time as the radiotherapy according to the following scheme: 5-FU (1,000 mg/m2/day) on days 1 to 5 and days 31 to 35 and mitomycin C (15 mg/m2) on day 1.

Following completion of chemotherapy and radiation therapy for squamous cell carcinoma of the anus, a 54-year-old man has a persistent anal mass. Biopsy of this confirms squamous cell carcinoma. In patients who have received the Nigro protocol but have residual squamous cell carcinoma of the anal canal, additional treatment options include which of the following? A) Additional chemoradiotherapy B) Local excision C) APR D) All of the above E) None of the above

To identify treatment options for residual squamous cell carcinoma of the anal canal following treatment with the Nigro protocol Answer D is correct Inadequate results from surgery or radiotherapy alone led Nigro at Wayne State University to propose initial chemotherapy (5-FU, mitomycin C) and radiotherapy followed by APR. Subsequent experience demonstrated that chemoradiotherapy was curative in most patients, and the subsequent APR has been abandoned unless the tumor persists or recurs. The reported experience with multimodality treatment for anal carcinoma continues to expand. The radiotherapy entails 30 to 60 Gy (given over 3 to 5 weeks) using apposed fields. Chemotherapy is given at the same time as the radiotherapy according to the following scheme: 5-FU (1,000 mg/m2/day) on days 1 to 5 and days 31 to 35 and mitomycin C (15 mg/m2) on day 1. Using an anoscope, the lesion site is inspected 4 to 6 weeks after the radiotherapy is completed, and a biopsy is performed on any abnormalities. Some providers will allow additional time after therapy to confirm complete clinical regression of the tumor before embarking on additional therapy. An APR, local excision, or additional chemoradiotherapy is offered to patients with residual disease following combined therapy.

A 73-year-old woman with known lung cancer not amenable to resection, who is currently undergoing chemotherapy, presents to the emergency department with hemoptysis. What is the most important step in the appropriate management of this patient? A) Admit the patient to the hospital B) Obtain a chest x-ray C) Administer antibiotics D) Check prothrombin time and partial thromboplastin time E) Perform bronchoscopy

To learn the proper management of a patient with hemoptysis Answer E is correct Hemoptysis may be a harbinger of life-threatening illness and should be taken seriously in all circumstances. Any patient with significant hemoptysis should be admitted to a hospital and evaluated promptly. Routine chest x-rays are insensitive and will likely not assist in management. Infection is commonly associated with hemoptysis, and one should consider administering antibiotics. Coagulopathy as an underlying cause or a contributor to hemoptysis should be considered and investigated. Bronchoscopy is frequently able to define the pathology causing hemoptysis and is the key to diagnosis; it is therefore the most important step in the management of a patient with hemoptysis.

Following completion of chemotherapy and radiation therapy for squamous cell carcinoma of the anus, a 54-year-old man has a persistent anal mass. Biopsy of this confirms squamous cell carcinoma. In patients who have received the Nigro protocol but have residual squamous cell carcinoma of the anal canal, additional treatment options include which of the following? A) Additional chemoradiotherapy B) Local excision C) APR D) All of the above E) None of the above

To identify treatment options for residual squamous cell carcinoma of the anal canal following treatment with the Nigro protocol Answer D is correct Inadequate results from surgery or radiotherapy alone led Nigro at Wayne State University to propose initial chemotherapy (5-FU, mitomycin C) and radiotherapy followed by APR. Subsequent experience demonstrated that chemoradiotherapy was curative in most patients, and the subsequent APR has been abandoned unless the tumor persists or recurs. The reported experience with multimodality treatment for anal carcinoma continues to expand. The radiotherapy entails 30 to 60 Gy (given over 3 to 5 weeks) using apposed fields. Chemotherapy is given at the same time as the radiotherapy according to the following scheme: 5-FU (1,000 mg/m2/day) on days 1 to 5 and days 31 to 35 and mitomycin C (15 mg/m2) on day 1. Using an anoscope, the lesion site is inspected 4 to 6 weeks after the radiotherapy is completed, and a biopsy is performed on any abnormalities. Some providers will allow additional time after therapy to confirm complete clinical regression of the tumor before embarking on additional therapy. An APR, local excision, or additional chemoradiotherapy is offered to patients with residual disease following combined therapy.

A 79-year-old man undergoes a chest x-ray because of a persistent cough. The chest x-ray shows a solitary pulmonary nodule, and this is confirmed on a chest computed tomographic (CT) scan. Which of the following features is most predictive of malignancy for a solitary pulmonary nodule? A) Age over 70 years B) Lesion size 2.1 to 3 cm C) Doubling time less than 465 days D) History of smoking E) Spiculated margins on CT scan

To know the features most strongly associated with malignancy of a solitary pulmonary nodule Answer E is correct There are several potentially significant indicators of malignant disease for a solitary pulmonary nodule. The top five factors affecting malignant probability of a solitary pulmonary nodule, in descending order of significance, are spuculated margins on a CT scan, age over 70 years, lesion size 2.1 to 3 cm, doubling time less than 465 days, and a history of smoking.

A 55-year-old woman presents with a solitary 4 cm right thyroid nodule. She has no palpable cervical lymph nodes. Fine-needle aspiration reveals a medullary carcinoma. What is the recommended treatment for this patient? A) Right thyroid lobectomy B) Total thyroidectomy with central neck dissection C) Total thyroidectomy D) Total thyroidectomy with central neck dissection and ipsilateral functional neck dissection E) Right thyroid lobectomy with ipsilateral functional neck dissection

To learn about the indications and extent of neck dissections in patients with thyroid cancers Answer D is correct Lymph nodes in the central compartment of the neck (i.e., between two carotid sheaths) are frequently involved in patients with papillary, medullary, and Hürthle cell cancers. Prophylactic or elective neck dissections are generally not indicated in thyroid cancers. In patients with papillary or Hürthle cell carcinoma, ipsilateral modified neck dissections in addition to central neck dissections are indicated if the lymph nodes are palpable. However, in patients with MTC, a meticulous and thorough central neck dissection is mandatory even if there are no palpable lymph nodes. When a primary medullary tumor is larger than 1 cm in size or the central neck nodes are obviously involved, an ipsilateral functional neck dissection (with preservation of the spinal accessory nerve, internal jugular vein, and sternocleidomastoid muscle), along with central neck dissection, would be beneficial and is therefore recommended.

A 55-year-old woman presents with a solitary 4 cm right thyroid nodule. She has no palpable cervical lymph nodes. Fine-needle aspiration (FNA) reveals a medullary carcinoma. What is the recommended treatment for this patient? A) Right thyroid lobectomy B) Total thyroidectomy with central neck dissection C) Total thyroidectomy D) Total thyroidectomy with central neck dissection and ipsilateral functional neck dissection E) Right thyroid lobectomy with ipsilateral functional neck dissection

To learn about the indications and extent of neck dissections in patients with thyroid cancers Answer D is correct Lymph nodes in the central compartment of the neck (i.e., between two carotid sheaths) are frequently involved in patients with papillary, medullary, and Hürthle cell cancers. Prophylactic or elective neck dissections are generally not indicated in thyroid cancers. In patients with papillary or Hürthle cell carcinoma, ipsilateral modified neck dissections in addition to central neck dissections are indicated if the lymph nodes are palpable. However, in patients with medullary thyroid cancer, a meticulous and thorough central neck dissection is mandatory even if there are no palpable lymph nodes. When a primary medullary tumor is larger than 1 cm in size or the central neck nodes are obviously involved, an ipsilateral functional neck dissection (with preservation of the spinal accessory nerve, internal jugular vein, and sternocleidomastoid muscle), along with central neck dissection, would be beneficial and is thereforeTo learn about the indications and extent of neck dissections in patients with thyroid cancers

A primary care physician refers a patient to you after detecting a thyroid mass on an annual physical examination. A thyroid mass is more likely to be malignant in all of the following demographic groups except: A) Female gender B) Elderly patients C) Pregnancy D) History of radiation exposure E) Age less than 10 years

To learn risk factors for malignancy in a thyroid mass Answer A is correct Thyroid disease is a relatively common cause of neck masses: in the United States, about 4% of women and 2% of men have a palpable thyroid nodule. Patients should be questioned about local symptoms (pain, dysphagia, pressure, hoarseness, or a change in the voice), about the duration of the nodule, and about systemic symptoms (from hyperthyroidism, hypothyroidism, or any other illness). Although most nodules are benign, malignancy is a significant concern. Nodules in children, young men, older persons, pregnant women, or persons with a history of radiation exposure or a family history of thyroid cancer are more likely to be malignant. Nodules that are truly solitary, feel firm or hard on examination, are growing rapidly, or are nonfunctional on scans are more likely to be malignant.

A primary care physician refers a patient to you after detecting a thyroid mass on an annual physical examination. A thyroid mass is more likely to be malignant in all of the following demographic groups except A) Females B) Elderly patients C) Pregnant women D) Persons with a history of radiation exposure E) Persons less than 10 years of age

To learn the risk factors for malignancy in a thyroid mass Answer A is correct Thyroid disease is a relatively common cause of neck masses: in the United States, about 4% of women and 2% of men have a palpable thyroid nodule. Patients should be questioned about local symptoms (pain, dysphagia, pressure, hoarseness, or a change in the voice), about the duration of the nodule, and about systemic symptoms (from hyperthyroidism, hypothyroidism, or any other illness). Although most nodules are benign, malignancy is a significant concern. Nodules in children, young men, older persons, pregnant women, or persons with a history of radiation exposure or a family history of thyroid cancer are more likely to be malignant. Nodules that are truly solitary, feel firm or hard on examination, are growing rapidly, or are nonfunctional on scans are more likely to be malignant.

The morning after an open aortic aneurysm repair, a 54-year-old male is found to have three toes with acral cyanosis. Which of the following is true of this disorder? A) Absence of distal pulses B) Cold limb C) Acute onset of severe pain D) Macroembolization of cholesterol-laden debris E) Digital rubor

To recognize " blue toe syndrome" and the associated symptoms and etiology. Answer C is correct. Atheroembolism is a condition in which microscopic cholesterol-laden debris travels from proximal arteries until it reaches the most distal arterial segments, typically in the skin of the lower extremities and in the end-organs. This debris usually originates from unstable plaque found at inflection points in the arterial tree, especially in the aorta. It may also originate from aneurysmal sacs either in the aorta or in the peripheral arteries. Patients with atheroembolism usually present with focal toe ischemia—the so-called blue toe syndrome—in conjunction with palpable pulses in the distal extremity. Acute pain of sudden onset is typically noted in the affected area. This pain can often establish the exact timing of embolization. Cyanosis is present either on the toe or over a more extensive area if the atheroemboli were circulated throughout the extremity. When both lower extremities are involved, an aortic source of the microemboli is commonly found. A complete vascular examination should be performed and pulses documented. Although a patent arterial tree is the rule, emboli that are sufficiently small may travel through collateral channels. Palpation should be done to detect any aneurysmal disease. A massive proximal atheroembolic event may affect the entire abdominal wall and both extremities, giving the appearance of livedo reticularis. As the source of the atheroemboli ascends in the arterial tree, more vital organs (e.g., the kidneys and the gastrointestinal tract) may be damaged. Manipulation of an intra-arterial catheter or clamping or surgical manipulation of the arterial tree can also result in plaque disruption. In these cases, the adverse effects are usually apparent immediately after the procedure.

The morning after an open aortic aneurysm repair, a 54-year-old male is found to have three toes with acral cyanosis. Which of the following is true of this disorder? A) Absence of distal pulses B) Cold limb C) Acute onset of severe pain D) Macroembolization of cholesterol-laden debris E) Digital rubor

To recognize "blue toe syndrome" and the associated symptoms and etiology Answer C is correct. Atheroembolism is a condition in which microscopic cholesterol-laden debris travels from proximal arteries until it reaches the most distal arterial segments, typically in the skin of the lower extremities and in the end-organs. This debris usually originates from unstable plaque found at inflection points in the arterial tree, especially in the aorta. It may also originate from aneurysmal sacs either in the aorta or in the peripheral arteries. Patients with atheroembolism usually present with focal toe ischemia—the so-called blue toe syndrome—in conjunction with palpable pulses in the distal extremity. Acute pain of sudden onset is typically noted in the affected area. This pain can often establish the exact timing of embolization. Cyanosis is present either on the toe or over a more extensive area if the atheroemboli were circulated throughout the extremity. When both lower extremities are involved, an aortic source of the microemboli is commonly found. A complete vascular examination should be performed and pulses documented. Although a patent arterial tree is the rule, emboli that are sufficiently small may travel through collateral channels. Palpation should be done to detect any aneurysmal disease. A massive proximal atheroembolic event may affect the entire abdominal wall and both extremities, giving the appearance of livedo reticularis. As the source of the atheroemboli ascends in the arterial tree, more vital organs (e.g., the kidneys and the GI tract) may be damaged. Manipulation of an intra-arterial catheter or clamping or surgical manipulation of the arterial tree can also result in plaque disruption. In these cases, the adverse effects are usually apparent immediately after the procedure.

A 72-year-old female with coronary artery disease, severe heart failure, hypertension, and peripheral arterial disease presents with abdominal pain to the emergency department. The history and physical examination reveal a cachectic woman with evidence of an upper respiratory infection and dry mucous membranes. Although her abdomen is soft and nondistended, she complains of constant, severe abdominal pain, which began several hours prior to presentation. A diagnosis of nonocclusive mesenteric ischemia (NOMI) is made following a diagnostic angiogram. Which of the following statements regarding NOMI is false? A) Management of NOMI largely consists of medical optimization. B) Despite the etiology, laparotomy is required when peritonitis is present. C) Angiography typically reveals absence of SMA filling. D) Vasopressors should be avoided during the resuscitative period. E) Intra-arterial infusion of papavarine may be of benefit

To recognize NOMI and appropriate management Answer C is correct. Patients with NOMI typically exhibit angiographic evidence of SMA vasospasm. A small SMA trunk is visualized, with very few branching vessels visible, and the branches that are visualized show a characteristic tapering of the vessel to the point of occlusion. These patterns are best seen on the anteroposterior projection. Management of NOMI is largely nonoperative. Once the diagnosis has been established with angiography, treatment of the underlying precipitating cause is the key therapeutic intervention. Optimization of fluid resuscitation, improvement in cardiac output, and elimination of vasopressors are the measures that have the greatest impact on outcome. Selective catheterization of the SMA with direct intra-arterial infusion of papaverine (30 to 60 mg/hr) may be employed as adjunctive therapy. The infusion is continued for at least 24 hours, with repeat angiography performed at regular intervals to determine the effectiveness of this therapy. Alternatively, some authors described using direct intra-arterial administration of tolazoline and glycerol trinitrite as local vasodilators in patients with NOMI with good clinical success. If the patient presents with peritoneal signs on physical examination, an exploratory laparotomy will be required for resection of frankly necrotic or gangrenous bowel. If an intra-arterial infusion of papaverine has been initiated, it should be continued throughout the exploratory laparotomy. Given the known propensity of this disease process for waxing and waning, a second-look laparotomy is also imperative.

A 32-year-old male with a newly diagnosed atrial myxoma presents with a cold, pulseless right leg. Emergent angiography and mechanical thrombectomy are performed. Total limb ischemic time is 9 hours. Following restoration of flow to the limb, which of the following potential sequelae is false? A) Shock may occur shortly after thrombectomy. B) Despite the presence of soft compartments, prophylactic fasciotomy is indicated. C) Cardiac arrhythmias may occur. D) Aggressive hydration may be necessary to prevent renal failure. E) Reperfusion is associated with minimal tissue injury as the toxic metabolites are rapidly removed by restoration of flow.

To recognize and anticipate the sequelae of ischemia-reperfusion Answer E is correct The reperfusion state that results when flow is restored can be as detrimental to the ischemic extremity as the hypoperfusion state was. During reperfusion, highly active oxygen metabolites are produced by neutrophils. These free radicals destroy cells by attacking the unsaturated bonds of fatty acids within the phospholipid membrane, thereby disrupting the cell membrane, allowing water to enter the cell, and eventually causing cell lysis. Free radical scavengers (e.g., mannitol and superoxide dismutase) have a slight protective effect against reperfusion injury when given before large-scale release of these radicals. In addition, myoglobin from injured muscle cells is released into the circulation and is cleared via the renal system. Myoglobin may cause renal failure through its direct toxic effect on the renal tubules and through the accumulation of casts in the tubules. Creatinine phosphokinase levels may also increase to dramatic levels once perfusion is reestablished. High concentrations of lactic acid, potassium, thromboxane, and cellular enzymes are secreted as a consequence of the rhabdomyolysis; these substances accumulate in the ischemic limb and are released into the systemic circulation on reperfusion. In one study that measured the venous effluent from a series of patients with limb ischemia, the pH was 7.07 and the mean potassium level was 5.7 mEq/L 5 minutes after surgical embolectomy. Detrimental physiologic changes are seen when toxic oxygen metabolites are released systemically. Depression of myocardial function, an increase in cardiac dysrhythmias, and loss of vascular tone may induce shock and even death. After surgical embolectomy, prophylactic four-compartment fasciotomy is recommended if the period of ischemia has been longer than 6 to 8 hours.

A patient with a solitary pulmonary nodule on a chest x-ray undergoes CT scanning to better characterize the lesion. What feature seen on a CT scan is most suggestive that the lesion is benign? A) Solid attenuation within areas of ground-glass opacity B) Lesion wall thinner than 4 mm C) Enhancement by 25 Hounsfield units (HU) after intravenous contrast injection D) Absence of intranodular fat E) Air bronchograms

To recognize computed tomographic (CT) scan findings consistent with benign etiology of a solitary pulmonary nodule Answer B is correct The morphologic characteristics of a solitary pulmonary nodule visualized on a CT scan can be extremely helpful in characterizing the probability of malignancy. Air bronchograms, solid attenuation within areas of ground-glass opacities, enhancement by more than 20 HU after intravenous contrast injection, and absence of intranodular fat are all suggestive of malignancy. Solitary pulmonary nodules with walls thinner than 4 mm are much more likely to be benign, whereas those with walls thicker than 16 mm are more likely to be malignant.

A 57-year-old man with dysphagia undergoes a barium swallow. A stricture is seen. Which of these features observed on barium swallow are suggestive of a malignant esophageal stricture as opposed to a peptic stricture? A) Short segment B) Mucosal irregularity C) Location at squamocolumnar junction D) Circumferential E) Narrow caliber

To recognize features of an esophageal stricture concerning for an underlying esophageal cancer Answer B is correct. A barium swallow shows the presence of an esophageal mass lesion as the lesion changes the caliber of the esophagus. Likewise, peptic strictures cause luminal narrowing. In the presence of esophageal cancer, mucosal irregularity can be seen in the narrowed segment. Peptic strictures, associated with gastroesophageal reflux disease, are short segment, circumferential, and located at the squamocolumnar junction; the mucosa is smooth, and the caliber is narrow.

An 81-year-old man undergoes an Ivor Lewis esophagectomy. His postoperative course is relatively uneventful, and he is discharged. He subsequently presents for his postoperative visits as scheduled. What symptom is not an indication that a late procedure-related complication has occurred? A) Regurgitation B) Dysphagia C) Dumping D) Cough E) Poor gastric emptying

To recognize late complications of esophagectomy Answer D is correct At every postoperative visit, symptoms of reflux, regurgitation, dumping, poor gastric emptying, and dysphagia must be specifically sought; these are major quality of life issues for postesophagectomy patients. Reflux symptoms generally respond to dietary modifications. Regurgitation is usually related to the supine position, and elevating the bed and avoiding late meals may suffice for symptoms control. Dumping is exacerbated by foods with high fat and sugar content. Dysphagia may be related to narrowing at the anastomosis. An incomplete pyloromyotomy may cause poor gastric emptying. Cough is less likely to be a direct complication related to esophagectomy.

A 72-year-old female with coronary artery disease, severe heart failure, hypertension, and peripheral arterial disease presents with abdominal pain to the emergency department. The history and physical examination reveal a cachectic woman with evidence of an upper respiratory infection and dry mucous membranes. Although her abdomen is soft and nondistended, she complains of constant, severe abdominal pain, which began several hours prior to presentation. A diagnosis of nonocclusive mesenteric ischemia (NOMI) is made following a diagnostic angiogram. Which of the following statements regarding NOMI is false? A) Management of NOMI largely consists of medical optimization. B) Despite the etiology, laparotomy is required when peritonitis is present. C) Angiography typically reveals absence of SMA filling. D) Vasopressors should be avoided during the resuscitative period. E) Intra-arterial infusion of papavarine may be of benefit.

To recognize nonocclusive mesenteric ischemia (NOMI) and appropriate management Answer C is correct. Patients with NOMI typically exhibit angiographic evidence of SMA vasospasm. A small SMA trunk is visualized, with very few branching vessels visible, and the branches that are visualized show a characteristic tapering of the vessel to the point of occlusion. These patterns are best seen on the anteroposterior projection. Management of NOMI is largely nonoperative. Once the diagnosis has been established with angiography, treatment of the underlying precipitating cause is the key therapeutic intervention. Optimization of fluid resuscitation, improvement in cardiac output, and elimination of vasopressors are the measures that have the greatest impact on outcome. Selective catheterization of the SMA with direct intra-arterial infusion of papaverine (30 to 60 mg/hr) may be employed as adjunctive therapy. The infusion is continued for at least 24 hours, with repeat angiography performed at regular intervals to determine the effectiveness of this therapy. Alternatively, some authors described using direct intra-arterial administration of tolazoline and glycerol trinitrite as local vasodilators in patients with NOMI with good clinical success. If the patient presents with peritoneal signs on physical examination, an exploratory laparotomy will be required for resection of frankly necrotic or gangrenous bowel. If an intra-arterial infusion of papaverine has been initiated, it should be continued throughout the exploratory laparotomy. Given the known propensity of this disease process for waxing and waning, a second-look laparotomy is also imperative.

A 62-year-old diabetic male patient presents to his primary care physician for a routine visit. He has an ulcer on the plantar surface of his great toe. Which of the following does not contribute to the development of diabetic foot disease? A) Macroangiopathic disease B) Sensory and autonomic neuropathy C) Skin fissuring D) Venous stasis E) Hyperkeratosis

To recognize risk factors and underlying factors that contribute to the development of diabetic foot disease) Answer D is correct The pathology of diabetic foot ulceration is multifactorial. Chronic hyperglycermia leads to advanced glycosylation end products and cross-linking of proteins, eventually resulting in foreshortening of tendons. Relative ischemia (particularly in the vasa vasorum), coupled with the glycosylated protein buildup, leads to sensory and autonomic neuropathy. The autonomic neuropathy leads to anhidrosis and ultimately to fissuring, which affords bacteria a means of entry through the skin. In addition, it leads to hyperkeratosis, which increases the pressure over pressure points and reduces sensation in a foot already afflicted by sensory compromise. The tibial nerves may also be compressed. The immune response is altered in diabetic patients. In particular, granulocyte function is impaired and surgical débridements and advanced dressings are often required to minimize the effects of this deficiency. Moreover, signs and symptoms of infection are notoriously inaccurate and masked in this setting. Blood flow is typically impaired in diabetic patients. The impairment usually develops at the macrovascular level as the result of accelerated atherosclerosis in the infrapopliteal area (in contrast to the peripheral vascular disease seen in nondiabetic patients, which mostly affects vessels in the thigh). This process underlies the phenomenon of pedal sparing, whereby the vessels in the foot and the ankle may be free of disease even though the more proximal vessels are not. Pedal sparing makes pedal bypasses and lower leg bypasses possible; therefore, revascularization is a mainstay of therapy for an ischemic diabetic foot. For years, many practitioners adhered to the concept of diabetic "microvascular disease," which suggested that performing revascularizations in diabetic patients was futile. This concept was based on anatomic studies that were later shown to be inaccurate. It would be a mistake, however, to maintain that there is no microangiopathy in diabetic patients. In poorly controlled diabetic patients, a functional microangiopathy probably does exist, particularly in regard to ischemia-mediated angiogenesis. Indeed, numerous preclinical studies have demonstrated decreased growth factor expression in diabetic healing models. Venous stasis is typically not a cause of diabetic foot disease.

A 62-year-old male with a history notable for peripheral arterial disease, hypertension, and smoking presents with a cold and pulseless left leg. Which of the following interventions would be inadequate as sole therapy in treating this patient? A) Catheter-directed thrombolysis B) IV heparin infusion C) Catheter-mediated thrombectomy D) Arterial bypass E) Percutaneous mechanical thrombectomy (PMT)

To recognize that heparin does not have intrinsic thrombolytic activity Answer B is correct. Heparin administration should be started as soon as the diagnosis of acute limb ischemia (ALI) is entertained. Numerous studies have shown that this measure decreases the morbidity and mortality associated with ALI and increases the limb salvage rate. Heparin impedes the propagation of clots and, in the instance of embolism, may help prevent additional events. Heparin acts at multiple sites in the normal coagulation system, inhibiting reactions that lead to the clotting of blood and the formation of fibrin clots both in vitro and in vivo. Small amounts of heparin, in combination with antithrombin (heparin cofactor), can inhibit thrombosis by inactivating activated factor X and inhibiting the conversion of prothrombin to thrombin. Once active thrombosis has developed, larger amounts of heparin can inhibit further coagulation by inactivating thrombin and preventing the conversion of fibrinogen to fibrin. Heparin also prevents the formation of a stable fibrin clot by inhibiting the activation of fibrin-stabilizing factor. Heparin does not have fibrinolytic activity and therefore does not lyse existing clots.

A 62-year-old male recently diagnosed with paroxysmal atrial fibrillation presents to the emergency department complaining of severe abdominal pain out of proportion to the physical examination. Mesenteric ischemia is considered in the differential diagnosis, and a CT angiogram is ordered. Which of the following is correct with regard to superior mesenteric artery (SMA) occlusion in thromboembolic mesenteric ischemia? A) There is no preference for SMA involvement relative to the celiac or inferior mesenteric arteries (IMAs). B) The SMA is the least likely mesenteric vessel to be involved in mesenteric ischemia. C) Emboli typically lodge at the origin of the SMA. D) Thrombi typically lodge distal to the middle colic branch of the SMA. E) Thrombi typically form at the origin of the SMA.

To recognize the angiographic pattern of superior mesenteric artery (SMA) occlusion in thrombotic versus embolic mesenteric ischemia Answer E is correct Of the three mesenteric vessels, the SMA is most likely to be the site of embolic lodgment because it takes off from the main axis of the aorta at a less sharp angle than the celiac artery and the IMA, which arise from the aorta more perpendicularly. When emboli lodge in the SMA, they usually lodge distal to the middle colic branch and the jejunal branch. In thrombotic disease, the thrombus usually forms at the atherosclerotic plaque, which, for most patients, is usually at the origin of the mesenteric vessel. Consequently, the angiogram typically demonstrates complete absence of flow in the mesenteric vessel, which often makes it difficult to ascertain the location of the vessel's origin.

A 37-year-old woman presents with bright red blood per rectum, and biopsy of the lesion reveals melanoma. Which of the following statements regarding anorectal melanoma is not true? A) Reported 5-year survival ranges between 0 and 12% B) Mean survival is improved with APR when compared with local resection C) Prognosis is related to tumor size, thickness, and clinical stage D) It accounts for 1% of all melanoma cases E) Prophylactic lymphadenectomy is not indicated for clinically negative nodes

To understand the basic characteristics of management of patients with anorectal melanoma Answer B is correct Anorectal melanomas are rare; they account for 1% of all melanomas and 0.25 to 1% of anorectal tumors. The mean age at occurrence is in the fifth decade; females are affected more frequently than males. The most frequent presenting symptom is bleeding, followed by an anal mass or pain. The lesions are usually elevated, and 34 to 75% will be pigmented. These tumors are locally invasive and have a high metastatic potential. Because many patients present late, the reported 5-year survival rates range from 0 to 12%. The prognosis is related to tumor size, thickness, and clinical stage. Evaluation should include a biopsy and a search for metastatic disease (by computed tomography of the abdomen, pelvis, and chest; liver function tests; chest x-ray evaluation; and bone scans). Special stains or electron microscopy may be required to confirm the diagnostic biopsy. Surgery provides the only hope for cure. However, the small chance for cure and limited experience have led to controversy about the appropriate procedure. An APR has significant morbidity and mortality. Local excision has less associated morbidity. Reports have shown little difference in the mean survival rates following either procedure. Prophylactic lymphadenectomy is not indicated for clinically negative nodes but is helpful for clinically suspicious nodes. Radiotherapy and chemotherapy have demonstrated little benefit in this disease.

A 62-year-old male recently diagnosed with paroxysmal atrial fibrillation presents to the emergency department complaining of severe abdominal pain out of proportion to the physical examination. Mesenteric ischemia is considered in the differential diagnosis, and a computed tomographic (CT) angiogram is ordered. Which of the following is correct with regard to superior mesenteric artery (SMA) occlusion in thromboembolic mesenteric ischemia? A) There is no preference for SMA involvement relative to the celiac or inferior mesenteric arteries (IMAs). B) The SMA is the least likely mesenteric vessel to be involved in mesenteric ischemia. C) Emboli typically lodge at the origin of the SMA. D) Thrombi typically lodge distal to the middle colic branch of the SMA. E) Thrombi typically form at the origin of the SMA.

To recognize the angiographic pattern of superior mesenteric artery (SMA) occlusion in thrombotic versus embolic mesenteric ischemia Answer E is correct Of the three mesenteric vessels, the SMA is most likely to be the site of embolic lodgment because it takes off from the main axis of the aorta at a less sharp angle than the celiac artery and the IMA, which arise from the aorta more perpendicularly. When emboli lodge in the SMA, they usually lodge distal to the middle colic branch and the jejunal branch. In thrombotic disease, the thrombus usually forms at the atherosclerotic plaque, which, for most patients, is usually at the origin of the mesenteric vessel. Consequently, the angiogram typically demonstrates complete absence of flow in the mesenteric vessel, which often makes it difficult to ascertain the location of the vessel's origin.

A 68-year-old male lifelong smoker with hypertension relates to his primary care physician that he has not been able to walk more than one block without cramping in his legs. In addition to a complete physical examination, the physician obtains an ankle-brachial index (ABI) for each leg in the office Which of the following is false regarding the measurement obtained? A) An ABI < 0.6 is consistent with occlusive arterial disease. B) In a claudicant, an ABI of 0.8 rules out arterial insufficiency; other causes should be investigated. C) Doppler ultrasonography is used to listen to the dorsalis pedis and posterior tibial arteries to identify the systolic pressure. D) The ABI is calculated as the ratio of the ankle to brachial artery systolic pressure. E) ABIs should be calculated in all vasculopaths

To recognize the limitations of the ABI Answer B is correct The ABI is derived from the ankle systolic pressure and the brachial systolic pressure and is determined as follows. The systolic pressure is measured in each arm, and the higher of the two measurements is taken to be the brachial systolic pressure. A cuff is then placed on each calf, and the examiner listens to signals in the dorsalis pedis and posterior tibial arteries. The cuff is inflated until the signal is no longer heard. At this point, the cuff is slowly released, and the systolic pressure is recorded at the point where the signal is once again audible. Again, the higher of the two systolic measurements is taken to be the ankle systolic pressure. The systolic ankle pressure is then divided by the brachial systolic pressure to yield the ABI. An ABI in the range of 0.9 to 1.0 is normal; however, the ABI can be falsely elevated if the distal arteries are not compressible. When the ABI falls below 0.6, there is a significant difference in blood pressure between the proximal arterial tree and the distal extremity, which usually denotes an occlusive process. Next, segmental pressures are obtained by placing cuffs at the ankle, below the knee, above the knee, and on the thigh. Systolic blood pressures are measured at each location, and any pressure drop greater than 15 mm Hg is considered significant.

A 68-year-old male lifelong smoker with hypertension relates to his primary care physician that he has not been able to walk more than one block without cramping in his legs. In addition to a complete physical examination, the physician obtains an ankle-brachial index (ABI) for each leg in the office. Which of the following is false regarding the measurement obtained? A) An ABI < 0.6 is consistent with occlusive arterial disease. B) In a claudicant, an ABI of 0.8 rules out arterial insufficiency; other causes should be investigated. C) Doppler ultrasonography is used to listen to the dorsalis pedis and posterior tibial arteries to identify the systolic pressure. D) The ABI is calculated as the ratio of the ankle to brachial artery systolic pressure. E) ABIs should be calculated in all vasculopaths

To recognize the limitations of the ankle-brachial index (ABI) Answer B is correct The ABI is derived from the ankle systolic pressure and the brachial systolic pressure and is determined as follows. The systolic pressure is measured in each arm, and the higher of the two measurements is taken to be the brachial systolic pressure. A cuff is then placed on each calf, and the examiner listens to signals in the dorsalis pedis and posterior tibial arteries. The cuff is inflated until the signal is no longer heard. At this point, the cuff is slowly released, and the systolic pressure is recorded at the point where the signal is once again audible. Again, the higher of the two systolic measurements is taken to be the ankle systolic pressure. The systolic ankle pressure is then divided by the brachial systolic pressure to yield the ABI. An ABI in the range of 0.9 to 1.0 is normal; however, the ABI can be falsely elevated if the distal arteries are not compressible. When the ABI falls below 0.6, there is a significant difference in blood pressure between the proximal arterial tree and the distal extremity, which usually denotes an occlusive process. Next, segmental pressures are obtained by placing cuffs at the ankle, below the knee, above the knee, and on the thigh. Systolic blood pressures are measured at each location, and any pressure drop greater than 15 mm Hg is considered significant.

A 54-year-old female admitted to the medical intensive care unit with acute-onset chronic respiratory failure and new-onset atrial fibrillation is noted to have a cold, mottled left leg, which the patient states "feels heavy." After attempting to warm the limb with blankets and a heating pad to no avail, a surgical consultation is obtained approximately 4 hours later. Which of the following physical examination findings is not consistent with acute limb ischemia (ALI)? A) No palpable pulses in the left foot B) Erythema below the knee C) Inability to move the leg D) Cool left leg E) Loss of sensation to light touch

To recognize the physical examination findings of acute limb ischemia (ALI) Answer D is correct The characteristic signs of ALI may be summarized as the six Ps: pulselessness, pain, pallor, poikilothermia, paresthesia, and paralysis. 1. Pulses should be palpated and documented. Any previous documentation should be noted and used for comparison. Careful evaluation of pulses will help localize the area of arterial obstruction. When a clot is fresh, its soft, semiliquid consistency allows the pulse to be translated at the level of obstruction. Only when the thrombus becomes organized and densely compacted is the pulse lost at the site of occlusion. As an example, in a patient with obstruction at the popliteal artery, popliteal pulses remain palpable in the earlier stages of the process, but distal pulses are lost. 2. Pain should be documented with regard to severity, area, and progression. 3. Pallor may be seen in the early stages, followed by cyanosis. Erythema is not seen in ALI. 4. Poikilothermia may propagate the ischemic cascade through its vasoconstrictive effects. The level of coolness and pallor is typically one level below the point of occlusion on the arterial tree and should correlate with the pulses or signals found. As always, baseline documentation should be done so that the progression or resolution of the process can be tracked. 5. Paresthesia is an essential finding. The earliest sign of tissue loss is the loss of light touch, two-point discrimination, vibratory perception, and proprioception, especially in the first dorsal web space of the foot. 6. Paralysis, if present, is an indication of advanced limb-threatening ischemia.

A 54-year-old female admitted to the medical intensive care unit with acute-onset chronic respiratory failure and new-onset atrial fibrillation is noted to have a cold, mottled left leg, which the patient states "feels heavy." After attempting to warm the limb with blankets and a heating pad to no avail, a surgical consultation is obtained approximately 4 hours later. Which of the following physical examination findings is not consistent with ALI? A) No palpable pulses in the left foot B) Erythema below the knee C) Inability to move the leg D) Cool left leg E) Loss of sensation to light touch

To recognize the physical examination findings of acute limb ischemia (ALI) Answer D is correct The characteristic signs of ALI may be summarized as the six Ps: pulselessness, pain, pallor, poikilothermia, paresthesia, and paralysis. 1. Pulses should be palpated and documented. Any previous documentation should be noted and used for comparison. Careful evaluation of pulses will help localize the area of arterial obstruction. When a clot is fresh, its soft, semiliquid consistency allows the pulse to be translated at the level of obstruction. Only when the thrombus becomes organized and densely compacted is the pulse lost at the site of occlusion. As an example, in a patient with obstruction at the popliteal artery, popliteal pulses remain palpable in the earlier stages of the process, but distal pulses are lost. 2. Pain should be documented with regard to severity, area, and progression. 3. Pallor may be seen in the early stages, followed by cyanosis. Erythema is not seen in ALI. 4. Poikilothermia may propagate the ischemic cascade through its vasoconstrictive effects. The level of coolness and pallor is typically one level below the point of occlusion on the arterial tree and should correlate with the pulses or signals found. As always, baseline documentation should be done so that the progression or resolution of the process can be tracked. 5. Paresthesia is an essential finding. The earliest sign of tissue loss is the loss of light touch, two-point discrimination, vibratory perception, and proprioception, especially in the first dorsal web space of the foot. 6. Paralysis, if present, is an indication of advanced limb-threatening ischemia.

A 52-year-old male with an ischemic right leg following a traumatic dislocation of the knee joint undergoes a revascularization procedure for several hours. Which of the following does not occur? A) Depressed myocardial function B) Generation of oxygen free radicals by macrophages C) Disruption of cellular phospholipid membranes D) Renal failure E) Vasodilatation

To recognize the physiologic insults resulting specifically from reperfusion Answer B is correct The reperfusion state that results when flow is restored can be as detrimental to the ischemic extremity as the hypoperfusion state was. During reperfusion, highly active oxygen metabolites are produced by neutrophils. These free radicals destroy cells by attacking the unsaturated bonds of fatty acids within the phospholipid membrane, thereby disrupting the cell membrane, allowing water to enter the cell, and eventually causing cell lysis. Free radical scavengers (e.g., mannitol and superoxide dismutase) have a slight protective effect against reperfusion injury when given before large-scale release of these radicals. In addition, myoglobin from injured muscle cells is released into the circulation and is cleared via the renal system. Myoglobin may cause renal failure through its direct toxic effect on the renal tubules and through the accumulation of casts in the tubules. Creatinine phosphokinase levels may also increase to dramatic levels once perfusion is reestablished. High concentrations of lactic acid, potassium, thromboxane, and cellular enzymes are secreted as a consequence of the rhabdomyolysis; these substances accumulate in the ischemic limb and are released into the systemic circulation upon reperfusion. Detrimental physiologic changes are seen when toxic oxygen metabolites are released systemically. Depression of myocardial function, an increase in cardiac dysrhythmias, and loss of vascular tone may induce shock and even death.

A 44-year-old male type 1 diabetic patient presents with an erythematous, draining wound on the medial aspect of the first metatarsophalangeal joint of the right foot. Wound cultures are sent prior to the initiation of antibiotic therapy. Which of the following organisms might be identified in the culture from this wound? A) Escherichia coli B) Klebsiella C) Bacteroides D) Pseudomonas E) All of the above

To recognize the polymicrobial nature of infected diabetic foot wounds so that adequate antimicrobial therapy can be initiated Answer E is correct The microbiology of the diabetic foot varies according to the depth and severity of the infection and the nature of the patient's environment (e.g., hospitalized or outpatient). Certain general assumptions can be made about likely causative organisms. Mild localized and superficial ulcerations, particularly in outpatients, are usually caused by aerobic gram-positive cocci (e.g., Staphylococcus aureus and streptococci). In contrast, deeper ulcers and generalized limb-threatening infections are usually polymicrobial. In addition to gram-positive cocci, potential causative organisms include gram-negative bacilli (e.g., E. coli, Klebsiella, Enterobacter aerogenes, Proteus mirabilis, and Pseudomonas aeruginosa) and anaerobes (e.g., Bacteroides fragilis and peptostreptococci). Enterococci may also be isolated from the wound, notably in hospitalized patients; in the absence of other cultured virulent organisms, they should probably be considered pathogenic.

A 44-year-old male type 1 diabetic patient presents with an erythematous, draining wound on the medial aspect of the first metatarsophalangeal joint of the right foot. Wound cultures are sent prior to the initiation of antibiotic therapy. Which of the following organisms might be identified in the culture from this wound? A) Escherichia coli B) Klebsiella C) Bacteroides D) Pseudomonas E) All of the above

To recognize the polymicrobial nature of infected diabetic foot wounds so that adequate antimicrobial therapy can be initiated Answer E is correct The microbiology of the diabetic foot varies according to the depth and severity of the infection and the nature of the patient's environment (e.g., hospitalized or outpatient). Certain general assumptions can be made about likely causative organisms. Mild localized and superficial ulcerations, particularly in outpatients, are usually caused by aerobic gram-positive cocci (e.g., Staphylococcus aureus and streptococci). In contrast, deeper ulcers and generalized limb-threatening infections are usually polymicrobial. In addition to gram-positive cocci, potential causative organisms include gram-negative bacilli (e.g., E. coli, Klebsiella, Enterobacter aerogenes, Proteus mirabilis, and Pseudomonas aeruginosa) and anaerobes (e.g., Bacteroides fragilis and peptostreptococci). Enterococci may also be isolated from the wound, notably in hospitalized patients; in the absence of other cultured virulent organisms, they should probably be considered pathogenic.

A 65-year-old female undergoes an uncomplicated right hemicolectomy for adenocarcinoma. Her past medical history is notable for multiple occurrences of deep vein thrombosis following prior cesarean sections, although she has not undergone a formal evaluation for a hypercoagulable state. Several days following an uncomplicated postoperative recovery and discharge to home, she returns with acute-onset abdominal pain. Physical examination reveals a soft, nondistended abdomen without guarding, although the patient complains of nonfocal, severe abdominal pain. Which of the following tests has the highest sensitivity and specificity for diagnosing mesenteric venous thrombosis (MVT)? A) Catheter-based angiography B) CT angiography C) D-dimer and lactate levels D) Physical examination E) History

To recognize the presentation of mesenteric venous thrombosis (MVT) and its diagnosis Answer B is correct Patients with MVT often present with various nonspecific abdominal complaints; accordingly, this diagnosis may be especially challenging. Common complaints include nausea, vomiting, diarrhea, abdominal cramping, and nonlocalized abdominal pain. As a rule, these symptoms are not acute. A study of MVT patients found that 84% presented with abdominal pain. Of those 84%, only 16% presented with peritoneal signs, whereas 68% presented with vague abdominal pain. Other presenting symptoms included diarrhea (42%), nausea and vomiting (32%), malaise (16%), and upper gastrointestinal bleeding (10%). The risk factors for MVT include a history of previous venous thrombosis or pulmonary embolism, a known or suspected hypercoagulable state, oral contraception, and estrogen supplementation. In a study of 31 patients who presented with MVT at Northwestern University, 13 (42%) were diagnosed with a hypercoagulable state, six (19%) had a history of previous thrombotic episodes, and four (13%) had a history of cancer. In contrast to its relatively limited role in the diagnosis of acute mesenteric ischemia of embolic or thrombotic origin, CT plays a valuable role in diagnosing MVT and is the preferred diagnostic imaging modality in patients presenting with abdominal pain who have a history of deep vein thrombosis or a known hypercoagulable disorder. CT scanning can readily reveal thrombosis of the superior mesenteric vein (SMV), with or without associated bowel abnormalities. In fact, CT scans of SMV thrombosis in asymptomatic patients have provided useful information on the pathophysiology of MVT and broadened our understanding of the wide spectrum of this disease entity. In a study from the Mayo Clinic, CT scanning correctly identified 100% of patients who presented with acute MVT and 93% of those who presented with chronic venous thrombosis. In a subsequent study from our institution (Northwestern University), CT scanning identified 100% of MVT patients who presented with vague abdominal pain or diarrhea and 90% of MVT patients who underwent a CT scan. In contrast, conventional angiography correctly diagnosed MVT in only five of nine patients.

An 18-year-old male presents to his pediatrician following soccer practice complaining of severe right calf pain after being kicked in the leg earlier in the day. His past medical history is unremarkable, and on physical examination, a tense calf is appreciated with numbness identified in the first web space of the right foot. If left untreated, tissue ischemia and necrosis will be caused by which of the following? A) Cell membrane destruction by elevated pressure B) Arterial inflow obstruction C) Venous outflow obstruction D) Activation of apoptotic pathways E) Elevated lactate level

To recognize the underlying pathophysiology resulting in tissue ischemia/death in an acute compartment syndrome Answer C is correct Regardless of the cause, increasing pressure within a fixed space between fascial structures may result in venous outflow obstruction. As this pressure approaches 25 mm Hg, flow through compartmental venules is significantly impeded such that inflow is ultimately affected. Left unchecked, the compartment becomes ischemic, which may further exacerbate the rising pressure (increasing tissue edema from ischemia). If not addressed in a timely manner, the resulting ischemia will progress to tissue necrosis. The "ischemic time" tolerated is dictated by the tissue in question—the shortest times are associated with neural tissue (i.e., the brain), with a progressively longer ischemic time tolerated by intra-abdominal viscera and the longest tolerated by muscle groups.

An 18-year-old male presents to his pediatrician following soccer practice complaining of severe right calf pain after being kicked in the leg earlier in the day. His past medical history is unremarkable, and on physical examination, a tense calf is appreciated with numbness identified in the first web space of the right foot. If left untreated, tissue ischemia and necrosis will be caused by which of the following? A) Cell membrane destruction by elevated pressure B) Arterial inflow obstruction C) Venous outflow obstruction D) Activation of apoptotic pathways E) Elevated lactate level

To recognize the underlying pathophysiology resulting in tissue ischemia/death in an acute compartment syndrome Answer C is correct. Regardless of the cause, increasing pressure within a fixed space between fascial structures may result in venous outflow obstruction. As this pressure approaches 25 mm Hg, flow through compartmental venules is significantly impeded such that inflow is ultimately affected. Left unchecked, the compartment becomes ischemic, which may further exacerbate the rising pressure (increasing tissue edema from ischemia). If not addressed in a timely manner, the resulting ischemia will progress to tissue necrosis. The "ischemic time" tolerated is dictated by the tissue in question—the shortest times are associated with neural tissue (i.e., the brain), with a progressively longer ischemic time tolerated by intra-abdominal viscera and the longest tolerated by muscle groups.

You are planning surgery in a 28-year-old man with ulcerative colitis (UC). In patients with UC, absolute contraindications to immediate restorative proctectomy with ileal pouch-anal anastomosis (IPAA) include all of the following except: A) Perforation B) Peritonitis C) Sepsis D) Suspicion of Crohn colitis or indeterminate colitis E) Diagnosis of fulminant colitis

To recognize when a staged procedure is indicated and when immediate restorative proctectomy with ileal pouch-anal anastomosis (IPAA) is contraindicated Answer E is correct The precise parameters under which it is best to stage the procedure with an initial abdominal colectomy have not been clearly defined. It is universally accepted that patients with perforation, peritonitis, or sepsis require a staged procedure. Any patient with suspicion of Crohn colitis or indeterminate colitis should undergo subtotal colectomy to allow for further diagnostic evaluation prior to creation of IPAA. Beyond this, there is no clear consensus. The available studies addressing this issue unfortunately involve a small number of patients, do not clearly define what is meant by "fulminant" colitis, or do not directly compare results between the two alternative strategies of staged colectomy versus immediate ileoanal anastomosis. Clearly, there is a small subset of patients with symptoms severe enough to require hospitalization who are healthy enough to safely undergo a primary ileoanal anastomosis. On the other end of the spectrum, severely ill patients, that is, most patients with fulminant colitis, should have a staged procedure. Because specific criteria to quantify the risk have not been defined, the decision to stage or not to stage ultimately rests with the clinical judgment of the experienced surgeon.

A 58-year-old female with chronic venous stasis disease presents with a new ulceration, which developed approximately 2 weeks previously. Despite keeping the wound clean with a dry dressing, the wound has continued to enlarge with accumulation of a yellowish base. Which of the following must be removed to facilitate healing? A) Biofilm B) Exposed tendon C) Nonviable tissue D) Exposed periosteum E) 1 and 3

To recognize wound characteristics that impede healing Answer E is correct There are three components of a leg ulcer that must be removed by means of débridement: (1) biofilm and bacteria, (2) callus, and (3) nonviable tissue. Whereas the role of bacteria in wound infections has long been recognized, it is only comparatively recently that the contributions of biofilm to wound chronicity have come to be appreciated. Biofilm consists of a sessile community of multiple bacteria species encased by a protective carbohydrate-rich polymeric matrix that is resistant to antimicrobial and immune cell penetration. Most wounds are, in fact, colonized by bacteria that set up residence in a biofilm. Unfortunately, biofilm is exceedingly tenacious and readily accumulates after débridement. Thus, proper care consists of dressings that both treat the wound and minimize biofilm accumulation. Nonviable tissue can give rise to an excessively proinflammatory state characterized by high levels of free radical and proteases. This proinflammatory state is hostile to healing, propagates cellular damage, and competes with host cells for scarce resources. Therefore, all eschar and nonviable tissues should be removed.

An elderly man with a history of alcohol abuse presents with dysphagia. He has a 20-pound weight loss, and you suspect esophageal cancer. What is the first step in the diagnostic workup of a patient with dysphagia attributable to suspected esophageal cancer? A) Endoscopic ultrasonography (EUS) B) Computed tomography (CT) and positron emission tomography (PET) C) Upper endoscopy D) Barium swallow E) Manometric assessment

To select the appropriate initial diagnostic test for a patient with suspected esophageal cancer Answer D is correct Many diagnostic tests can be used to assess dysphagia. In all cases of dysphagia, except caustic ingestion, the barium swallow is the ideal first test as it is readily available, cost-effective, and rapidly performed. In addition to the information that can be gained from the barium study, the safety and diagnostic yield of subsequent upper endoscopy are also enhanced. Upper endoscopy follows the barium swallow if an esophageal mass is suspected and allows for diagnostic maneuvers. EUS permits the characterization of esophageal masses and assessment of adjacent lymphadenopathy. CT and PET are required to assess esophageal cancer and to aid in the detection of unsuspected metastatic lesions. Esophageal manometry has no role in the diagnostic workup of a patient with suspected esophageal cancer.

A 53-year-old man is undergoing his third small bowel resection for Crohn disease. When operating on a patient with Crohn disease, if bowel is going to be resected, the extent of the resection margin is best determined by: A) Previous response to medical therapy B) The distribution of lymph nodes throughout the mesentery C) Intraoperative frozen-section margins D) The number of strictureplasties that can be safely performed E) Identification of a palpably normal mesenteric border of bowel

To understand basic surgical principles in the management of Crohn disease Answer E is correct The area to be resected should be as short as possible. There is no need to obtain frozen-section margins to determine the extent of resection; doing so leads to unnecessary loss of small bowel length. The resection should extend into palpably normal areas of small bowel. The easiest way to determine the area to be resected is to feel the mesenteric margin of the bowel until palpably normal tissue is reached. Because Crohn disease is generally more severe on the mesenteric side of the bowel, palpation in this area gives the most accurate impression of the intraluminal character of the bowel. Because it is not uncommon for patients to have multifocal Crohn disease, the entire small bowel should always be inspected at the time of operation. Operating on one area of disease while failing to treat a more proximal lesion is clearly not in the patient's interest.

A 42-year-old man presents with palpitations, weight loss, and increased appetite. Physical examination is notable for exophthalmos and a diffusely enlarged thyroid. Laboratory studies reveal an elevated T3 and T4, with a low TSH. Which of the following is true in Graves disease? A) A bruit over the thyroid gland excludes the diagnosis of Graves disease B) Radioactive iodine treatment is only transiently effective in Graves disease C) Partial thyroidectomy is sufficient surgical therapy for Graves disease D) Radioactive iodine is no more cost-effective than surgery or antithyroid medication E) Beta blockade and thionamides are contraindicated preoperatively in patients with Graves disease

To understand diagnosis and treatment of Graves disease Answer D is correct In a patient with Graves disease and a goiter, the gland becomes hypervascular and a bruit is commonly heard on examination. Treatment options for Graves disease include antithyroid medications (thionamides such as propylthiouracil [PTU] or methimazole), radioactive iodine, or surgery. Radioactive iodine is effective and rarely results in relapse. Surgical therapy should consist of total thyroidectomy as partial thyroidectomy does not typically alleviate the hyperthyroidism. The cost-effectiveness of the three treatment options is comparable and is not typically the deciding factor in which method to use. Preoperatively, beta blockade and thionamides are useful in controlling the symptoms of thyrotoxicosis.

A patient presents to the surgeon with a presumed diagnosis of gastroesophageal reflux disease (GERD), which is confirmed by preoperative testing. A laparoscopic Nissen fundoplication is considered. What patient factor predicts a high failure rate for this operation? A) The patient has endoscopy-documented grade III esophagitis B) The patient shows good response to proton pump inhibitors (PPIs) preoperatively C) The patient complains of heartburn D) The patient's 24-hour pH score is abnormal E) The patient's body mass index (BMI) is greater than 35 kg/m2

To understand predictors of good outcome after laparoscopic Nissen fundoplication Answer E is correct The development of laparoscopic surgery has caused a shift in the treatment of GERD such that surgical management is considered at an earlier stage. However, patients with a BMI of 35 kg/m2 or greater experienced higher failure rates after laparoscopic Nissen fundoplication. Severe GERD documented on endoscopy will likely benefit from Nissen fundoplication. Similarly, response to a PPI, an abnormal 24-hour pH score, and the presence of a typical primary symptom of GERD have been shown to predict successful outcome of Nissen fundoplication.

A patient is diagnosed with sternoclavicular joint infection secondary to an infected subclavian central line. What is not part of the treatment for this condition? A) Débridement of the sternoclavicular joint B) Débridement of the manubrium C) Resection of the proximal first or second rib D) Antibiotic therapy E) Reconstruction with a muscle flap

To understand proper management of a patient with sternoclavicular joint infection Answer A is correct Sternoclavicular joint infection requires an aggressive approach. Treatment consists of wide resection of the sternoclavicular joint, including the proximal third of the clavicle, as well as débridement of the manubrium. The proximal portion of the first or second ribs can be involved and also requires resection. Immediate reconstruction is performed by rotation of a pectoralis flap into the resection cavity. Postoperatively, the patients are treated with 6 weeks of IV antibiotics.

An elderly patient with a nearly obstructing esophageal cancer is a candidate for esophagectomy. What measure should not be considered or executed to optimize the patient's health status in preparation for esophagectomy? A) Administer intravenous fluids B) Place a percutaneous endoscopic gastrostomy (PEG) tube for enteral feeding C) Perform laboratory investigation to assess renal and hepatic function D) Initiate total parenteral nutrition E) Obtain an echocardiogram

To understand proper preparation of patients for esophagectomy Answer B is correct For patients who are being prepared for major surgery, cardiovascular, renal, and hepatic function should be documented and, if possible, optimized. Electrolyte abnormalities can be corrected with intravenous fluid administration. Nutrition is often insufficient in patients with obstructing esophageal disease. In preoperative patients, most surgeons prefer not to have a PEG tube placed in the stomach for fear of compromising its blood supply when an esophagectomy is planned. If supplementary enteral nutrition is necessary, a nasogastric or jejunostomy tube should be considered; total parenteral nutrition is also an acceptable alternative if enteral feeding is not possible.

Which of the following is not true regarding the anatomy of the anal canal and anal margin? A) The anal canal runs from the anorectal junction to the intersphincteric groove B) Above the dentate line the mucosa is formed by transitional epithelium and squamous epithelium C) The anal margin runs from the intersphincteric groove to approximately 5 cm on the perineum D) Keratinized epithelium covers the anal margin E) All of the above are true

To understand the anatomy of the anal canal and anal margin Answer D is correct For clinical purposes, the anus can be divided into two areas: the anal canal and the anal margin. The anal canal runs from the anorectal junction (top of the anal sphincter muscles) to the intersphincteric groove (approximately 2 cm distal to the dentate line). Thus, it corresponds to the internal sphincter. The lining of this portion of the anus is formed by transitional epithelium, which contains elements of both columnar and squamous epithelium above the dentate line and squamous epithelium distal to the dentate line. The anal margin runs from the intersphincteric groove to approximately 5 cm on the perineum. This area is covered by nonkeratinizing squamous epithelium, which changes to keratinizing squamous epithelium at the anal margin's outer border with the perineal skin.

You are in the process of performing a left thyroid lobectomy on a 34-year-old woman. Which of the following is true regarding the recurrent laryngeal nerve? A) Identification of the left recurrent laryngeal nerve is best performed where it enters the cricothyroid muscle B) If operating for a known cancer, the left recurrent laryngeal nerve should be sacrificed for oncologic purposes C) The left recurrent laryngeal nerve takes a more oblique course than the right recurrent laryngeal nerve D) The left recurrent laryngeal nerve never branches before entering the cricothyroid muscle E) Anterior and medial retraction of the thyroid lobe facilitates identification of the recurrent laryngeal nerve

To understand the anatomy of the recurrent laryngeal nerves Answer E is correct To identify the recurrent laryngeal nerve, the carotid sheath is retracted laterally, and the thyroid gland is retracted anteriorly and medially. This retraction puts tension on the inferior thyroid artery and consequently on the recurrent laryngeal nerve, thereby facilitating identification of the nerve. The right and left recurrent laryngeal nerves must be preserved during every thyroid operation, even if operating for cancer. Although both nerves enter at the posterior medial position of the larynx in the cricothyroid muscle, their courses vary considerably. The right recurrent laryngeal nerve takes a more oblique course than the left recurrent laryngeal nerve does and may pass either anterior or posterior to the inferior thyroid artery. In about 0.5% of the population, the right recurrent laryngeal nerve is, in fact, nonrecurrent and may enter the thyroid from a superior or lateral direction. On rare occasions, both a recurrent and a nonrecurrent laryngeal nerve may be present on the right. The left recurrent laryngeal nerve almost always runs in the tracheoesophageal groove because of its deeper origin within the thorax as it loops around the ductus arteriosus. Either recurrent laryngeal nerve may branch before entering the larynx; the left nerve is more likely to do this. Such branching is important to recognize because the motor fibers of the recurrent laryngeal nerve are usually located within the most medial branch. It is usually safest to identify the recurrent laryngeal nerve low in the neck and then to follow it to the site where it enters the cricothyroid muscle through the Berry ligament.

You are in the process of performing a left thyroid lobectomy on a 34-year-old woman. Which of the following is true regarding the recurrent laryngeal nerve? A) Identification of the left recurrent laryngeal nerve is best performed where it enters the cricothyroid muscle B) If operating for a known cancer, the left recurrent laryngeal nerve should be sacrificed for oncologic purposes C) The left recurrent laryngeal nerve takes a more oblique course than the right recurrent laryngeal nerve D) The left recurrent laryngeal nerve never branches before entering the cricothyroid muscle E) Anterior and medial retraction of the thyroid lobe facilitates identification of the recurrent laryngeal nerve

To understand the anatomy of the recurrent laryngeal nerves Answer E is correct To identify the recurrent laryngeal nerve, the carotid sheath is retracted laterally, and the thyroid gland is retracted anteriorly and medially. This retraction puts tension on the inferior thyroid artery and consequently on the recurrent laryngeal nerve, thereby facilitating identification of the nerve. The right and left recurrent laryngeal nerves must be preserved during every thyroid operation, even if operating for cancer. Although both nerves enter at the posterior medial position of the larynx in the cricothyroid muscle, their courses vary considerably. The right recurrent laryngeal nerve takes a more oblique course than the left recurrent laryngeal nerve does and may pass either anterior or posterior to the inferior thyroid artery. In about 0.5% of the population, the right recurrent laryngeal nerve is, in fact, nonrecurrent and may enter the thyroid from a superior or lateral direction. On rare occasions, both a recurrent and a nonrecurrent laryngeal nerve may be present on the right. The left recurrent laryngeal nerve almost always runs in the tracheoesophageal groove because of its deeper origin within the thorax as it loops around the ductus arteriosus. Either recurrent laryngeal nerve may branch before entering the larynx; the left nerve is more likely to do this. Such branching is important to recognize because the motor fibers of the recurrent laryngeal nerve are usually located within the most medial branch. It is usually safest to identify the recurrent laryngeal nerve low in the neck and then to follow it to the site where it enters the cricothyroid muscle through the Berry ligament.

A 56-year-old man is postoperative day 7 from a sigmoid colectomy for a previous history of diverticulitis. There was no evidence of active diverticulitis at the time of operation, and the final pathology report ruled out any evidence of malignancy. He has not passed flatus or stool since his operation and complains of nausea as well as crampy abdominal pain. He has a nasogastric tube, which is having bilious output. On examination, he is afebrile, with normal vital signs, and has a distended abdomen without peritoneal signs. His wound is clean, dry, and intact, without evidence of infection. His white blood cell count is normal. A CT scan demonstrates distended, fluid-filled loops of small bowel with air-fluid levels, hyperemia, and bowel wall thickening. There is no evidence of abscess. There is air in the colon, which is decompressed, and the colorectal anastomosis appears patent, without evidence of leak. What is the next most appropriate step in management? A) Observation B) Replace the nasogastric tube with a long intestinal tube C) Exploratory laparotomy D) Colonoscopic decompression E) Diagnostic laparoscopy

To understand the appropriate management for early postoperative bowel obstruction Answer A is correct Early postoperative obstruction is caused by adhesions in about 90% of patients. When there are no signs of toxicity and no acute abdominal signs, such an obstruction can usually be managed safely with nasogastric decompression. As many as 87% of patients respond to nasogastric suction within 2 weeks. About 70% of the patients who respond to nonoperative treatment do so within 1 week, and an additional 25% respond during the following 7 days. If postoperative obstruction does not resolve in the first 2 weeks, it is unlikely to do so with continued nonoperative therapy, and reoperation is probably indicated; about 25% of patients whose postoperative obstruction was initially treated nonoperatively eventually require reoperation. An exception to this guideline arises in patients known to have severe dense adhesions (sometimes referred to as obliterative peritonitis) in response to multiple sequential laparotomies. These patients may have a combination of mechanical obstruction and diffuse small bowel and colonic ileus. The risk of closed-loop obstruction, volvulus, or strangulation in this group of patients is low. Repeat laparotomies and attempts to lyse adhesions may lead to complications, the development of enterocutaneous fistulae, or exacerbation of the adhesions. Often the best approach to managing these patients is observation for prolonged periods (i.e., months). Total parenteral nutrition (TPN) is indicated. The addition of octreotide to the TPN solution may be helpful and may make patients more comfortable. Because the risk of intestinal strangulation in patients with postoperative adhesive obstruction is extremely low (< 1%), one can generally treat these patients nonoperatively for longer periods. In fact, the conservative approach is often the wise one: reoperation may do more harm than good (e.g., by causing enterotomies and inducing denser adhesions). The traditional indications for operation in patients with early postoperative obstruction include (1) deteriorating clinical status, (2) worsening obstructive symptoms, and (3) failure to respond to nonoperative management within 2 weeks. With the rising cost of hospitalization, it might, in fact, be more cost-effective to reoperate on patients who have persistent obstruction after 7 days. This speculation would have to be tested by a well-organized cost-benefit study conducted in a prospective fashion.

A 38-year-old man presents to you with a chief complaint of hemorrhoids. On examination, you find hemorrhoids that are proximal to the dentate line and prolapse but spontaneously reduce. What is the most appropriate classification for these hemorrhoids? A) Grade 1 B) Grade 2 C) Grade 3 D) Grade 4 E) External

To understand the classification of internal hemorrhoids Answer B is correct. Internal hemorrhoidal disease is demonstrated by two main symptoms: painless bleeding and protrusion. Pain is rarely associated with internal hemorrhoids because they originate above the dentate line in insensate rectal mucosa. The most popular etiologic theory states that hemorrhoids result from chronic straining at defecation (upright posture and heavy lifting may also contribute). This straining not only causes hemorrhoidal engorgement but also creates forces that decrease the fixation between the hemorrhoids and the anal muscular wall. Continued straining causes further engorgement and bleeding, as well as hemorrhoidal prolapse. Internal hemorrhoids are categorized into four grades based on clinical findings and symptoms: 1, bleeding without prolapse; 2, prolapse that spontaneously reduces; 3, prolapse requiring manual reduction; and 4, irreducible prolapse.

A 58-year-old woman presents with a chest wall mass. Pathologic examination shows that her mass is a desmoid tumor. Which of the following is true about desmoid tumors? A) They are histologically malignant B) They are managed with radical surgical excision C) They arise from bone D) They do not affect surrounding structures E) They tend not to recur

To understand the clinical behavior of chest wall desmoid tumors Answer B is correct Desmoid tumors arise in the muscle and fascia. They appear histologically benign but are locally aggressive and can infiltrate surrounding structures and have a high tendency for local recurrence. Desmoid tumors are best treated with radical surgical excision to prevent morbidity or disfigurement from recurrence.

A 35-year-old, otherwise healthy woman presents with a mass over her right chest. What is not a useful part of the initial clinical evaluation of a primary chest wall mass? A) Obtain a history B) Order a chest x-ray C) Order magnetic resonance imaging (MRI) D) Perform a physical examination E) Obtain tissue for pathologic evaluation

To understand the clinical evaluation of chest wall masses Answer B is correct Chest x-ray is of little use in the evaluation of chest wall masses. The initial evaluation of patients with chest wall masses begins with a careful history and a complete physical examination. An MRI is useful to characterize primary chest wall masses. To determine definitive therapy, tissue diagnosis can be helpful.

A patient underwent laparoscopic Nissen fundoplication for GERD. Ten weeks after surgery, the patient presents to the clinic complaining of recurrent heartburn. Which is not a next step in management? A) Restart PPIs B) Esophageal manometry assessment C) Postoperative pH monitoring D) Barium swallow E) Endoscopic evaluation

To understand the evaluation of foregut symptoms after laparoscopic antireflux surgery Answer A is correct During the first 6 to 8 weeks after surgery, dysphagia, bloating, and abdominal discomfort may be present. If symptoms persist or heartburn and regurgitation occur, a thorough evaluation with barium swallow, endoscopy, esophageal manometry, and pH monitoring is carried out. As postoperative pH monitoring yields abnormal results in only about 20% of patients, acid-reducing medications should not simply be restated.

Which of the following factors do not affect the type of operation offered to a patient for rectal prolapse? Please choose the single most appropriate answer to the question A) A patient's age B) A patient's physical condition C) The amount of rectum that is prolapsed D) A previous history of hemorrhoids E) The degree of fecal incontinence

To understand the factors involved in the decision-making process for selecting the appropriate rectal prolapse procedure Answer D is correct The choice of operation is determined by the patient's age, sex, level of operative risk, associated pelvic floor defects, degree of incontinence, and history of constipation, as well as by the surgeon's experience. The goal is to correct the greatest number of anatomic problems (including the prolapse and any associated functional disorders) safely and efficiently while minimizing both perioperative morbidity and postoperative recurrence. The best way to think of prolapse operations is to divide them into broad categories of abdominal and perineal. The balance that must be discussed with the patient is the balance between immediate safety and long-term results. The abdominal approaches, including rectopexy, resection rectopexy, and other choices, whether by laparotomy or laparoscopy, tend to have lower recurrence rates and better functional outcomes than do the perineal methods. Conversely, the perineal approaches tend to have less intraoperative and postoperative morbidity and mortality but higher rates of recurrence and inferior functional outcomes compared with the abdominal approaches. Therefore, major considerations to selection of the operation are the age and overall physical condition of the patient. In general, young, healthy patients are far better served by an abdominal procedure as they can better tolerate any potential intraoperative or postoperative morbidity and have a longer life expectancy. Therefore, the best possible function over the longest possible period of time is of paramount importance. The preferred operation then is a laparoscopically assisted resection rectopexy to optimize function, decrease recurrence potential, and minimize incisional length. Elderly, frail patients may best be served by a perineal approach to limit intra-abdominal trauma. The perineal procedure preferred is the perineal rectosigmoidectomy. If only a minor amount of prolapse can be delivered, then a Delorme procedure is performed. In our experience, the Delorme procedure has a higher recurrence rate and far less improvement in continence than does the perineal rectosigmoidectomy. Therefore, the Delorme procedure is infrequently undertaken.

A 23-year-old woman with Crohn disease since age 17 is now 3 months pregnant. In a pregnant patient with Crohn disease, all of the following statements are true except: A) Metronidazole is the preferred antibiotic to treat disease flares B) Pregnancy does not exacerbate Crohn disease C) Preferred imaging modalities include ultrasonography and magnetic resonance imaging D) Cesarean section is advisable if there is a risk of obstetrically related sphincter trauma E) Prednisone is safe for use

To understand the impact pregnancy may have on the management of patients with Crohn disease Answer A is correct There is no evidence that pregnancy exacerbates Crohn disease; however, some specific concerns apply to pregnant patients with this condition. Because patients with Crohn disease often have bowel movements that are more liquid, they have a particular need for a well-functioning anal sphincter. If there is any chance of an obstetrics-related injury (e.g., from a large baby in a primagravida or from a breech presentation), a cesarean section is advisable to minimize the risk of sphincter trauma. The same is true in the presence of severe perianal Crohn disease. During pregnancy, prednisone and 5-acetylsalicylic acid medications are safe, whereas drugs such as metronidazole are not. If imaging studies are needed, magnetic resonance imaging and ultrasonography are the modalities of choice

A 39-year-old woman underwent total thyroidectomy 3 years ago for papillary thyroid cancer. She is noncompliant with her thyroid replacement hormone. She now reports weight gain, lethargy, and weakness. Thyroid studies are sent. Which of the following results would you expect in this patient? A) Low triiodothyronine (T3), low thyroxine (T4), low thyroid-stimulating hormone (TSH) B) Low T3, high T4, high TSH C) High T3, high T4, low TSH D) Low T3, low T4, high TSH E) High T3, low T4, low TSH

To understand the laboratory evaluation of patients with thyroid abnormalities Answer D is correct Patients who have undergone thyroid surgery, particularly total thyroidectomy, are susceptible to hypothyroidism. Typical symptoms include weight gain, lethargy, and weakness, as well as fatigue, cold intolerance, constipation, dry skin, and depressed mood. In patients with these symptoms, thyroid function tests should be checked. Normally, the thyroid secretes T3 and T4 in response to TSH, which is secreted by the pituitary gland. T3 and T4 then provide feedback inhibition to the hypothalamic-pituitary axis, regulating TSH release. In hypothyroidism, T3 and T4 release by the thyroid is low, resulting in low serum levels. TSH is elevated because of the lack of feedback inhibition from T3 and T4.

A 53-year-old woman with a history of cancer presents with a chest wall mass. A tissue specimen obtained for pathologic examination suggests that the mass is not a primary chest wall mass. What is the most likely source of this secondary chest wall mass? A) Esophageal cancer B) Colon cancer C) Thyroid cancer D) Breast cancer E) Uterine cancer

To understand the likely source of secondary chest wall masses Answer D is correct Secondary chest wall masses arise as direct extension of a malignancy from a contiguous organ. Breast and lung cancer are the most common secondary chest wall masses.

A 63-year-old man with chronic renal failure and diabetes has a chronic ulcer of the right heel that has not healed after multiple revascularization attempts. Amputation is being considered. Which of the following is false regarding below-the-knee amputation? A) Flexure contracture at the knee is easily corrected with a prosthetic limb. B) The tibia should be divided no more than 1 cm proximal to the anterior skin incision. C) A below-the-knee amputation is likely to heal if the patient has a palpable femoral pulse and a patent deep femoral artery. D) The length of the posterior flap should equal one third of the circumference of the leg measured 10 cm distal to the tibial tuberosity. E) The fibula should be divided cephalad to the division of the tibia.

To understand the planning, technique, and outcomes from below-the-knee amputation Answer A is correct Below-the-knee amputation is indicated when the lower extremity is functional but the foot cannot be salvaged by arterial reconstruction or by amputation of one or more of the toes or the forefoot. Healing can be expected if there is a palpable femoral pulse with at least a patent deep femoral artery provided that the skin is warm and free of lesions at the distal calf. Before formal below-the-knee amputation, infection should be controlled with antibiotic therapy, débridement, and, if indicated, guillotine amputation. It is advisable to obtain consent to possible above-the-knee amputation beforehand in case unexpected muscle necrosis is encountered below the knee. The lines of incision should be marked on the skin. The primary level of amputation is determined by measuring a distance of 10 cm from the tibial tuberosity. The circumference of the leg at this level is then measured by passing a heavy ligature around the leg and cutting the ligature to a length equal to the circumference. The ligature is folded into thirds and cut once more at one of the folds, so that two segments of unequal length remain. The longer segment of the ligature, which is equal in length to two thirds of the leg's circumference 10 cm below the tibial tuberosity, is used to measure the anterior transverse incision; this incision is centered not on the tibial crest but on the gastrocnemius-soleus muscle complex. The shorter segment, which is one third of the leg's circumference at this level, is used to measure the posterior flap; the line of the posterior incision runs parallel to the gastrocnemius-soleus complex. The tibia is scored circumferentially, and a periosteal elevator is used to dissect the soft tissues away from the tibia for a distance of approximately 3 to 4 cm. The tibia is then transected just proximal to the transverse skin incision. Dividing the tibia more than 1 cm proximal to the anterior skin incision will cause the thin skin of the anterior leg to be pulled taut over the cut end of the tibia by the weight of the posterior flap, thereby leading to skin ulceration. The tibia is transected perpendicularly, with a cephalad bevel of the anterior 1 cm to keep from creating a sharp point at the tibial crest. A periosteal elevator is used to dissect the soft tissues away from the fibula to a point 2 to 3 cm cephalad to the level at which the tibia was transected. The fibula is then transected with a bone cutter at least 1 cm cephalad to the tibial transection level. To walk with a prosthetic leg, the patient must be capable of fully extending and locking the knee; thus, flexion contracture at the knee is a major complication. Such contractures are usually attributable either to poor pain control or to noncompliance with knee extension exercises. Good perioperative analgesia is of vital importance because knee flexion is the position of comfort and the patient will be unwilling to extend the knee if doing so proves too painful. To maintain knee extension, the patient should be placed in a splint in the early postoperative period. Once postoperative pain has abated, the splint can be removed. At this point, the patient must be taught extension exercises, in which the quadriceps muscles are contracted to maintain the length of the hamstring muscles. If a patient spends all of his or her time in a sitting position with the knee flexed, a flexion contracture will quickly develop. Once this happens, the patient may find it very difficult to regain full knee extension, and without full knee extension, prosthetic limb rehabilitation is impossible.

A 63-year-old man with chronic renal failure and diabetes has a chronic ulcer of the right heel that has not healed after multiple revascularization attempts. Amputation is being considered. Which of the following is false regarding below-the-knee amputation? A) Flexure contracture at the knee is easily corrected with a prosthetic limb. B) The tibia should be divided no more than 1 cm proximal to the anterior skin incision. C) A below-the-knee amputation is likely to heal if the patient has a palpable femoral pulse and a patent deep femoral artery. D) The length of the posterior flap should equal one third of the circumference of the leg measured 10 cm distal to the tibial tuberosity. E) The fibula should be divided cephalad to the division of the tibia.

To understand the planning, technique, and outcomes from below-the-knee amputation Answer A is correct Below-the-knee amputation is indicated when the lower extremity is functional but the foot cannot be salvaged by arterial reconstruction or by amputation of one or more of the toes or the forefoot. Healing can be expected if there is a palpable femoral pulse with at least a patent deep femoral artery provided that the skin is warm and free of lesions at the distal calf. Before formal below-the-knee amputation, infection should be controlled with antibiotic therapy, débridement, and, if indicated, guillotine amputation. It is advisable to obtain consent to possible above-the-knee amputation beforehand in case unexpected muscle necrosis is encountered below the knee. The lines of incision should be marked on the skin. The primary level of amputation is determined by measuring a distance of 10 cm from the tibial tuberosity. The circumference of the leg at this level is then measured by passing a heavy ligature around the leg and cutting the ligature to a length equal to the circumference. The ligature is folded into thirds and cut once more at one of the folds, so that two segments of unequal length remain. The longer segment of the ligature, which is equal in length to two thirds of the leg's circumference 10 cm below the tibial tuberosity, is used to measure the anterior transverse incision; this incision is centered not on the tibial crest but on the gastrocnemius-soleus muscle complex. The shorter segment, which is one third of the leg's circumference at this level, is used to measure the posterior flap; the line of the posterior incision runs parallel to the gastrocnemius-soleus complex. The tibia is scored circumferentially, and a periosteal elevator is used to dissect the soft tissues away from the tibia for a distance of approximately 3 to 4 cm. The tibia is then transected just proximal to the transverse skin incision. Dividing the tibia more than 1 cm proximal to the anterior skin incision will cause the thin skin of the anterior leg to be pulled taut over the cut end of the tibia by the weight of the posterior flap, thereby leading to skin ulceration. The tibia is transected perpendicularly, with a cephalad bevel of the anterior 1 cm to keep from creating a sharp point at the tibial crest. A periosteal elevator is used to dissect the soft tissues away from the fibula to a point 2 to 3 cm cephalad to the level at which the tibia was transected. The fibula is then transected with a bone cutter at least 1 cm cephalad to the tibial transection level. To walk with a prosthetic leg, the patient must be capable of fully extending and locking the knee; thus, flexion contracture at the knee is a major complication. Such contractures are usually attributable to either poor pain control or noncompliance with knee extension exercises. Good perioperative analgesia is of vital importance because knee flexion is the position of comfort and the patient will be unwilling to extend the knee if doing so proves too painful. To maintain knee extension, the patient should be placed in a splint in the early postoperative period. Once postoperative pain has abated, the splint can be removed. At this point, the patient must be taught extension exercises, in which the quadriceps muscles are contracted to maintain the length of the hamstring muscles. If a patient spends all of his or her time in a sitting position with the knee flexed, a flexion contracture will quickly develop. Once this happens, the patient may find it very difficult to regain full knee extension, and without full knee extension, prosthetic limb rehabilitation is impossible.

A 76-year-old woman has biopsy-proven esophageal cancer. As the next step in the diagnostic evaluation, an endoscopic sonogram is ordered. What is not a benefit of EUS in this context? A) Permits determination of the depth of tumor invasion and tumor stage B) Permits fine-needle aspiration biopsies C) Permits evaluation of adjacent lymphadenopathy D) Permits evaluation of esophageal wall and surrounding tissue E) Permits detection of metastatic lesions

To understand the role of endoscopic ultrasonography (EUS) in the management of esophageal cancer Answer E is correct EUS is a diagnostic modality that allows for assessment of the esophageal wall and surrounding tissues. This permits the characterization of esophageal masses (depth of invasion, T stage) and an assessment of adjacent lymphadenopathy (N stage); EUS also guides endoscopic fine-needle aspiration biopsies. CT with PET is necessary to detect other metastatic lesions.

A 39-year-old woman presents with a mass in her left thyroid lobe. Ultrasonography reveals this to be 2 cm in size, and fine-needle aspiration confirms a PTC. What is the appropriate treatment? A) Enucleation B) Left thyroid lobectomy C) Left thyroid lobectomy with ipsilateral cervical lymphadenectomy D) Total thyroidectomy E) Total thyroidectomy with ipsilateral cervical lymphadenectomy

To understand the surgical management of papilllary thyroid carcinoma (PTC) Answer D is correct Total thyroidectomy should be performed for most PTCs exceeding 1 cm; this procedure addresses multicentricity, maximizes the therapeutic impact of postoperative radioactive iodine (RAI, 131I), and facilitates surveillance for recurrences. Lobectomy suffices for small, intrathyroidal PTCs discovered incidentally in glands resected for other diagnoses when radiation exposure, nodal disease, and poor prognostic variants are absent. The favorable natural history of PTC and the morbidity of lateral cervical lymphadenectomy suggest that this procedure be added only for pathologically proven lateral nodal disease.

A 58-year-old woman is referred to you with a neck mass. You suspect that she has hyperthyroidism. Which of the following is not a symptom of hyperthyroidism? A) Palpitations B) Constipation C) Tremor D) Weight loss E) Hyperphagia

To understand the symptoms of hyperthyroidism Answer B is correct. Hyperthyroidism can often present in patients with a neck mass. Thyroid function tests (TSH, T3, T4) should be checked in all patients with a thyroid mass. Patients with hyperthyroidism will have elevated T4 and, typically, T3 (although T3 is less sensitive for the diagnosis). TSH may or may not be elevated, depending on the etiology of the increased thyroid function. Hyperthyroidism results in an increase in metabolic function, resulting in symptoms of sweating, heat intolerance, heart palpitations, tremor, hyperphagia, weight loss, increased thirst, and insomnia. Constipation is a common symptom of hypothyroidism.

A 39-year-old woman underwent total thyroidectomy 3 years ago for papillary thyroid cancer. She is noncompliant with her thyroid replacement hormone. She now reports weight gain, lethargy, and weakness. Thyroid studies are sent. Which of the following results would you expect in this patient? A) Low triiodothyronine (T3), low thyroxine (T4), low thyroid-stimulating hormone (TSH) B) Low T3, high T4, high TSH C) High T3, high T4, low TSH D) Low T3, low T4, high TSH E) High T3, low T4, low TSH

understand the laboratory evaluation of patients with thyroid abnormalities Answer D is correct Patients who have undergone thyroid surgery, particularly total thyroidectomy, are susceptible to hypothyroidism. Typical symptoms include weight gain, lethargy, and weakness, as well as fatigue, cold intolerance, constipation, dry skin, and depressed mood. In patients with these symptoms, thyroid function tests should be checked. Normally, the thyroid secretes T3 and T4 in response to TSH, which is secreted by the pituitary gland. T3 and T4 then provide feedback inhibition to the hypothalamic-pituitary axis, regulating TSH release. In hypothyroidism, T3 and T4 release by the thyroid is low, resulting in low serum levels. TSH is elevated because of the lack of feedback inhibition from T3 and T4.


Set pelajaran terkait